Internal Medicine Practice Exam

Ace your homework & exams now with Quizwiz!

A 44-year-old woman with a history of chronic pancreatitis secondary to alcohol abuse presents to the clinic for routine care. She reports her symptoms are well-controlled. She was last seen a year ago. Which of the following tests is indicated for this patient? A. Abdominal ultrasound B. Fasting glucose level C. Magnetic resonance cholangiopancreatography D. Serum amylase and lipase

A fasting glucose level is indicated for this patient as she has a history of chronic pancreatitis, which greatly elevates her risk for the development of diabetes, specifically diabetes type 3c. Chronic pancreatitis results in scar tissue deposition in and around the pancreas. Reduced exocrine and endocrine functioning often result as the disease progresses. Chronic pancreatitis may present with abdominal pain, bloating, nausea, vomiting, weight loss, diarrhea, and steatorrhea with recurrent exacerbations. Laboratory assessment may reveal elevated serum amylase and lipase, though these are often normal in chronic pancreatitis, as well as diminished stool elastase testing, which assesses pancreatic exocrine functioning. Testing for complications should be conducted, with emphasis on screening for diabetes, electrolyte disturbances, and vitamin deficiencies, both fat-soluble vitamins as well as cobalamin, folic acid, and thiamine. A complete blood count is often normal unless a vitamin deficiency occurs. Either an annual fasting glucose or hemoglobin A1C test is advised to screen for diabetes. Patients with chronic pancreatitis also have an increased risk for the development of pancreatic cancer, therefore significant symptoms may indicate the need for further work-up. Bone mineral density testing may also be advisable. Diagnostic imaging may include plain films, CT scan, endoscopic ultrasound, magnetic resonance cholangiopancreatography, endoscopic retrograde cholangiopancreatography. Pancreatic calcifications on abdominal X-ray strongly suggest the diagnosis of chronic pancreatitis but are seen in only 30% of patients. CT scan is often the initial study of choice and may show calcifications not seen on plain radiographs. Magnetic resonance cholangiopancreatography (MRCP) is becoming the diagnostic test of choice. It can demonstrate calcifications and pancreatic duct obstruction consistent with chronic pancreatitis with no radiation risk involved. No routine imaging is required once the diagnosis is made and a good response to treatment is noted. Treatment consists of eliminating alcohol intake, smoking cessation, pancreatic enzyme replacement, low-fat diet, insulin (if diabetes is present) and pain control. Surgery or endoscopic treatment with stenting confer benefits for complications and may reduce pain. Complications of chronic pancreatitis include nausea, vomiting, malnutrition, pseudocyst formation, biliary adhesions or obstruction, gastric outlet obstruction, diabetes, bleeding, splenic or portal vein thrombosis, pseudoaneurysm, osteoporosis or osteopenia, and pancreatic cancer. Abdominal ultrasound (A) may be implemented in the evaluation of acute pancreatitis, but has no value as a screening test in chronic pancreatitis. It may also be used for long-term monitoring of a stable pancreatic pseudocyst. Magnetic resonance cholangiopancreatography (C) may be considered as an initial evaluation test for chronic pancreatitis, but it is not recommended as a screening test. Early changes in chronic pancreatitis are more likely to appear on magnetic resonance imaging compared to CT or X-ray. Serum amylase and lipase (D) are not indicated as screening tests but may be useful in evaluating acute pancreatitis.

A 54-year-old man presents to the clinic with a complaint of a productive cough and progressive dyspnea on exertion. His cough was initially dry, 12 years ago, but progressed to productive one year ago. He denies any smoking history, and has otherwise been healthy. He is a mason by trade and enjoys stone sculpting with his wife as a hobby. Which of the following is most likely to confirm the suspected diagnosis? A. Chest X-ray B. Complete blood count with differential C. Pulmonary function test D. Tuberculin skin test

A. Chest X-ray A chest X-ray is most likely to confirm the suspected diagnosis of pneumoconiosis, specifically silicosis. Pneumoconioses are a group of restrictive chronic lung diseases caused by inhalation of particulate matter and are usually related to occupational exposure

A 42-year-old woman presents with right upper quadrant abdominal pain that began four hours prior to arrival in the emergency department. She has associated nausea and has had one episode of emesis. An ultrasound is ordered of the right upper quadrant which shows pericholecystic fluid and thickening of the gallbladder wall. Which of the following signs or symptoms would be consistent with this patient's diagnosis? A. Abdominal distention B. Jaundice C. Right scapular pain D. Shaking chills

Acute cholecystitis is most often caused by obstruction of the cystic duct. Pain from acute cholecystitis typically involves the right upper quadrant of the abdomen and can be referred to the right scapular area. Patients may complain of nausea and have associated emesis although this is usually not severe. On physical exam, patients may have a tender, palpable gallbladder. Palpating the right upper quadrant while having the patient take a deep inspiratory breath may elicit pain (positive Murphy's sign). The preferred imaging study is a right upper quadrant ultrasound which can demonstrate pericholecystic fluid, gallbladder wall thickening, and gallstones. If the ultrasound is inconclusive, a radionuclide excretion scan (HIDA scan) may be ordered to examine the function of the gallbladder. The treatment of cholecystitis is a cholecystectomy; however, the timing of surgery is somewhat controversial. Most surgeons agree that the gallbladder should be removed within three days of the onset of symptoms to prevent scarring and other complications. Abdominal distention (A) can occur with gallstone ileus but is typically not a sign of acute cholecystitis. Mild jaundice (B) only occurs in about 10% of patients with acute cholecystitis. This finding is more common in those with cholangitis and choledocholithiasis. Shaking chills (D) and a high fever are uncommon and suggest another diagnosis such as cholangitis or emphysematous cholecystitis.

A 70-year-old male is beginning treatment for peripheral vascular disease. He has a 50 pack year smoking history and is currently smoking one pack per day. His ankle-brachial index is 0.76, and his dorsalis pedis pulses are diminished bilaterally. His main complaint is a reduction in his ability to continue morning walks with his brother due to pain in his calves that resolves with rest. Which of the following is the most appropriate therapy? A. Aspirin, cilostazol, rosuvastatin, smoking cessation, and structured exercise B. Aspirin, rosuvastatin, smoking cessation, and surgical referral C. Clopidogrel, endovascular reperfusion, rosuvastatin, and smoking cessation D. Clopidogrel, structured exercise, and smoking cessation

Aspirin, cilostazol, rosuvastatin, smoking cessation, and structured exercise represent the core medical treatment regimen for peripheral vascular disease, also called peripheral artery disease. In the majority of cases, progressive atherosclerosis of the peripheral arteries leads to a reduction in perfusion to the limbs via luminal reduction. Clinical manifestations may include muscular atrophy, decreased hair growth, thick toenails, and decreased skin temperature. Ischemic ulcerations (on the toes, lateral malleolus) may be present in more advanced cases. Diagnosis is usually established with a history of limb pain (intermittent claudication or rest pain), diminished or absent pulses, and an ankle-brachial index (ABI) of 0.9 or less, or a toe-brachial index of 0.7 or less, if significant vascular calcification is present. Additional diagnostic studies may include segmental pressure and pulse volume recordings, exercise testing, and vascular imaging. Angiography is the gold standard for diagnosing and locating peripheral vascular disease. Factors such as medications, activity, smoking status, glycemic control, blood pressure, and renal function affect progression and complications of the disease. Treatment consists of antiplatelet medication (clopidogrel or aspirin), structured exercise, smoking cessation, treatment of hypertension and hyperglycemia if present, a statin (reduces mortality and may reduce atheroma size), and cilostazol, a phosphodiesterase inhibitor, for claudication. Surgery or endovascular treatment may be indicated when symptoms remain severe despite optimal medical therapy, if ulceration or gangrene occurs, or when acute ischemia occurs. Acute ischemia is always an emergency, as the risk of amputation is high. There is also a high occurrence of coronary artery disease, therefore patients diagnosed with peripheral vascular disease should have a thorough assessment to identify concomitant disease requiring ongoing assessment and treatment. Aspirin, rosuvastatin, smoking cessation, and surgical referral (B) does not include medication to address claudication or functional improvement (exercise), and plans surgical involvement prior to initiating optimal medical treatment. There is no evidence that early surgery or endovascular treatment provides a better outcome than medical management alone. Clopidogrel, endovascular reperfusion, rosuvastatin, and smoking cessation (C) does not address claudication symptoms or functional improvement and plans endovascular treatment prior to optimal medical treatment. Clopidogrel, structured exercise, and smoking cessation (D) fails to include statin medication or treatment for claudication.

What is the most common presenting symptom of bladder carcinoma? A. Nocturia B. Painless hematuria C. Unintentional weight loss D. Urinary frequency

B. Painless hematuria The primary modality to diagnose bladder cancer is with cystoscopy, which allows for direct visualization of tissues and biopsy of any abnormalities. Smoking is the most important risk factor of bladder cancer.

A 28-year-old woman presents to the clinic with worsening symptoms related to her recently diagnosed ulcerative colitis. What would you expect her signs or symptoms to include? A. Bloody diarrhea B. Fistula formation C. Mouth sores D. Night sweats

Bloody diarrhea is the classic finding in ulcerative colitis. This can occur in Crohn's disease as well, especially if the rectum is involved, but it is a more consistent finding with ulcerative colitis. Others symptoms include crampy abdominal pain, tenesmus, and loss of appetite. Ulcerative colitis is an inflammatory bowel disease that only affects the colon and spares the rest of the digestive system. Diagnosis is made by biopsy during colonoscopy. On biopsy, ulcerative colitis only affects the innermost lining (mucosal layer) of the colon. Crohn's disease, another inflammatory bowel disease, affects all layers of the gastrointestinal tract and can span the entire gastrointestinal tract from the mouth to the anus. Fistula formation (B) is when a new tract is formed off of an area of the GI tract and leading out of the body, most commonly an area near the anus. This can occur with Crohn's disease but not ulcerative colitis. Mouth sores (C) can occur with Crohn's disease (mouth to anus), but the mouth is not involved in ulcerative colitis. Night sweats (D) have been reported with Crohn's disease but is not a typical finding in ulcerative colitis.

A 67-year-old man presents to the clinic for evaluation of cough, wheezing, malaise, and shortness of breath. His social history is positive for daily cigar smoking for 30 years. He reports his symptoms have been present for four months. He had a similar presentation one year ago and was diagnosed with bronchitis. Which of the following is most likely on physical examination? A. Barrel chest B. Diastolic murmur C. Pectus carinatum D. Scaphoid abdomen

Barrel chest, or an increased anteroposterior diameter of the thorax, is a common finding in chronic obstructive pulmonary disease (COPD). The primary disease processes contributing to COPD are asthma, chronic bronchitis, and emphysema. The underlying pathology is not completely understood, but the main underlying effect is disordered airway inflammation and a high concentration of elastases and proteases. Inhaled toxins, particularly tobacco, are strongly correlated to the development of COPD, however, any chronic airway irritant may be contributory, as well as other medical conditions, such as alpha-1 antitrypsin deficiency. Chronic bronchitis is characterized histopathologically with mucous gland hyperplasia and a largely intact vascular bed, leading to high circulation but poor oxygenation. Emphysema is characterized by hyperexpansion of alveoli due to loss of elastic recoil, loss of alveolar wall structure (surface area), and subsequent loss of pulmonary vasculature, leading to poor circulation and poor oxygenation. Asthma is a condition with many possible pathophysiologic changes but is outlined by airway obstruction, inflammation, and bronchial hyperresponsiveness. Most cases of COPD involve varying degrees of emphysema, chronic bronchitis, and asthma, and culminate in air trapping and poor oxygenation. Formal diagnosis includes a thorough history and physical examination, as well as pulmonary function testing (PFT). Chronic bronchitis is suggested by two episodes of bronchitis within two years, each lasting three months or longer. Emphysema often presents as progressive shortness of breath and often with accompanying weight loss due to the increased effort of ventilation. Physical exam findings may include barrel chest, decreased breath sounds, hyperresonance to percussion, prolonged expiration, and often wheezing and rhonchi. Poor oxygenation is often reflected in clubbing of the digits, tachypnea, and in severe cases, altered mental status. Some patients may have features more consistent with either chronic bronchitis, such as cyanosis (blue bloater), or emphysema, such as a pink complexion (pink puffer), though these changes are noted late in the disease course. A diagnosis of COPD can be made using PFT if the ratio of forced expiratory volume in one second to forced vital capacity is less than 70%. Radiography suggests COPD, and often includes hyperinflation, flattened diaphragms, and increased bronchovascular markings. Treatment is multifaceted and includes inhaled corticosteroids, anticholinergics, bronchodilators, as well as smoking cessation, antibiotics during COPD exacerbations, phosphodiesterase inhibitors, vaccinations, and supplemental oxygen in select patients, which is proven to extend life. Care must be taken with the administration of oxygen, however, as this can lead to profound loss of the respiratory drive. Administration of alpha-1 antitrypsin may be indicated for COPD secondary to a deficiency. Surgical options for symptomatic relief and quality of life enhancement include bullectomy, lung volume reduction, and lung transplantation. Pulmonary rehabilitation is also proven to be beneficial. Complications from COPD include pulmonary hypertension, heart failure, frequent respiratory infections, and respiratory failure, which will occur as the disease progresses. Diastolic murmur (B) is not directly related to COPD, however, pulmonic murmurs may be found, especially in the presence of cor pulmonale. Pectus carinatum (C) is a protrusion of the sternum outward and is not associated with COPD. Pectus excavatum, or inward projection of the sternum, may alter pulmonary function. Scaphoid abdomen (D) is a sunken or concave contour of the abdomen, and may be seen in emphysema, but is unlikely in a patient with chronic bronchitis.

A 20-year-old woman with a history of cystic fibrosis presents for evaluation of a two year history of multiple episodes of what she describes as "bronchitis". She complains of chronic cough productive of foul-smelling sputum. Auscultation reveals crackles at the lung bases. Chest X-ray is notable for airway dilatation and thickening. Which of the following is the most likely diagnosis? A. Asthma B. Bronchiectasis C. Chronic bronchitis D. Emphysema

Bronchiectasis is characterized by abnormal, permanent dilation and destruction of bronchial walls. Multiple etiologies can contribute to the development of bronchiectasis, including airway obstruction, cystic fibrosis (CF), recurrent infections, primary ciliary dyskinesia, rheumatic and systemic diseases, humoral immunodeficiency, and cigarette smoking. Cystic fibrosis is the most common cause of bronchiectasis. The classic clinical manifestations of bronchiectasis are cough with large amounts of mucopurulent, foul-smelling sputum, dyspnea, rhinosinusitis, hemoptysis, and pleuritic chest pain. Symptoms may last months to years. There is generally a past history of repeated respiratory tract infections over several years, although a single episode of severe bacterial pneumonia, pertussis, or tuberculosis may result in bronchiectasis. Physical examination findings include crackles, wheezing, and rarely digital clubbing. Chest X-ray may reveal dilated and thickened airways described as "tram-tracks". High resolution CT is the diagnostic study of choice revealing airway dilatation,"signet-ring sign" (the jewel on the ring), bronchial wall thickening, and mucopurulent plugs and debris accompanied by post-obstructive air trapping (tree-in-bud). Treatment consists of antibiotics for acute exacerbations, bronchodilators, hydration and chest physiotherapy. The classic signs and symptoms of asthma (A) are intermittent dyspnea, cough, and wheezing. It is diagnosed before the age of seven years in approximately 75% of cases. Symptoms occur following exposure to triggers and resolve with trigger avoidance or asthma medications. Chronic bronchitis (C) is defined as a chronic productive cough for at least three months per year for at least two successive years in patients in whom other causes of chronic cough have been excluded. It is commonly seen in cigarette smokers. Imaging is useful to exclude alternative diagnoses or evaluate for complications. Emphysema (D) causes abnormal and permanent enlargement of the airspaces distal to the terminal bronchioles that is accompanied by the destruction of the airspace walls. It is commonly seen in cigarette smokers. Both chronic bronchitis and emphysema are subtypes of chronic obstructive pulmonary disease (COPD). The most common symptoms of COPD are dyspnea, chronic cough, and sputum production. Exertional dyspnea is the most common early symptom of COPD.

A 60-year-old man with a history of hypertension and heavy tobacco use has been compliant with his 80 mg dose of atorvastatin for the past twelve months. His current total cholesterol measurement is 220 mg/dL. You decide to add another lipid lowering agent. Which of the following drugs has proven beneficial for the prevention of adverse cardiovascular disease events when added to statin therapy? A. Cholestyramine B. Ezetimibe C. Fenofibrate D. Niacin

Cardiovascular disease includes coronary artery disease, peripheral vascular disease, or cerebrovascular disease. Modifiable risk factors for cardiovascular disease include smoking, obesity, diabetes mellitus, sedentary lifestyle, hypertension, and hypercholesterolemia. Diagnosis of cardiovascular disease can be made using several different modalities, depending on the presentation, including measurement of serum markers, stress testing, angiography, carotid ultrasound, MRI and CT scan. Treatment is tailored to the specific disease. Lifestyle modifications involve weight loss, aerobic exercise, low sodium diet, low fat diet, and smoking cessation. Medical treatment includes treating risk factors such as hypertension, diabetes mellitus, and dyslipidemia. Statin drugs are indicated for patients with established cardiovascular disease or with a ten-year cardiovascular risk assessment (using an online risk calculator which takes into account age, race, total cholesterol, high-density lipid cholesterol, smoking status, and comorbidities such as hypertension and diabetes) which is greater than 7.5 percent. Two drugs have demonstrated the ability to cause regression in the size of atheromas. For this reason, rosuvastatin and atorvastatin are recommended as first-line therapy for patients with cardiovascular disease or with increased risk for cardiovascular disease who have no contraindications to statin drugs. If a patient is unable to obtain a total cholesterol under 200 mg/dL on high dose rosuvastatin or atorvastatin therapy, adjunctive medications can be added. The only non-statin lipid-lowering agent that has proven to have additive effects on the prevention of cardiovascular adverse events is ezetimibe, which inhibits intestinal absorption of cholesterol. Ezetimibe is administered in an oral daily dose of 10 mg. Common side effects include diarrhea and cough. Cholestyramine (A) is a bile acid sequestrant which increases fecal loss of low-density lipoprotein. Indications for use of cholestyramine include hyperlipidemic patients who are unable to take statin drugs. Cholestyramine is not indicated for use as monotherapy in patients with hypertriglyceridemia. Common side effects of cholestyramine include constipation and malabsorption of fat-soluble vitamins. Fenofibrate (C) is a cholesterol lowering agent whose mechanism of action is to increase catabolism of very-low-density lipoproteins and increase elimination of triglycerides by enhancing synthesis of lipoprotein lipase. Fenofibrate is indicated for the treatment of hypertriglyceridemia and for treatment of mixed dyslipidemia. Statins are the only lipid lowering agents which have proven to decrease mortality, however, fenofibrate can be used in patients who are not able to tolerate a statin drug. Niacin (D) is vitamin B3, which is a necessary component in lipid metabolism. Niacin has been shown to lower total cholesterol, low-density lipoprotein, and triglycerides and is indicated as adjunctive therapy in the treatment of severe hypertriglyceridemia and in combination with a bile acid sequestrant for treatment of hyperlipidemia. However, niacin has not been shown to clinically reduce morbidity and mortality when combined with a statin drug. Flushing is a significant side effect of niacin and reduces its tolerability considerably.

A 37-year-old woman with a two-year history of rheumatoid arthritis presents to the clinic for worsening joint pain. Previously her disease had been well controlled on 10 mg weekly of methotrexate. You decide to advance her dose to 20 mg weekly to help control her synovitis. What changes in her health maintenance might you have to make with this change? A. Check a complete blood count and blood chemistry profile within six weeks B. Decrease how much folic acid she is supplementing C. Have her screened for tuberculosis before increasing her methotrexate D. Referral to eye provider for a baseline visual field test

Check a complete blood count (CBC) and blood chemistry profile (CMP) within six weeks when making a change in the dose of methotrexate. Methotrexate is known to cause liver toxicity and requires frequent monitoring. Also, reduction in cell counts can occur including white blood cells, red blood cells, and platelets with methotrexate use. Methotrexate is also a folate inhibitor and increasing the patient's folic acid supplementation may be needed. Having the patient return to follow up within six weeks is recommended to check for side effects of the drug, including mouth sores, hair loss, and gastrointestinal (GI) upset occurring over the following 24-48 hours after dosing. Methotrexate is available in both oral and injectable form, with lower incidences of GI upset with the injections. Decreasing how much folic acid she is supplementing (B) would worsen any potential side effects. Signs of hair loss and mouth sores are common side effects pointing to a folate deficiency. Typically patients are started on 1 mg daily of folic acid when methotrexate is introduced and, if side effects are present, can be increased up to 5 mg daily. Have her screened for TB before increasing her methotrexate (C) is not required, as there is no causal relationship. This does need to be done, however, before starting most biologic agents for rheumatoid arthritis. Referral to eye provider for baseline visual field test (D) is not needed for methotrexate but is helpful before starting hydroxychloroquine, another rheumatic disease modifying agent, for comparison in the event that changes to the retina are seen on future exams.

A 65-year-old man presents to the clinic reporting fever, nausea, diffuse joint pains, and a rash. He states his symptoms began four days ago after a camping trip in Tennessee. He denies any sore throat, cough, vomiting, diarrhea, or itching. Physical examination reveals a painless maculopapular rash on his wrists and palms, diffuse joint tenderness, and mild upper abdominal tenderness. His temperature is 100.5°F, but the rest of his vital signs are within normal limits for his age. Which of the following is the most likely diagnosis? A. Cellulitis B. Lyme disease C. Rocky Mountain spotted fever D. Syphilis

Rocky Mountain spotted fever is an infection caused by Rickettsia rickettsii, a gram-negative coccobacillus. The name is misleading, as most cases are diagnosed in the central and southeastern United States. The primary vector that harbors R. rickettsii is the dog tick (Dermacentor variabilis). Endothelial cells are the primary target for infection, which eventually causes a varying degree of vasculitis. Confirmatory testing is via serology, however, it may take weeks for seroconversion to occur. Additionally, blood tests may indicate hemolysis, anemia, thrombocytopenia, hyponatremia, azotemia, and elevated aminotransferase levels. Skin biopsy may also indicate infection, however, this test is not routinely performed. A clinical diagnosis can be made if there is a history of tick exposure or travel to an endemic area and fever, myalgias, and a rash. Classically, the rash appears as a blanching maculopapular rash around the wrists and ankles and has a centripetal spread toward the body. It is one of the few diseases that can cause a rash on the palms and soles. The rash often begins as maculopapular, becoming petechial as the disease progresses. It is important to remember the rash may not occur, and having a high clinical suspicion is important to establish a correct diagnosis. The treatment of choice is doxycycline, even in children, as the risks of untreated disease are greater than the risks of medication adverse events, specifically dental staining and hepatotoxicity in the mother. Alternatively, chloramphenicol may be used if an allergy to doxycycline is present. Treatment should commence prior to obtaining results from confirmatory tests. Complications of Rocky Mountain spotted fever are numerous and include: encephalopathy, seizures, neuropathy, myocarditis, acute respiratory distress syndrome, acute kidney injury, disseminated intravascular coagulation, and even death. Cellulitis (A) is an infection of the dermis and subcutaneous tissues that presents as a poorly defined, tender, indurated area of erythema. It is most commonly caused by streptococci and staphylococci. Lyme disease (B) is an infection caused by Borrelia burgdorferi, a spirochete, and is associated with a target lesion, or erythema migrans, at the location of a bite from an infected deer tick. Syphilis (D) is a sexually transmitted disease caused by the spirochete Treponema pallidum. Secondary syphilis may present with a maculopapular rash on the palms and soles, but may also present with lymphadenopathy and condyloma lata, which characteristically become grey as they resolve.

A 23-year-old African American woman, with no risk factors for heart disease, presents to the clinic with complaints of sternal chest pain, fatigue, nonproductive cough, and shortness of breath. Vitals and physical exam are normal. Chest X-ray reveals bilateral hilar adenopathy and diffuse reticular opacities. Which of the following is most likely to confirm the diagnosis? A. Endobronchial lung biopsy B. Positron emission tomography scan C. Pulmonary function tests D. Surgical biopsy of mediastinum and lung

Endobronchial lung biopsy is recommended for patients with suspicious chest X-ray findings (i.e., bilateral hilar adenopathy and diffuse, ground glass, reticular opacities). Biopsy is performed on the most accessible lesion that appears to be affected (skin, lungs). Flexible bronchoscopy should be performed looking for endobronchial lesions to biopsy. In sarcoidosis, endobronchial mucosal biopsies are frequently positive and may increase the diagnostic yield, compared to transbronchial biopsies alone. Sarcoidosis is a multisystem granulomatous disorder characterized pathologically by the presence of noncaseating granulomas in involved organs (lungs, skin, lymph nodes, eyes, kidneys, joints). It is more common in African American women < 40 years of age. Common symptoms include cough, sternal chest pain, dyspnea, fatigue, malaise, fever, weight loss, muscle weakness, and exercise intolerance. Patients should also be asked about new skin lesions, visual changes, dry eyes or mouth, parotid swelling, palpitations, syncope, joint pain or swelling, or muscle weakness. Systemic corticosteroids are the treatment of choice for sarcoidosis. Positron emission tomography scan (B) may be helpful in identifying occult lesions that are more accessible to biopsy than lung lesions, however, it does not differentiate between sarcoidosis and malignancy. Pulmonary function tests (C) are used to assess the severity of respiratory impairment of sarcoidosis but not used for confirmation of diagnosis. Surgical biopsy of mediastinum and lung (D) may be needed if endobronchial lung biopsy is not successful in yielding a diagnosis, but the least invasive procedure is tried first.

A 77-year-old woman presents to the clinic complaining of recent onset of headaches. She denies any visual changes and vitals are normal. On exam, she is tender over the right temporal area. She follows with cardiology for a history of a heart valve replacement and with rheumatology for a disease that "has a long name" which she can't remember. Which of the following is the most likely diagnosis? A. Cluster headache B. Fibromyalgia C. Giant cell arteritis D. Trigeminal neuralgia

Giant cell arteritis, or temporal arteritis, is the most common vasculitis of large and medium-sized vessels. It is more commonly seen in women > 50 years of age. Clinical manifestations may include constitutional symptoms (fatigue, weight loss, low-grade fever), headaches, claudication of jaw when chewing, visual impairment (may lead to blindness), tenderness over temporal artery. These symptoms are especially suspicious of temporal arteritis in patients with a history of polymyalgia rheumatica. Typically, giant cell arteritis is associated with high erythrocyte sedimentation rate and high serum C-reactive protein. Diagnosis is confirmed by biopsy of the temporal artery, which shows an autoimmune inflammatory response. If a diagnosis of temporal arteritis is suspected, immediate treatment with high-dose steroids (prednisone) is required to prevent blindness. Scheduling of temporal artery biopsy should never interfere with the initiation of prednisone in a patient with a high likelihood of temporal arteritis. Cluster headache (A) is more common in men, typically younger men between 20-40 years of age. It can cause temporal, but more often periorbital tenderness, accompanied by eye watering, nasal congestion, or swelling around the eye. Fibromyalgia (B) can cause headaches and tender points but is not associated with temporal tenderness. It occurs much more often in women. Trigeminal neuralgia (D) manifests as a brief (seconds to minutes), episodic, unilateral, stabbing facial pain in the distribution of one or more divisions of the Trigeminal nerve (CN V). Pain is worse with chewing, talking, brushing teeth, cold air, smiling, and grimacing.

A 64-year-old man presents to the clinic for management of chronic symptomatic hypercalcemia of malignancy. Despite the continued use of bisphosphonates, his total calcium level remains elevated. Which of the following is the most appropriate medication this patient should receive next? A. Calcitonin B. Cinacalcet C. Denosumab D. Zoledronic acid

Hypercalcemia occurs when the entry of calcium into circulation exceeds the excretion of calcium into urine or the deposition of calcium into bone. Two of the most common causes of hypercalcemia are primary hyperparathyroidism (commonly caused by a parathyroid adenoma) and malignancy. Clinical manifestations vary. For patients with mild hypercalcemia (serum total calcium level of less than < 12 mg/dL), symptoms are often vague and can include constipation, fatigue, and depression. Moderate hypercalcemia, defined as total serum calcium levels of 12 to 14 mg/dL, may cause additional symptoms such as polyuria (most common renal manifestation), polydipsia, dehydration, muscle weakness, and altered sensorium. Severe hypercalcemia is diagnosed when serum total calcium levels rise above 14 mg/dL and can be associated with lethargy, stupor, and even coma. Treatment of mild hypercalcemia and asymptomatic moderate hypercalcemia focuses on avoidance of aggravating factors, such as thiazide diuretics and volume depletion. Treatment for acute hypercalcemia is reserved for patients who present with either altered sensorium or severe hypercalcemia and includes volume expansion with isotonic saline, administration of calcitonin, and concurrent administration of zoledronic acid or pamidronate (bisphosphonates). Ultimately, the underlying disease causing the hypercalcemia should be treated. In patients with chronic hypercalcemia due to malignancy, it is recommended to use intravenous zoledronic acid or pamidronate to prevent long term skeletal complications. In the case of a patient with chronic symptomatic hypercalcemia of malignancy that is refractory to bisphosphonate therapy, denosumab, a monoclonal antibody which inhibits bone resorption via inhibition of receptor activator of nuclear factor kappa-B ligand (RANKL) can be used. In addition, denosumab is an option for patients in whom bisphosphonates are contraindicated due to severe renal impairment, as it is not renally excreted. Calcitonin (A) is used in the acute treatment of hypercalcemia. It works to reduce the serum calcium concentration by increasing renal calcium excretion and decreasing bone resorption by interfering with osteoclast function. The use of calcitonin is only beneficial during the first 48 hours of severe hypercalcemia and thus plays no role in long-term management. Cinacalcet (B) is a calcium-sensing receptor agonist which reduces parathyroid hormone. It is used acutely to treat hypercalcemia caused by parathyroid carcinoma and secondary hyperparathyroidism in chronic kidney disease. Zoledronic acid (D) is a bisphosphonate, which works by inhibiting bone resorption through interference with osteoclast recruitment and function. While this is used to treat both acute hypercalcemia and the preferred agent for treating chronic hypercalcemia, the patient has already failed treatment with a bisphosphonate.

A 50-year-old man with a history of asthma and hypertension presents to the emergency department with shortness of breath, wheezing, and cough. He reports poor compliance with his medications due to lack of access to a primary care provider. He denies chest pain. His vital signs in triage are 36.9°C, HR 87, BP 240/140, RR 22. His physical examination reveals mild wheezing, bibasilar rales, and an S3. His ECG shows normal sinus rhythm. His laboratory workup is significant for a BNP of 360 ng/L and a troponin of 0.3 ng/mL. Which of the following is the most likely cause of the patient's symptoms? A. Acute thrombotic myocardial ischemia B. Asthma exacerbation C. Hypertensive emergency D. Pulmonary embolism

Hypertensive emergency is an acute elevation of blood pressure associated with end-organ damage, typically to the kidneys, brain, heart, aorta, or eyes. It results from the eventual overwhelming or failure of compensatory mechanisms for end-organ perfusion in the setting of increased mean arterial pressure. This leads to fibrinoid necrosis and end-organ hypoperfusion and ischemia. It is most commonly seen in patients with chronic hypertension who are noncompliant with medications. Blood pressure control is the cornerstone of management for all hypertensive emergencies, but the method of blood pressure control differs based both on the etiology of the hypertensive emergency and the evidence of end-organ damage. Hypertensive pulmonary edema is most commonly due to systolic or diastolic left ventricular dysfunction. Vasodilator and diuretics are the mainstay of therapy in hypertensive pulmonary edema. Nitroglycerin is the preferred antihypertensive agent in the setting of pulmonary edema. A mild troponin elevation may be present in a hypertensive emergency due to myocardial hypoperfusion in the setting of increased mean arterial pressure. This is rarely due to an acute thrombotic event (A), especially in a patient who does not have chest pain. An asthma exacerbation (B) presents with shortness of breath and wheezing but it is important to remember that not all wheezing is asthma. Asthma exacerbations do not typically present with jugular venous distention and S3, nor are they associated with elevations in BNP or troponin. Pulmonary embolism (D) is a cause of shortness of breath, and, if massive, can result in right heart failure. Tachycardia and tachypnea may be seen but are not diagnostic. Bibasilar rales are typically not heard on exam.

Which of the following laboratory values is most likely to be elevated in a patient with hypertonic hyponatremia? A. Serum glucose B. Serum potassium C. Serum sodium D. Serum triglycerides

Hyponatremia is defined as a serum concentration of sodium less than 135 mEq/L. Most cases of hyponatremia are related to disordered renal excretion of water, although the causes of hyponatremia are many. Hyponatremia can be divided into three main categories: isotonic (pseudohyponatremia), hypertonic, hypotonic, with the most common form being hypotonic (true hyponatremia). Patients with hypotonic hyponatremia may have renal failure, congestive heart failure, liver failure, syndrome of inappropriate antidiuretic hormone, hypothyroidism, gastrointestinal fluid losses, or may be taking drugs such as thiazide diuretics or ecstasy. Patients with isotonic hyponatremia (pseudohyponatremia) have an artificially low serum sodium concentration associated with a normal serum osmolality due to marked elevations in lipids or proteins, resulting in a reduction in the water-containing fraction of serum. Patients with hypertonic hyponatremia have an increased concentration of serum solutes which do not readily cross the cell membrane. This increases tonicity and draws water into the serum, diluting the serum sodium concentration. Examples of processes which cause increases in serum solutes would be hyperglycemia, intravenous immune globulin or mannitol administration, or absorption of surgical irrigation solutions such as glycine, sorbitol or mannitol. The complete metabolic panel can be very useful in diagnosing and differentiating causes of hyponatremia, along with a detailed history and physical exam. If no apparent diagnosis presents itself after these tests, the next step would be to obtain a urinary sodium and chloride concentration and osmolality. Once an etiology of the hyponatremia has been discovered, treatment can be directed at the specific disease process. In hyponatremia concurrent with hyperglycemia, a corrected serum sodium should be estimated by remembering that the serum sodium concentration decreases by about 2 mEq/L with every 100 mg/dL increase in glucose concentration. To avoid osmotic demyelination syndrome, hyponatremia of any cause should never be corrected by more than 8 mEq/L in a 24-hour period. Serum potassium (B) may be elevated in patients with the syndrome of inappropriate antidiuretic hormone (SIADH). These patients often have increased serum potassium and decreased serum sodium as a result of antidiuretic hormone activity on the renal tubules. As a result of increased water reabsorption and increased sodium excretion, patients with this syndrome normally have hypotonic hyponatremia with euvolemia. Serum sodium (C) would not be elevated in a patient with hyponatremia, by definition. As previously mentioned, serum sodium in patients with hyponatremia should not be rapidly or overly corrected, as this may result in osmotic demyelination syndrome, where brain cell volume shrinks in response to correction of hyponatremia and demyelination results, which can be permanent. Serum triglycerides (D), when severely elevated, can cause a pseudohyponatremia, not a hypertonic hyponatremia. The pseudohyponatremia results from falsely low serum sodium concentrations in the presence of hyperlipidemia, hypertriglyceridemia, or hyperproteinemia when measured with certain analyzers. These analyzers assume a serum water concentration of 93%, which is not the case in patients with high levels of triglycerides, lipids, or proteins. Analyzers with direct ion-selective electrodes will not show this artifact and will render an accurate serum sodium concentration.

A 26-year-old man presents to the clinic four hours after receiving benzathine penicillin G for primary syphilis. He states he has sweating after resolution of a brief fever two hours after his injection. Physical examination reveals diaphoresis, a significantly more pronounced chancre, tender lymphadenopathy, and mild tachycardia. His vital signs are as follows: temperature of 100.2°F, heart rate of 108 beats per minute, blood pressure 104/60 mm Hg, and respiratory rate of 24 breaths per minute. Which of the following is the best treatment option? A. Ceftriaxone B. Diphenhydramine C. Dobutamine D. Ibuprofen

Ibuprofen is a treatment option for the Jarisch Herxheimer reaction, a phenomenon that classically occurs within the first 24 hours after beginning treatment for any spirochetal infection, including syphilis. The exact mechanism of the reaction remains unknown at this time. Diagnosis is made clinically. Symptoms in an immunocompetent patient usually begin within eight hours of initiation of antibiotic therapy, however, disease type and host immune factors may alter both the onset and duration of the reaction. Presentation includes fever, chills, malaise, headache, tender lymphadenopathy, hyperventilation, decreased blood pressure, and worsening of lesions caused by the infection. Treatment is usually symptomatic, with acetaminophen or ibuprofen being the most commonly used medications. If a severe reaction occurs, appropriate care is oriented to the nature of the complication, such as intubation and mechanical ventilation if respiratory compromise occurs. Complications are most often related to constitutional symptoms and brief exacerbation of cutaneous lesions. Serious complications are related to exacerbation of localized infection, such as degradation of gummas, or heart failure from cardiovascular syphilis. Pregnancy presents a unique risk for the development of complications from syphilis treatment, as a Jarisch Herxheimer reaction may precipitate early labor, however, this risk does not preclude treatment of syphilis. Ceftriaxone (A) is a third-generation cephalosporin that may be used for many different types of infections, including sepsis. The presentation of sepsis is usually progressive and therefore less likely in this scenario, as the patient reports resolution of his fever. Diphenhydramine (B) may be useful for allergic reactions, but this patient has no evidence of an allergic response, such as urticaria, dyspnea, or hypotension. Dobutamine (C) is a vasopressor that is indicated for the treatment of cardiac decompensation in shock. While it may be used for extreme reactions, its use is not indicated in this scenario as the patient has a mild presentation of a Jarisch Herxheimer reaction.

A 60-year-old man presents to the clinic with a high-pitched, blowing systolic murmur heard best at the apex and radiating to his left axilla. Which of the following is the most likely diagnosis? A. Aortic stenosis B. Mitral regurgitation C. Pulmonic stenosis D. Tricuspid regurgitation

Mitral regurgitation occurs when the leaflets of the mitral valve do not fully close during systole, allowing retrograde flow of blood from the left ventricle to the left atrium. Causes of mitral valve regurgitation include rheumatic heart disease, ruptured chordae tendineae, focal myocardial dysfunction, and congenital valvular disease. Patients with chronic mitral valve regurgitation may suffer fatigue, dyspnea, or palpitations. Longstanding mitral valve regurgitation can lead to increased intracardiac pressure and left ventricular dysfunction, with a hyperdynamic left ventricle and a third heart sound (S3) on auscultation. Mitral regurgitation is associated with a high-pitched, blowing systolic murmur heard best at the apex and radiating to the left axilla, as well as a wide splitting of S2 due to early closure of the aortic valve. Transesophageal echocardiography is the preferred modality for definitive diagnosis of mitral valve regurgitation. Treatment consists of agents that decrease afterload, such as nitrates, beta-blockers, calcium-channel blockers and angiotensin-converting enzyme inhibitors. Anticoagulant therapy may become necessary if atrial fibrillation develops, and valve replacement can be considered in patients who are surgical candidates and have clinically severe disease. Patients who present with mitral valve regurgitation in an acute setting should be evaluated for myocardial infarction, as this is a common cause of acute mitral valve regurgitation. Aortic stenosis (A) will result in a harsh systolic murmur best auscultated at the right sternal border and there will be an absent or delayed aortic component of the second heart sound due to calcified and immobile aortic valve leaflets. Aortic stenosis occurs when the aortic valve becomes calcified and no longer fully opens during systole. The only definitive treatment is aortic valve replacement. Pulmonic stenosis (C) presents with a systolic crescendo-decrescendo murmur best auscultated at the left sternal border and which increases in intensity with inspiration. Most cases are congenital and diagnosed in infancy. The majority of patients with pulmonic stenosis are asymptomatic but may experience exertional dyspnea. Treatment consists of percutaneous balloon valvuloplasty. Tricuspid regurgitation (D) produces a high-pitched pansystolic murmur which is loudest at the fourth intercostal space in the parasternal region and is often associated with an S3 gallop. Tricuspid valve regurgitation can be caused by rheumatic heart disease, papillary muscle dysfunction, right ventricular dilatation, connective tissue diseases, medications, endocarditis, or trauma. Treatment of tricuspid valve regurgitation depends on the etiology and may include management of left-sided heart failure, antidysrhythmics, annuloplasty or valve replacement.

A 40-year-old man presents to the clinic with burning abdominal pain that extends from his navel to his chest for the past couple of days. Loss of appetite is his only other complaint. Physical exam is normal. He smokes one pack of cigarettes per day and drinks two to three beers daily. An ulceration in the gastric lining is seen on upper endoscopy. What is the most likely diagnosis? A. Barrett esophagus B. Gastric cancer C. Peptic ulcer disease D. Ulcerative colitis

Peptic ulcer disease results from an increase in digestive acid production or loss of protection of the stomach or proximal small intestine lining, leading to either gastric or duodenal ulcers depending upon location. It is associated with two major factors, Helicobacter pylori infection and nonsteroidal anti-inflammatory use. Burning epigastric pain is by far the most common symptom and may be the only symptom, however, the majority of cases are asymptomatic. Other common symptoms are early satiety, fatty food intolerance, heartburn, nausea, and sometimes hematemesis. Definitive diagnosis is made by endoscopy, where ulcers can be visually confirmed and biopsy performed on the edges of suspicious ulcers. Treatment usually consists of discontinuation of offending agents: smoking, alcohol use, medications such as nonsteroidal anti-inflammatories, and offending foods. In addition, appropriate diagnosis and treatment for Helicobacter pylori infection, the leading cause of peptic ulcer disease, is necessary and tested for before any invasive diagnostics or treatment. Reduction of acid production with proton pump inhibitors is also a standard of care. Barrett esophagus (A) usually presents in patients with a history of frequent heartburn and gastroesophageal reflux disease. Chest pain is common, rather than epigastric, but may be asymptomatic. Endoscopy would show columnar epithelium in the distal esophagus and biopsy demonstrating intestinal metaplasia would confirm the diagnosis. Gastric cancer (B) may be present in this patient, especially if the edges of his ulcer are overhanging, irregular, or thickened, but this is not the most likely diagnosis. Presence of Helicobacter pylori increases the risk of cancer developing. Ulcerative colitis (D) is an inflammatory disease of the large intestine and would thus present with lower abdominal complaints. Bloody diarrhea is the hallmark sign of the disease.

Which of the following scenarios is most likely to result in a metabolic acidosis? A. Bacterial pneumonia with underlying chronic obstructive lung disease B. Hyperventilation due to anxiety C. Persistent diarrhea D. Vomiting

Persistent diarrhea causes loss of bicarbonate in the stool and a resulting metabolic acidosis. Metabolic acidosis is usually confirmed with arterial blood gas (ABG) test, which would show a low pH and low bicarbonate level. To help determine the underlying cause of the metabolic acidosis, the anion gap is calculated. The anion gap is the difference between the major cations and anions, calculated as Na+ - (Cl- + HCO3-). Normal values are 8 to 14 mEq/L. Normal anion gap metabolic acidosis is commonly caused by severe diarrhea resulting in loss of bicarbonate. Other causes include renal tubular acidosis, early renal failure, and carbonic anhydrase inhibitors. Increased anion gap metabolic acidosis causes can be remembered using the common mnemonic MUDPILES. These causes are methanol, uremia, diabetic ketoacidosis, propylene glycol, isoniazid intoxication, lactic acidosis, ethanol ethylene glycol, and salicylates. Hyperventilation due to anxiety (B) is a cause of respiratory alkalosis. Pneumonia in a patient with chronic obstructive lung disease (A) is a cause of respiratory acidosis. Vomiting (D) is a cause of metabolic alkalosis.

A 32-year-old woman presents to the clinic for evaluation of palpitations and is diagnosed with Graves' disease. Which of the following is the most appropriate therapy for acute management of her symptoms? A. Dexamethasone B. Methimazole C. Propranolol D. Propylthiouracil

Propranolol is a nonselective beta-adrenergic receptor antagonist that diminishes the response of cardiomyocytes to the effects of thyroid hormone. Thyroid hormone induces increased adrenergic receptor density and tone, causing amplification of the body's response to adrenergic stimuli. The most common cause of hyperthyroidism in the United States is a diffuse goiter, or Graves' disease, an autoimmune disorder associated with thyroid-stimulating immunoglobulin antibodies directed at thyrotropin receptors (TRab) on the surface of thyroid cells, which stimulates excessive production and release of thyroid hormone. Symptoms typically include tremor, tachycardia or palpitations, anxiety, weight loss, heat intolerance, pretibial myxedema, and ophthalmopathy (exophthalmos). Diagnosis is established with laboratory testing, most often revealing low thyrotropin (thyroid stimulating hormone) and elevated thyroid hormone, particularly T3, elevated thyrotropin receptor antibodies, and occasionally anti-thyroid peroxidase antibodies. Scintigraphy (radioiodine scan) is commonly used and often reveals diffuse uptake, but it is contraindicated in pregnancy. Ultrasound is the preferred imaging study in pregnant patients. Treatment with adrenergic blockade blunts many of the systemic signs and symptoms of excessive circulating thyroid hormone. Hyperthyroidism may be treated with antithyroid medication (methimazole or propylthiouracil), radioactive iodine, or thyroidectomy, depending on the etiology, severity, and circumstances at presentation. While methimazole and propylthiouracil both act to decrease the production of thyroid hormone, propylthiouracil may also prevent the conversion of T4 to the more active form of thyroid hormone T3. Due to potential fetotoxicity of methimazole, propylthiouracil is usually prescribed in pregnant patients during the first trimester. In nonpregnant patients, methimazole is the preferred agent. Complications from Graves' disease can include ophthalmopathy, heart failure, cardiac conduction abnormalities, dermopathy, dehydration, and hypothyroidism. Dexamethasone (A) is a glucocorticoid that may be used to treat severe thyroiditis, but has no utility in treating Graves' disease. Methimazole (B) is an antithyroid medication that is appropriate to treat Graves' disease by lowering thyroid hormone production, however, it is not indicated to treat acute symptoms. Propylthiouracil (C) is another antithyroid medication which may be implemented to treat Graves' disease but does not treat acute symptoms. Additionally, use of propylthiouracil has declined, as it carries a greater risk of hepatitis, aplastic anemia, vasculitis, and agranulocytosis.

A 44-year-old woman presents to the clinic for evaluation of a syncopal episode that occurred while walking her dog two days ago. She denies amnesia or head trauma. She has had increasing dyspnea on exertion and pedal edema. Physical examination reveals clubbing of her fingers and central cyanosis. Auscultation of the heart reveals tricuspid insufficiency, widely split second heart sound with a palpable P2. Echocardiogram reveals a large ostium secundum atrial septal defect with bidirectional flow. Which of the following is the most likely diagnosis? A. Acute pericarditis B. Atrial myxoma C. Pulmonary hypertension D. Takotsubo cardiomyopathy

Pulmonary hypertension is usually associated with symptoms of chest pain, dyspnea, fatigue, edema and occasionally syncope or ascites. Physical examination reveals a splitting of S2 with a loud pulmonic component. Fingernail clubbing and cyanosis may be present due to underlying pulmonary disease. Increased resistance within the pulmonary circulation causes the right ventricle to work harder and eventually enlarge in response. Other changes that may occur are right atrial enlargement, decreased left ventricular cavity size, and tricuspid regurgitation. Tricuspid regurgitation will result in increased jugular pulsation in the neck, along with a palpable venous pulse in the liver. ECG will show right axis deviation and RV hypertrophy. Another lab finding may include polycythemia secondary to chronic hypoxemia. Treatment consists of oxygen administration, sodium restriction, and prescription of diuretics, as well as treatment of any underlying pulmonary condition. Acute pericarditis (A) shows diffuse ST segment elevation with depression of the PR interval on ECG. Atrial myxoma (B) is characterized by fever, weight loss, malaise, embolization, and a diastolic murmur on examination. Takotsubo cardiomyopathy (D) (known as "broken heart syndrome") is commonly seen in postmenopausal women, who experience signs and symptoms of acute coronary syndrome. Echocardiogram shows left ventricular apical dyskinesia.

A 30-year-old man presents to the clinic reporting joint pain and pain with urination. He states that symptoms started two weeks ago. He has left knee and right ankle pain with mild limitations in range of motion secondary to stiffness. Additionally, he has significant bilateral conjunctivitis, and edema at the urethral meatus, but no discharge. His vital signs are all within normal limits. Which of the following is the most likely diagnosis? A. Reactive arthritis B. Rheumatoid arthritis C. Septic arthritis D. Systemic lupus erythematosus

Reactive arthritis, formerly called Reiter syndrome, is a systemic inflammatory disease, characterized as an aseptic spondyloarthropathy. The disease occurs after exposure to enteric (salmonella, shigella, campylobacter, yersinia, E. coli, clostridia) or genitourinary (chlamydia) infections. The pathogenesis is not entirely understood but involves autoimmune and inflammatory processes. Symptoms are often more severe in patients with human immunodeficiency virus (HIV) coinfection. Classically, the disease is described as a triad of urethritis, arthritis, and conjunctivitis, however, most patients do not present with all three components. Onset of symptoms occurs one to six weeks after infection. Constitutional symptoms are common, especially early in the disease course. Urethritis may present with a discharge. A crusty, pustular psoriasiform rash may arise, especially around the glans (circinate balanitis). Articular involvement primarily affects the lower extremities, particularly the ankles and knees, and is often an asymmetric monoarthritis or oligoarthritis. Enthesopathy of the Achilles tendon and plantar fascia, as well as dactylitis, may be noted. Anterior uveitis, hypopyon, or even macular edema, in addition to conjunctivitis, may be seen on the eye exam. Very rarely cardiovascular structures may be involved in chronic or severe disease. Reactive arthritis is a clinical diagnosis, however, testing for chlamydia and HIV should be performed in all newly diagnosed cases. Additional tests may include human leukocyte antigen B27 (positive in 30-50% of patients), stool cultures (if diarrhea is present), antineutrophil antibody, rheumatoid factor, erythrocyte sedimentation rate, C-reactive protein, complete blood count, joint fluid analysis, hepatic, and renal function tests. Early reactive arthritis rarely produces any abnormality on X-rays. Nonsteroidal anti-inflammatories are the cornerstone of arthritis treatment, but if ineffective, glucocorticoids (systemic or intra-articular) may be used. Disease-modifying antirheumatic drugs, such as methotrexate or sulfasalazine, may be used in the setting of severe or long-term reactive arthritis. Prednisolone eye drops may be used to treat iritis. Treatment for reactive arthritis caused by chlamydia may include doxycycline or azithromycin, however, antibiotic treatment of enteric infections is controversial. Complications are related to affected tissues and may include joint destruction, cardiovascular complications, ankylosing spondylitis, glaucoma, urethral stricture, and macular edema. Rheumatoid arthritis (B) is also an autoimmune arthritis but is not associated with urethritis or conjunctivitis, and usually presents with a prolonged, gradual onset. Septic arthritis (C) may be a diagnostic consideration, but this patient's symptoms have been present for two weeks, which would be more than enough time for severe symptoms and total destruction of the joints to occur. Systemic lupus erythematosus (D) is an autoimmune disease that causes serositis, and may have some overlapping features with reactive arthritis, but is often associated with a malar rash, vasculitis, Raynaud's phenomenon, photosensitivity, and additional abnormal autoantibody testing (anti-dsDNA, anti-Sm).

A 23-year-old man presents to the emergency department after being stabbed in the flank during a bar fight. He received 2 liters of normal saline by EMS. On arrival, his vital signs are T 37.2°C, HR 130, BP 75/50, RR 30. He appears anxious and diaphoretic. On examination, he has a stab wound to the right flank, abdominal distention, no sensation below the level of T10, weakness of bilateral lower extremities, poor peripheral pulses and cool extremities. Which of the following is the most likely cause of this patient's hemodynamic instability? A. Hypovolemic shock B. Neurogenic shock C. Obstructive shock D. Spinal shock

Shock is defined as a state of circulatory insufficiency that results in an imbalance of oxygen delivery and metabolic demand. Shock, by definition, results in tissue hypoperfusion and end-organ dysfunction. Shock may occur with or without hypotension and it is vital to remember that there is no single vital sign or laboratory value that is diagnostic for shock. The four broad categories of shock are hypovolemic, cardiogenic, obstructive, and distributive. The presentation above is most consistent with hypovolemic shock. Hypovolemic shock occurs when decreased intravascular volume, either through hemorrhage or nonhemorrhagic fluid loss, causes decreased cardiac output, an imbalance in oxygen supply and demand, and organ hypoperfusion. This decreased cardiac output leads to compensatory tachycardia to try to maintain stroke volume and an increase in systemic vascular resistance. Arterial and venous vasoconstriction lead to a narrowing of pulse pressure, a sign that is present well before the onset of hypotension. Patients with hemorrhagic shock present with signs of poor peripheral perfusion, such as weak peripheral pulses, paleness, poor capillary refill, and cool extremities. Cerebral hypoperfusion may lead to subtle mental status changes including anxiety and confusion. Early intervention leads to a sharp decrease in the high morbidity and mortality associated with hypovolemic shock. Treatment of hypovolemic shock is founded on expanding intravascular volume, maintaining the oxygen-carrying capacity of the blood, and stopping ongoing hemorrhage. Intravascular expansion can be accomplished through transfusion of crystalloid and blood products. Neurogenic shock (B) is a type of distributive shock that occurs due to loss of sympathetic innervation, leading to extreme vasodilation from loss of sympathetic arterial tone. If spinal cord levels T1-T4 are damaged, loss of sympathetic innervation to the heart leads to bradycardia from unopposed vagal tone. This blocks the reflex tachycardia that would otherwise compensate for hypotension. Patients in neurogenic shock are typically hypotensive, bradycardic, and peripherally vasodilated, with well-perfused, warm extremities and good capillary refill. Neurogenic shock occurs in less than 20% of traumatic spine injuries and should be a diagnosis of exclusion in trauma patients. Obstructive shock (C) results from a decrease in venous return or cardiac compliance due to a left ventricular outflow obstruction or a significant decrease in preload, as seen in cardiac tamponade or a massive pulmonary embolism. Spinal shock (D) should not be confused with neurogenic shock and occurs due to the loss of spinal reflex activity below a complete or incomplete spinal cord injury. Patients typically present with flaccidity, loss of movement, and lack of deep tendon reflexes.

A 40-year-old woman is found to have a malar rash . Which laboratory test has the highest sensitivity for this medical condition? A. Anti-double-stranded DNA B. Anti-nuclear antibody C. Cyclic citrullinated peptide antibody D. Erythrocyte sedimentation rate

Systemic lupus erythematosus (SLE) is a chronic inflammatory disease affecting multiple organ systems as a result of immune system dysregulation. It is characterized by the formation of autoantibodies which can damage tissues directly or combine with corresponding antigens to form tissue-damaging immune complexes. Autoantibodies that have been identified in systemic lupus erythematosus are anti-nuclear antibodies (ANA), including anti-deoxyribonucleic acid (anti-DNA) antibodies, anti-Smith (Sm) antibodies, and antiphospholipid antibodies. Anti-nuclear antibodies test has 95% sensitivity and is the best initial screening tool. The specificity of anti-nuclear antibodies for systemic lupus erythematosus is low since it is reported in disorders other than systemic lupus erythematosus (Rheumatoid arthritis, scleroderma, polymyositis, dermatomyositis, Sjogren's syndrome, mixed connective tissue disease). Anti-double-stranded DNA and anti-Smith antibodies have high specificity and are highly diagnostic for systemic lupus erythematosus. Anti-Smith is the most specific antibody for systemic lupus erythematosus. Anti-double-stranded DNA test is fairly specific as well and it can be used to monitor the severity of the disease in patients who have been diagnosed with systemic lupus erythematosus. In particular, this test may be used to monitor lupus nephritis. Erythrocyte sedimentation rate (D) is a nonspecific test that has been used to detect inflammation associated with conditions such as infections, cancers, and autoimmune diseases. Elevated levels of ESR indicate the presence of the infection but doesn't tell you where the source of infection is or what causes it. Anti-double-stranded DNA (A) test is ordered when a patient has signs and symptoms that could be due to lupus and has had a positive ANA test. Anti-double-stranded DNA is fairly specific for lupus. High level of anti-double-stranded DNA in the blood is strongly associated with lupus, especially when anti-Smith test is also positive. Cyclic citrullinated peptide antibody (C) is used along with rheumatoid factor to diagnose rheumatoid arthritis.

A 42-year-old woman presents to the clinic with a history of stage I differentiated papillary thyroid carcinoma that was treated five years ago with thyroidectomy and had an excellent clinical response. The patient requests monitoring for recurrent thyroid disease. A serum thyroid-stimulating hormone (TSH) level is ordered. Which of the following is the recommended range to maintain in this patient? A. < 0.1 mU/L B. 0.1-0.5 mU/L C. 0.1-2.0 mU/L D. 0.5-5.0 mU/L

Thyroid malignancies can be divided broadly into five categories: primary thyroid lymphoma, medullary thyroid cancers, undifferentiated thyroid cancer, differentiated thyroid cancers, and metastases of other cancers to the thyroid. Differentiated thyroid cancers fall into two main groups: follicular and papillary, with papillary being the more common of the two. Diagnosis of thyroid cancer is made with ultrasound, fine-needle biopsy, and serum studies. Treatment of differentiated thyroid cancer can involve total or partial thyroidectomy and thyroid-stimulating hormone suppression with levothyroxine (T4) replacement, as well as radioiodine therapy in certain patients. Differentiated thyroid cancer is classified using the TNM classification system, which takes into account the patient's age, the tumor size and extension, regional lymph node involvement, and presence of metastasis. A patient under 55 years of age with stage I differentiated papillary thyroid carcinoma would have no distant metastasis and a favorable prognosis. Patients who have finished treatment for differentiated thyroid cancer should receive neck ultrasonography at six to twelve month intervals, as well as serum thyroid-stimulating hormone levels and serum thyroglobulin levels every three to six months during the first year post-treatment. Some higher-risk patients may also require full-body radioiodine scanning one year post-treatment. After the one year mark, patients who initially presented with low-risk disease and who had an excellent clinical response to treatment can be managed by maintaining a serum thyroid-stimulating hormone (TSH) level of 0.1-2.0 mU/L. Patients who initially present with differentiated thyroid cancer and have a structurally incomplete response to therapy (a portion of the cancerous mass remains) should have their serum thyroid-stimulating hormone level maintained below 0.1 mU/L (A). Patients who present with extensive disease but have an excellent or indeterminate clinical response to therapy should be maintained at a serum thyroid-stimulating hormone level of 0.1-0.5 mU/L (B). For patients with a structurally complete but biochemically incomplete response to therapy, the serum thyroid-stimulating hormone level should also be maintained at 0.1-0.5 mU/L. A range of 0.5-5.0 mU/L (D) includes values which are expected for a patient who has no prior history of thyroid cancer. In patients who are beyond one year post-treatment and who initially present with high-risk or low-risk differentiated thyroid cancer, the serum thyroid-stimulating hormone level should be measured annually, or six to eight weeks after any adjustments in levothyroxine (T4) replacement therapy.

A 23-year-old man presents for evaluation of a painless right-sided scrotal mass that he noted for the first time while showering one week ago. On physical examination, the mass is located on the anterior surface of the testicle. It is soft on palpation and transilluminates. Which of the following is the most likely diagnosis? A. Hydrocele B. Spermatocele C. Testicular cancer D. Varicocele

A hydrocele is a collection of peritoneal fluid between the parietal and visceral layers of the tunica vaginalis, which directly surrounds the testis and spermatic cord. Idiopathic hydroceles are most common and generally arise over a long period of time. Inflammatory conditions of the scrotal contents (epididymitis, orchitis, testicular torsion) can produce an acute reactive hydrocele. Hydroceles are most often painless, soft masses on the anterior or lateral surface of the testicle. They transilluminate well. Diagnosis is typically clinical; however, a scrotal ultrasound should be considered if the diagnosis is uncertain. Unless patients are symptomatic with pain or pressure sensation, intervention is generally not required. The most common intervention, if needed, is excision of the hydrocele sac. A spermatocele (B) is an epididymal cyst containing sperm (> 2 cm in size). A spermatocele is generally asymptomatic and palpated as a soft, round mass in the head of the epididymis, on the superior aspect of the testicle. It will transilluminate easily. Treatment is typically not indicated unless a patient is experiencing chronic pain. Testicular cancer (C) is the most common solid tumor in men between the ages of 15 and 35 years. It presents as a painless testicular mass. On exam, testicular cancer is usually a firm, fixed, nontender nodule or mass that does not transilluminate. Scrotal ultrasound is the test of choice to differentiate testicular mass from a hydrocele or epididymitis. A varicocele (D) is caused by dilatation of the pampiniform plexus of spermatic veins. It most commonly occurs on the left side. Patients may be asymptomatic or may present with dull, aching scrotal pain. Varicoceles are generally diagnosed by characteristic physical findings, which range from minimal left-sided scrotal fullness on Valsalva maneuver to a large, soft, left-sided scrotal mass that decompresses and disappears in the recumbent position. It may feel like a "bag of worms" on palpation. Varicoceles are often identified in infertile men. Intervention typically is not indicated.

A woman presents to the clinic complaining of pedal edema. Which of the following would be most suggestive of congestive heart failure as the likely etiology? A. History of bulimia nervosa B. History of chronic renal insufficiency C. History of diabetes mellitus D. History of inflammatory bowel disease

Congestive heart failure involves ventricular dysfunction and can affect the right ventricle, left ventricle, or both. Heart failure may be due to filling defects, poor contractility, or a combination of the two. The percent of blood expelled from the ventricle during systole is measured as the ejection fraction, which is diminished in patients with systolic heart failure. In patients with diastolic heart failure, ventricular filling is impaired by a stiff ventricle, pericarditis, or valvular disease. Patients at increased risk of heart failure are those with a history of myopathy, familial heart disease, rheumatic heart disease, hyperthyroidism, pheochromocytoma, dyslipidemia, diabetes mellitus, hypertension, sleep apnea, peripheral arterial disease, substance abuse, or chemotherapy or radiation to the chest. Patients who suffer congestive heart failure can present with dyspnea, fatigue, orthopnea, pedal edema, distended neck veins, a sustained and laterally displaced apical impulse, audible extra heart sounds, bibasilar crackles or dullness to percussion at the lung bases. Diagnosis of congestive heart failure can be made with chest X-ray, electrocardiogram, and echocardiography. Treatment of congestive heart failure includes addressing the causative factors. Patients with congestive heart failure should be advised to decrease sodium intake, monitor daily weights, and engage in light physical activity as tolerated. A history of bulimia nervosa (A) in the setting of pedal edema may suggest malnutrition and decreased osmotic pressure due to hypoproteinemia, leading to extravasation of fluid into the tissues. While anorexia nervosa in its extreme form can cause cardiac conduction abnormalities due to electrolyte imbalance, it is rare to see cardiac disease as a sequelae of bulimia nervosa. A history of chronic renal insufficiency (B) in a patient with pedal edema should alert the clinician to the possibility of worsening renal function and electrolyte imbalance, as well as malnutrition. Chronic renal disease can affect multiple organ systems, but is more likely to cause pericarditis than it is to cause congestive heart failure. A history of inflammatory bowel disease (D) in a patient with pedal edema should lead the clinician to suspect malabsorption and hypoproteinemia. Patients with inflammatory bowel disease can suffer from recurrent abdominal pain, cramping, and diarrhea (with or without blood). They can also suffer fever, malaise, and weight loss. Extraintestinal diseases caused by inflammatory bowel disease include uveitis, arthritis, and liver disease, but not cardiac disease.

A 35-year-old man with no history of tobacco use, diabetes or hypertension presents with substernal chest discomfort during physical exertion for the past six months. The pain is relieved by rest. His body mass index is 23. Which of the following physical exam findings would suggest a diagnosis of familial hypercholesterolemia? A. Bony nodules on the interphalangeal joints B. Hepatosplenomegaly C. Widespread flat hyperpigmented macules D. Yellow papules on the eyelids

According to the American Heart Association risk algorithm for atherosclerotic cardiovascular disease, there are certain factors which statistically increase a patient's risk for adverse cardiovascular events. These include age over 40 years, male sex, African-American race, current tobacco smoker, total cholesterol over 200 mg/dL, high-density lipoprotein cholesterol under 40 mg/dL, systolic blood pressure over 130 mm Hg, current treatment for hypertension, and diabetes mellitus. The above patient is presenting with angina, a sign of cardiovascular disease, in the absence of obvious risk factors. He should be evaluated for familial hypercholesterolemia, an autosomal dominant genetic disease. In familial hypercholesterolemia, the low density lipoprotein level is usually above 190 mg/dL, and there is often a family history of premature cardiovascular events. There may also be signs of hypercholesterolemia on physical exam. Such signs include cutaneous xanthoma, which are visible subcutaneous deposits of cholesterol. They are typically found over tendons, and when found on the eyelids are called xanthelasma. Any patient with preexisting cardiovascular disease or with a low density lipoprotein cholesterol greater than 189 mg/dL should be considered at high risk for atherosclerotic cardiovascular disease and treated with a statin unless otherwise contraindicated. HMG-CoA reductase inhibitors (statin drugs) have proven to decrease the risk of adverse cardiovascular events and to decrease mortality in patients at high risk for atherosclerotic cardiovascular disease. Patients with familial hypercholesterolemia who do not meet goals for lipid reduction on statin therapy should be referred to a lipid specialist. Bony nodules on the interphalangeal joints (A) are known as Heberden's nodes and are a sign of osteoarthritis. They are palpable osteophytes and present a cosmetic problem for patients but otherwise do not require specific treatment. The underlying osteoarthritis should be addressed with pain control. Hepatosplenomegaly (B) is not a presenting sign in familial hypercholesterolemia and is not associated directly with increased cholesterol. Disease states which cause hepatosplenomegaly may include infectious processes, infiltrative diseases, hematologic diseases, chronic hepatic insufficiency, and connective tissue disorders. Widespread flat hyperpigmented macules (C) refers to cafe-au-lait spots which can be seen in neurofibromatosis type 1 or McCune-Albright syndrome. Neurofibromatosis is an inherited disorder which causes widespread hyperpigmented macules, axillary freckling, bony deformities, and benign and malignant tumor formation in the nervous system. McCune-Albright syndrome is the result of a non-inherited gene mutation which causes overproduction of several different protein products. The three classic features are cafe-au-lait spots, polyostotic fibrous dysplasia, and autonomous endocrine hyperfunction. Hypercholesterolemia is not part of the clinical picture in neurofibromatosis or McCune-Albright syndrome.

A 24-year-old woman presents with complaints of excessive thirst over the past several weeks. She also notes waking up in the middle of the night to urinate. She recently underwent transsphenoidal resection of a pituitary adenoma. Which of the following is the most likely diagnosis? A. Central diabetes insipidus B. Nephrogenic diabetes insipidus C. Primary polydipsia D. Type 2 diabetes mellitus

Central diabetes insipidus, the most common type of diabetes insipidus, is associated with deficient secretion of antidiuretic hormone (ADH) by the posterior pituitary. It is most often idiopathic or can be induced by head trauma, pituitary surgery, or hypoxic or ischemic encephalopathy. Patients typically present with polyuria (colorless urine because it is so dilute), polydipsia, nocturia, and mild to moderate hypernatremia if patient has impaired thirst drive. Laboratory evaluation may reveal low urine specific gravity and low urine osmolality, and high plasma osmolality. Water restriction test is required to establish the diagnosis of diabetes insipidus. Patients with diabetes insipidus will continue to produce dilute urine with low osmolality and specific gravity (unlike patients with primary polydipsia). Further testing with desmopressin (exogenous ADH) stimulation test is required to differentiate central and nephrogenic diabetes insipidus. In central diabetes insipidus, urine osmolality will increase indicating a response to ADH. In nephrogenic diabetes insipidus, the patient will continue to produce dilute urine (low osmolality) indicating no response to ADH. Pituitary imaging may be needed to rule out a mass lesion in patients without obvious underlying cause. The treatment of choice for central diabetes insipidus is desmopressin acetate. Nephrogenic diabetes insipidus (B) is characterized by normal ADH secretion but failure of kidneys to respond to ADH. The most common cause of nephrogenic diabetes insipidus in adults is chronic lithium use. Treatment includes sodium restriction and thiazide diuretics. Primary polydipsia (C) is characterized by a primary increase in water intake. It is most often seen in middle-aged women and in patients with psychiatric illnesses and patients who have xerostomia. Type 2 diabetes mellitus (D) is by far the most common type of diabetes in adults. The majority of patients are asymptomatic, and hyperglycemia is noted on routine laboratory evaluation. Patients may complain of polyuria, polydipsia, and nocturia. Laboratory evaluation reveals hyperglycemia.

Which of the following patients meets the criteria for diagnosis of diabetes mellitus? A. A patient with a fasting plasma glucose of 125 mg/dL and a hemoglobin A1C of 6.0 percent B. A patient with polyuria, blurry vision, and a random glucose of 200 mg/dL C. An asymptomatic patient with a fasting glucose of 130 mg/dL (day 1) and 80 mg/dL (day 2) D. An asymptomatic patient with a random glucose of 250 mg/dL

Diabetes mellitus refers to several disease processes which cause impaired carbohydrate metabolism and subsequent hyperglycemia. Patients with diabetes mellitus can be classified into four main categories: type 1, type 2, gestational, and other causes. Type 1 diabetes mellitus encompasses patients who have islet cell destruction which leads to an absolute insulin deficiency. Type 2 diabetics have decreased secretion of insulin as well as insulin resistance. Gestational diabetes occurs in the setting of pregnancy and resolves in the postpartum period. Other types of diabetes mellitus include neonatal diabetes, maturity-onset diabetes of the young, pancreatic exocrine dysfunction, and drug-induced diabetes mellitus. The hyperglycemia caused by diabetes mellitus can result in polydipsia, polyuria, weight loss, fatigue, polyphagia, frequent mycotic infections, neuropathy and blurry vision. Patients who present with any combination of these classic symptoms should receive a random plasma glucose test. If the results are 200 mg/dL or above, a diagnosis of diabetes mellitus can be made. Asymptomatic patients may also be diagnosed with diabetes mellitus if they have a fasting plasma glucose level of 126 mg/dL or higher, a hemoglobin A1C of 6.5 percent or higher, or a wo-hour oral glucose tolerance test plasma glucose of 200 mg/dL or higher. Two different tests can be done on the same day for confirmation of the diagnosis, or the same test performed on two separate occasions. Upon diagnosis of diabetes mellitus, further testing may be necessary to differentiate the type of diabetes mellitus involved and the treatment needed. Type 1 diabetes mellitus will require insulin replacement therapy, while type 2 may be managed with oral agents such as biguanides or sulfonylureas. A patient with a fasting plasma glucose of 125 mg/dL and a hemoglobin A1C of 6.0 percent (A) has impaired glucose tolerance and is considered to be at increased risk for the development of diabetes mellitus. Patients with fasting glucose in the range of 101-125 mg/dL, or with a hemoglobin A1C of 5.7-6.4 percent, should receive counseling on the prevention of diabetes, including dietary changes and exercise. Diets that are low in sugar and refined carbohydrates and high in protein and fiber should be recommended. Exercise and weight loss can improve glucose tolerance and enhance endogenous insulin sensitivity in these individuals. An asymptomatic patient with a fasting glucose of 130 mg/dL (C) should receive another test to confirm the presence or absence of diabetes mellitus. This test could be a hemoglobin A1C, an oral glucose tolerance test, or a repeat fasting glucose on a separate day. If the patient comes back to the clinic the next day with a fasting glucose of 80 mg/dL, then a diagnosis of diabetes mellitus cannot be made. Further testing would be required to rule out diabetes. An asymptomatic patient with a random glucose of 250 mg/dL (D) requires further testing to diagnose diabetes mellitus. Checking a hemoglobin A1C or having a patient come back the next day for a fasting glucose test would help to rule out an isolated high glucose reading.

A 28-year-old male presents to the clinic with a four-year history of abdominal bloating, flatulence, diarrhea, and a pruritic rash on his buttocks and knees. He states his symptoms are worse with the consumption of pasta. Which of the following is most likely to confirm the diagnosis? A. 25-hydroxyvitamin D B. Duodenal biopsy C. HLA-DQ genotyping D. Tissue transglutaminase IgA antibody

Duodenal biopsy is the confirmatory test for celiac disease (gluten-sensitive enteropathy). Celiac disease is an autoimmune disease that is triggered by consumption of gluten and its metabolic byproducts (gliadin peptides). Associated diseases include diabetes, thyroid disease, and Turner syndrome. The exact etiology remains unclear, but environmental and genetic factors, particularly presence of human leukocyte antigen haplotypes DQ2 and DQ8, are believed to have a complex interaction predisposing to changes in IgA and IgG antibodies. When the small intestine mucosa is exposed to gluten and gliadin, inflammation of the microvilli and hypertrophy of the crypts occur. Chronic inflammation of the microvilli leads to atrophy and the characteristic smooth appearance of the small intestine on endoscopy. Malabsorption subsequently occurs with progressive loss of structure and function. Celiac disease classically presents with gastrointestinal symptoms, such as diarrhea, cramping, bloating, malabsorption, and steatorrhea, but may coincide with neurologic, dermatologic, psychiatric, hematologic, or skeletal diseases. The physical exam is non-specific, though the presence of dermatitis herpetiformis, often found on knees, buttocks, and elbows, may suggest the diagnosis. Some symptoms may be related to malabsorption, particularly of iron, calcium, fat, and fat-soluble vitamins (A, D, E, and K), which may lead to anemia, vitamin or mineral deficiencies, weakness, paresthesias, osteopenia, growth retardation, sterility, bleeding, and dermatitis. Diagnosis includes serology, as well as endoscopy and biopsy. Elevated levels of tissue transglutaminase IgA antibodies (IgA tTG), as well as anti-deamidated gliadin peptide IgG antibodies (IgG DPG) strongly suggest the diagnosis and are recommended screening tests. Guidelines recommend first testing tissue transglutaminase antibodies as a screening exam in those not deficient of IgA; anti-deamidated gliadin peptide antibodies are tested in children under two years old, or if IgA deficiency is present. Additional testing may include HLA DQ2 and DQ8, which essentially excludes celiac disease if negative. Endoscopic biopsy generally confirms the diagnosis. Scalloping of the small bowel folds, reduced number of folds, and a smooth or mosaic pattern of the mucosa are observed grossly. Histology may reveal an increased crypt to villi ratio, various stages of villi atrophy, and increased intraepithelial lymphocytes. A gluten-free diet is the primary treatment, however, glucocorticoids may rarely be required. Gluten is found in wheat, barley, and rye. Rice, and corn do not cause celiac disease. Complications include reduced quality of life related to symptoms, depression, lymphoma, osteopenia, and the myriad complications from vitamin and mineral deficiencies. 25-hydroxyvitamin D (A) may be deficient in patients with celiac disease, however, it does not confirm the diagnosis of celiac disease. HLA DQ genotyping (C) can be useful in ruling out celiac disease if negative, but the lack of specificity makes this test inappropriate for diagnosis. Tissue transglutaminase IgA antibody (D) is recommended as a screening test and to assess compliance with diet, but is generally not considered diagnostic. Some authorities state that children may be diagnosed with celiac disease if the IGA TTG is more than 10 times above the upper limit of normal, but only if symptomatic, with a positive HLA DQ test, positive anti-endomysial antibodies, and a reduction in symptoms with gluten avoidance.

A 46-year-old man presents with a painful, red first toe on his right foot. His symptoms came on suddenly one day ago. He states that even light touch is almost unbearable. He denies this ever happening in the past. His vital signs are within normal limits. Erythrocyte sedimentation rate and white blood cell count are normal. Synovial fluid analysis reveals negatively birefringent intra- and extracellular crystals. Which of the following is the most likely diagnosis? A. Gout B. Pseudogout C. Rheumatoid arthritis D. Septic arthritis

Gout is an inflammatory joint disease caused by the deposition of monosodium urate crystals in a joint space. All patients with gout have increased blood uric acid levels at some point in their disease, however, not all individuals with hyperuricemia will develop gouty arthritis. Gout generally presents as an inflammatory monoarthritis with severe tenderness, swelling, erythema, and warmth. It most often involves the lower extremities, and the great toe (podagra) is the most common joint affected. Other common joints affected are ankles and knees. Certain foods (foods rich in purines like red meat, seafood), alcohol consumption and medications (thiazide and loop diuretics) are known to trigger attacks. Men of 40-60 years of age are most commonly affected. Joint aspiration with synovial fluid analysis is used to confirm the diagnosis. Joint fluid analysis showing needle-shaped negatively birefringent urate crystals has a sensitivity of 100% for the diagnosis of gout. Radiographs are normal in early disease, however, punched-out erosions with "overhanging edges" of bone associated with bone erosions due to tophi may be seen in advanced disease. Initial treatment in acute gout is NSAIDs (indomethacin). Colchicine is an alternative for patients who cannot take NSAIDs or did not respond to NSAIDs. Corticosteroids are used if a patient does not respond or cannot tolerate the above treatment. Pseudogout (B), or calcium pyrophosphate crystal deposition disease, can mimic gout in its presentation and physical exam findings, however, a joint aspiration will show positively birefringent calcium pyrophosphate crystals. It typically occurs in larger joints (knee). Rheumatoid arthritis (C) can present with joint pain, however, it generally progresses more slowly than gout, and joint aspirate will not show negatively birefringent crystals. Septic arthritis (D) can present with an acutely inflamed, painful joint. Joint aspirate, as well as blood work, will show signs of infection that would not be present with gout.

A 21-year-old woman presents to the clinic with several large, anterior and posterior cervical lymph nodes. She complains of weight loss and night sweats for the past four months. Which of the following is the most effective next step in the diagnosis of Hodgkin lymphoma? A. CT scan of neck, chest and abdomen B. Fluorescence immunophenotyping C. Lymph node excisional biopsy D. Lymph node fine needle aspiration

Hodgkin lymphoma is a malignant proliferation of cells in the lymphoreticular system and can be localized or disseminated. It has a bimodal distribution peaking around age 20 years and age 65 years although most patients are young adults. Clinical signs and symptoms include lymphadenopathy (often cervical and painless), pruritus, fever, night sweats, unintentional weight loss, and frequent infections. Diagnosis is made with excisional lymph node biopsy, where pathognomonic Reed-Sternberg cells (owl-eye appearance) can be visualized in an inflammatory background. Morphologic features of the Reed-Sternberg cells help determine the subtype of Hodgkin lymphoma and inform treatment. Treatment depends on disease subtype and stage at diagnosis and can involve chemotherapy, radiation, surgery, and stem cell transplantation. The cure rate for Hodgkin lymphoma is between seventy and eighty percent with treatment. Health maintenance in patients who are considered to be in complete remission consists of follow-up office visits every three months in the first five years after treatment. These visits should include a history, physical, and laboratory studies (CBC, LDH, lipid panel, ESR, and glucose). Periodic evaluation for long-term complication of therapy (breast cancer, lung cancer, cardiac disease, hypothyroidism) is also indicated. CT scan of the neck, chest, and abdomen (A) may be indicated before surgical excision of an enlarged lymph node to evaluate the extent of dissemination of lymphadenopathy and to stage the disease, however, a CT scan cannot diagnose Hodgkin lymphoma. Therefore, the most effective next step in diagnosis would be lymph node biopsy. Fluorescence immunophenotyping (B) is performed on Reed-Sternberg cells after lymph node biopsy to distinguish classic Hodgkin lymphoma from nodular lymphocyte predominant Hodgkin lymphoma and other lymphoma subtypes. The excisional biopsy comes first. Lymph node fine needle aspiration (D) does not provide enough tissue for diagnosis of Hodgkin lymphoma, neither does it provide enough information about the structural composition of the lymph node. In certain cases, large core needle biopsy may be used for diagnosis, but excisional biopsy is preferred.

A 65-year-old man with a 30 pack-year smoking history presents for an annual physical exam. He is a current smoker. He asks about lung cancer screening. Which of the following screening methods would you recommend? A. Chest radiography B. Low-dose chest computed tomography C. Positron emission tomography D. Sputum cytology

The United States Preventive Services Task Force recommends annual screening for lung cancer with low-dose computed tomography in adults aged 55 to 80 years who have a 30 pack-year smoking history and currently smoke or have quit within the past 15 years. Screening should be discontinued once a person has not smoked for 15 years or develops a health problem that substantially limits life expectancy or the ability or willingness to have curative lung surgery. Lung cancer is the second most common cancer in men and women and the leading cause of mortality worldwide for both men and women. The most important risk factor for lung cancer is smoking. The sensitivity of chest radiography (A) for detecting lung cancer varies depending on the size and location of the lesion, image quality, and skills of the radiologist who interprets the scan. Low-dose chest computed tomography has emerged as a test with higher sensitivity and specificity for lung cancer than chest radiography. Positron emission tomography (C) is useful in the initial staging to identify sites of tumor involvement, detect lymph nodes, and detect distant metastases. It is not recommended as the first-line screening test. Sputum cytology (D) is now rarely used for lung cancer screening and no studies have been reported to validate this screening method. However, sputum cytology may be useful for patients in whom biopsy is contraindicated. It may aid in diagnosing central tumors but not peripheral. Question: What is the current standard of care for treatment of localized non-small cell lung cancer? Reveal Answer: Lung Cancer Patient with a history of smoking Complaining of a cough (most common), hemoptysis, dyspnea, and chest pain Non-pulmonary symptoms that suggest metastases:Hip and back painHorner's syndrome (ipsilateral ptosis, anhidrosis, and miosis)Neurologic symptomsHypotension with tachycardia Labs will show hypercalcemia, exudative effusion Diagnosis is made by CXR, CT scan, and biopsy to confirm Most common type is non-small cell (adenocarcinoma) Comments: Leading cause of cancer-related death among men and women Screening (American Cancer Society):Average risk patients - not recommended High-risk patients: 55 - 80 years of age, in good health with 30 pack-year smoking history (packs of cigarettes smoked per day x number of years the person has smoked) and Patient is currently smoking or quit within the last 15 years Test of choice is an annual low-dose helical CT (LDCT) scan References: 1. Deffebach M, Humphrey L. Screening for lung cancer. Post TW, ed. UpToDate. Waltham, MA: UpToDate Inc. Accessed October 23, 2017. 2. U.S. Preventive Services Task Force. Final Recommendation Statement: Lung Cancer: Screening. U.S. Preventive Services Task Force, 2016. U.S. Preventive Services Task Force Website. Accessed October 23, 2017. «PreviousNext» Go ToAll QuestionsAll Questions Flagged Questions Not Attempted Questions 12345678910111213141516171819202122232425262728293031323334353637383940414243444546474849505152535455565758596061626364656667686970717273747576777879808182838485868788899091929394959697 Category: Pulmonary Subcategory: Neoplasms Question: 142250 Submit Feedback Peer Comparison A12% B88% C0% D0%

Which of the following best represents an appropriate lipid screening protocol for an adult man with no other cardiovascular risk factors? A. Lipid profile beginning at age 20 B. Lipid profile beginning at age 30 C. Lipid profile beginning at age 35 D. Lipid profile beginning at age 40

According to the American Heart Association and the United States Preventive Screening Task Force, adult men with no other cardiovascular risk factors should begin screening for increased lipids at age 35 with a fasting lipid profile. The lipid profile may be repeated every five years (the American Heart Association recommends every four to six years) while cardiovascular risks remain stable. Factors which statistically increase a patient's risk for adverse cardiovascular events include: age over 40 years, male sex, African American race, current tobacco smoker, total cholesterol over 200 mg/dL, high-density lipoprotein cholesterol under 40 mg/dL, systolic blood pressure over 130 mm Hg, current treatment for hypertension, and diabetes mellitus. Patients with any of these risk factors should be evaluated using a cardiovascular risk calculator to determine their need for treatment with a statin. Any patient with preexisting cardiovascular disease or a low density lipoprotein cholesterol greater than 189 mg/dL should be considered at high risk for atherosclerotic cardiovascular disease and treated with a statin unless otherwise contraindicated. HMG-CoA reductase inhibitors (statin drugs) have proven to decrease the risk of adverse cardiovascular events and to decrease mortality in patients at high risk for atherosclerotic cardiovascular disease. Obtaining a lipid profile beginning at age 20 (A) is recommended for men and women who are at increased risk for coronary artery disease, such as those who smoke, have high blood pressure, or diabetes. Obtaining a lipid profile beginning at age 30 (B) is not a current recommendation, although the age range of 20 to 35 years of age is recommended as a starting point for screening in men who are at increased risk for coronary artery disease. Obtaining a lipid profile beginning at age 40 (D) is not recommended, as the rates of dyslipidemia and obesity are increasing in the United States and the disease is being encountered at younger ages. Studies involving patients under 40 years of age have not proven conclusive regarding the usefulness of statins at preventing cardiovascular risk, but younger patients can still benefit from dietary and lifestyle changes. Along with screening for dyslipidemia, a sedentary lifestyle is not advised, and increased exercise should be encouraged as it has been proven to lower total cholesterol levels. Thirty minutes of moderate intensity aerobic exercise five days a week should be recommended.

Which of the following is a classic sign of acute adrenal insufficiency? A. Abdominal pain B. Bluish-black discolorations of the mucous membranes C. Profound bradycardia D. Severe headache

Acute adrenal insufficiency can result from pituitary failure or from hypofunctioning of the adrenal cortex. The result of each is a deficiency of circulating mineralocorticoids and glucocorticoids. In the acute setting, these deficiencies lead to salt wasting, hypoglycemia, hypotension and circulatory collapse. The hallmarks of adrenal crisis are profound weakness, severe abdominal pain, peripheral vascular collapse, electrolyte abnormalities, and shock. In patients with chronic adrenal insufficiency, an acute adrenal crisis can be precipitated by stressors such as infection, trauma, or surgery. Laboratory studies in patients with acute adrenal insufficiency may show hyponatremia, hypoglycemia, hyperkalemia, low serum cortisol, eosinophilia, or sepsis. An abdominal CT scan may show adrenal hemorrhage, atrophy, or infiltrative disease, depending on the cause of the acute crisis. Treatment of acute adrenal insufficiency involves stabilizing the patient with fluid resuscitation, administration of glucocorticoids, regulation of serum sodium and potassium, and occasionally requires administration of fludrocortisone (a mineralocorticoid). Addressing underlying precipitating factors, such as sepsis, is also crucial. Bluish black discolorations of the mucous membranes (B) are seen in chronic primary adrenal insufficiency. The discoloration is due to increased concentration of serum ACTH and blood beta-lipotropin, which have melanocyte-stimulating activity. Although these discolorations can give the clinician a clue as to underlying adrenal disease, they are not indicative of an acute process, but rather of chronic cortisol insufficiency. Other signs of chronic adrenal insufficiency are cold intolerance, orthostatic hypotension, nausea, vomiting, diarrhea, and hyperpigmentation of the skin creases and extensor surfaces. Profound bradycardia (C) is not generally seen in patients suffering acute adrenal crisis. Instead, electrolyte imbalances resulting from disruptions in production of mineralocorticoids can cause prolongation of the QT interval and ventricular tachycardias. Hypovolemia can cause compensatory tachycardia as well. Severe headache (D) is not considered a classic sign of acute adrenal insufficiency, although patients with electrolyte imbalance or dehydration may complain of a headache.

An otherwise healthy 24-year-old woman presents with a cough. The cough is productive of white to yellow sputum and has been present for the last three weeks. She states that it began with a fever, sinus pressure, and fatigue, but those symptoms have since resolved. She has never had similar symptoms in the past. Her vital signs are all within normal limits. Lung auscultation is clear bilaterally, although deep breaths induce coughing. What is the most likely diagnosis? A. Acute bronchitis B. Asthma C. Chronic bronchitis D. Community-acquired pneumonia

Acute bronchitis can cause a cough for more than three weeks. Patients can initially present with constitutional symptoms including fever, malaise, myalgias, or rhinorrhea. The symptoms are typically caused by a viral infection. Viruses that have been implicated in causing acute bronchitis include respiratory syncytial virus, influenza, adenovirus, coronavirus, and rhinovirus. The diagnosis is clinical, although sputum cultures or nasopharyngeal swabs may yield a definitive pathogen. Treatment is typically supportive, however, some studies show that bronchodilators are effective in providing relief. The role of antibiotics is controversial in treating acute bronchitis because the majority of cases are caused by a virus. Asthma (B) can often present with a chronic cough. In this case, the patient had constitutional symptoms with the beginning of the cough and had no past medical history significant for asthma. This makes a diagnosis of asthma less likely, though the diagnosis should be considered in someone with a persistent cough. Chronic bronchitis (C) is diagnosed when there is a productive cough for at least three months in two consecutive years. The signs and symptoms of community-acquired pneumonia (D) include cough, fever, tachypnea, and hypoxia. Lung auscultation may reveal rales. A chest X-ray would be confirmatory.

A 24-year-old man presents to the emergency department with fever and chills, right upper quadrant pain, and jaundice. Abdominal ultrasound shows dilation of the common bile duct. What is the most likely diagnosis? A. Acute cholangitis B. Acute cholecystitis C. Acute hepatitis D. Cholelithiasis

Acute cholangitis is a bacterial infection caused by obstruction of the biliary tree, most commonly by a gallstone. Obstruction can also be associated with malignancy or stricture. It is typically caused by gram negative enteric organisms from the duodenum, with Escherichia coli being the most common organism, followed by Klebsiella, Enterococcus, Streptococcus, and Pseudomonas. Patients can present with classic symptoms of fever, right upper quadrant pain, and jaundice, known as Charcot triad. Additional symptoms may include altered mental status and shock. Presentation may vary widely from mild symptoms to fulminant sepsis. Treatment options include broad-spectrum antibiotics, common bile duct decompression with stone extraction, and, in emergency situations, surgical decompression of the biliary tree. Acute cholecystitis (B) is an infection of the gallbladder itself and will show gallbladder wall thickening and pericholecystic fluid with the sonographic finding of Murphy sign. Gallstones or sludge are typically present within the gallbladder. Acute hepatitis (C) is characterized by hepatomegaly with possible gallbladder thickening but has no effect on the bile duct system. Cholelithiasis (D) is the presence of stones in the gallbladder itself. Cholelithiasis may be an incidental finding on ultrasound or CT of the abdomen or may result in symptoms of biliary colic, cholecystitis, or ascending cholangitis.

What is the treatment of choice for an otherwise healthy patient with a confirmed diagnosis of acute idiopathic pericarditis? A. Antiviral therapy B. Glucocorticoids C. NSAIDs D. Pericardiocentesis

Acute pericarditis is an inflammatory process involving the pericardial sac. It is the most common disorder of the pericardium. Most cases are idiopathic (probably viral in etiology). Other etiologies include infections (Coxsackie, influenza, HIV, hepatitis A or B, tuberculosis, fungal infections), acute myocardial infarction, malignancy, and autoimmune disorders. Findings consist of pleuritic chest pain, pericardial friction rub, and diffuse ST-segment elevation and PR depression on ECG. First-line treatment in healthy individuals consists of NSAID therapy, which aids in reducing the inflammation of the pericardial sac. Ibuprofen or aspirin are good first-line choices, and it is generally recommended to combine these therapies with colchicine to reduce the rate of recurrent pericarditis. Patients with multiple risk factors, associated pericardial effusion, or hemodynamic instability may need to be admitted. Although acute idiopathic pericarditis is thought to be of viral etiology, antiviral therapy (A) is not generally recommended. Glucocorticoids (B) are considered an acceptable second-line therapy, especially for patients who do not respond to NSAID therapy. For patients who develop an associated pericardial effusion, pericardiocentesis (D) may be indicated, however, it is not considered first-line therapy.

A 62-year-old man presents to the emergency room complaining of acute onset hematuria and facial edema. Patient reports that he was out of work for several days due to having a sore throat about two weeks ago but he seemed to have recovered until these new symptoms began earlier today. Urine examination reveals hematuria with red blood cell casts and a decreased glomerular filtration rate. What is the most likely diagnosis? A. Acute poststreptococcal glomerulonephritis B. Idiopathic hematuria C. Immunoglobulin A nephropathy D. Minimal change disease

Acute poststreptococcal glomerulonephritis presents with acute onset of hematuria with red blood cell casts, edema, hypertension, and a decreased glomerular filtration rate. It is due to the inflammation and immune complex deposition in the glomerulus and occurs primarily in children ages 2 to 12 years old, and in older adults greater than 60 years of age. While rates of acute poststreptococcal glomerulonephritis in the United States have fallen in recent years, it is still seen clinically and is associated with an increased antistreptolysin-O titer. In addition to hematuria and red blood cell casts, the shape of the red blood cells is often distorted as they deform trying to pass through the inflamed glomerulus. Acute poststreptococcal glomerulonephritis typically occurs one to two weeks after an acute group A beta-hemolytic Streptococcus pharyngitis infection or three to six weeks after an acute group A beta-hemolytic Streptococcus skin infection. Prognosis is best in children without underlying renal disease and worse in adults with underlying renal disease. Nearly all patients are asymptomatic after five years; however, recovery is slow with proteinuria typically persisting for six months and microscopic hematuria persisting for up to one year. Treatment of acute poststreptococcal glomerulonephritis is supportive as there is no specific treatment to address the glomerular dysfunction. Idiopathic hematuria (B) may be considered in a young person with acute onset of hematuria but should not be accompanied by red blood cell casts and deformed red blood cells on microscopy as seen in this case. Immunoglobulin A nephritis (C) most commonly occurs concomitant with an upper respiratory infection. If it is not co-occurring, the time between infection and onset of nephritis is one to two days and not several weeks as seen in this case. Minimal change disease (D) is the most common cause of nephrotic syndrome in children. Facial edema is typically the first symptom noted. It often follows a viral upper respiratory syndrome accompanied by malaise and easy fatigability. The first-line therapy for minimal change disease is corticosteroids which typically resolves the proteinuria.

A 28-year-old woman with no known drug allergies presents to the clinic after discharge from the hospital for acute rheumatic fever with carditis. Echocardiographic records reveal subsequent mitral valve disease. Which of the following is the most appropriate choice for secondary prophylaxis of rheumatic fever in this patient? A. Intramuscular injection of penicillin G benzathine every 21-28 days for 10 years B. Intramuscular injection of penicillin G benzathine every 21-28 days until age 40 C. Oral azithromycin 250 mg once daily for 10 years D. Oral penicillin V 250 mg twice daily for 10 years

Acute rheumatic fever is a complication of group A Streptococcus pharyngitis. Signs and symptoms of the disease are divided into major and minor categories. Minor criteria for diagnosis include fever, arthralgia, elevated erythrocyte sedimentation rate or elevated C-reactive protein, and prolonged P-R interval on ECG. Major criteria for diagnosis include Sydenham's chorea, carditis, erythema marginatum, polyarthritis, and subcutaneous nodules. According to the modified Jones criteria, a patient who demonstrates previous infection with group A Streptococcus (via rising antibody titer or positive throat culture) must manifest two major or one major and two minor criteria for a diagnosis of acute rheumatic fever. Patients with acute rheumatic fever are at risk for rheumatic heart disease, which can appear years after infection and usually presents as valvular stenosis. The valve most commonly affected is the mitral valve. The risk of rheumatic heart disease increases with each subsequent group A Streptococcus infection. For this reason, patients who have suffered acute rheumatic fever should receive secondary prophylaxis against group A Streptococcus. For patients who are not allergic to penicillin, the prophylactic treatment of choice is with penicillin G benzathine, administered intramuscularly every 21-28 days. The duration of prophylactic therapy depends upon the age and health status of the patient at presentation. For a patient who has rheumatic fever with carditis and echocardiographic evidence of persistent valvular disease, current recommendations call for prophylaxis to continue for ten years or until age 40, whichever is longer. Intramuscular injection of penicillin G benzathine every 21-28 days for 10 years (A) is the current recommendation for patients with acute rheumatic fever, carditis, and echocardiographic evidence of heart disease. However, the recommendation also calls for prophylaxis to continue for 10 years or until age 40, whichever is longer. For the patient in the above vignette, age 40 is longer and would be preferred. In patients with rheumatic fever and carditis, but no evidence of residual heart disease, the recommendations call for prophylaxis to continue for 10 years or until the patient is 21 years of age, whichever is longer. Patients who suffer rheumatic fever without carditis and with no evidence of heart disease require prophylaxis for five years or until 21 years of age (whichever is longer). Oral azithromycin 250 mg once daily for 10 years (C) is the current recommendation for secondary prophylaxis in patients with rheumatic fever, carditis, and a penicillin allergy. Oral penicillin V 250 mg twice daily for 10 years (D) is the current recommendation for second-line therapy of rheumatic fever with carditis in patients who cannot tolerate or who refuse penicillin injections. Duration of second-line therapy mirrors that of first-line therapy.

A 40-year-old woman, diagnosed three months ago with fibromyalgia, returns to the clinic complaining of persistent overall diffuse pain. She followed your previous advice of starting a regular exercise routine, making better dietary choices, and joining a support group. Energy levels have improved but her pain is unchanged. What is the most likely medication you would start her on? A. Amitriptyline B. Carisoprodol C. Celecoxib D. Prednisone

Amitriptyline is a tricyclic antidepressant commonly used as a first-line treatment for fibromyalgia. Fibromyalgia is now thought to be a neurologic disorder in which there is a hypersensitivity to painful stimuli. It is characterized by widespread musculoskeletal pain, fatigue, anxiety, and depression. Amitriptyline improves anxiety and depression and has been shown to help with fatigue and myalgias as well. Carisoprodol (B), a muscle relaxer, has not proven beneficial in improving fibromyalgia symptoms. Celecoxib (C), a selective cyclooxygenase-2 inhibitor nonsteroidal anti-inflammatory, has not shown to improve fibromyalgia pain by itself. It may be used in combination with an antidepressant like amitriptyline, but not as a first-line treatment. Prednisone (D) is not recommended for fibromyalgia and is typically avoided by rheumatologists who now have a better understanding of the disease process. While patients typically do see improvement in their pain with steroids, the risks, including bone loss, elevated glucose, hypertension, acne, increased low-density lipoprotein, etc., outweigh the benefits, especially in long-term use.

A 27-year-old man presents with complaints of tearing pain while having a bowel movement. He notes bright red blood on the toilet paper after wiping. Physical examination reveals a superficial tear without a track in the posterior midline of the anus. Which of the following is the most likely diagnosis? A. Anal fissure B. Anorectal fistula C. Anorectal ulcer D. Hemorrhoid

An anal fissure is one of the most common benign anorectal conditions and the most common cause of anal pain and bleeding. Anal fissures are the result of the stretching of the anal mucosa beyond its normal capacity. The majority of anal fissures are primary and are caused by local trauma, such as passage of large, hard stools, prolonged diarrhea, vaginal delivery, or anal sex. Primary anal fissures are most commonly located posterior midline. Secondary anal fissures located other than midline and may be found in patients with Crohn disease, tuberculosis, sarcoidosis, malignancy, HIV, and syphilis. Patients with an acute anal fissure present with tearing pain during defecation. Patients may also describe bright red blood noted on the toilet paper or surface of the stool. A superficial longitudinal tear without a track is noted on physical examination. Most primary anal fissures respond to conservative management, including wound care and relief of constipation, though surgical management may be needed for chronic fissures. An anorectal fistula (B) typically presents as a painful pustule-like perianal skin lesion with chronic purulent drainage. A track may be identified on probing, extending from the perianal skin to the anus or rectum. Anorectal ulcers (C) are caused by inflammatory bowel diseases, such as Crohn's disease, and granulomatous diseases, such as tuberculosis. Sexually transmitted infections, such as syphilis, may also cause anorectal ulcers. Anal fissures are typically thinner than ulcers caused by underlying inflammatory conditions. Patients will complain of pain and bleeding with defecation and passage of mucus. Hemorrhoids (D) are normal vascular structures in the anal canal. Symptoms include bleeding, anal pruritus, and prolapse. Hemorrhoids are typically painless unless thrombosis has occurred.

Which of the following physical exam findings is most indicative of angina pectoris? A. Left-sided chest pain associated with a dermatomal vesicular rash B. Reproducible chest pain on chest wall palpation C. Substernal chest pain associated with a new third heart sound D. Substernal chest pain associated with decreased heart sounds and jugular venous distention

Angina pectoris is defined as pain or discomfort due to transient myocardial ischemia from atherosclerotic disease of the coronary vessels or vasospasm. Patients experience many varied forms of angina, with symptoms ranging from occasional substernal chest pain described as a deep ache, to more intense, crushing substernal pain. Some patients experience chest pain that radiates to the left arm or jaw, while others complain of dyspepsia, nausea or confusion. Angina pectoris is typically brought on by physical exertion and is relieved by rest or sublingual nitroglycerin (in stable angina). Unstable angina is a worsening of angina with symptoms at rest, longer episodes of chest pain, or pain necessitating increased doses of nitroglycerin. Physical exam is often normal in patients who suffer angina pectoris. If the patient is experiencing angina pectoris in the office, a new S3 or S4 is sometimes auscultated due to left ventricular dysfunction. Patients who have angina should receive an electrocardiogram and exercise or pharmacologic stress testing. If these tests demonstrate ischemia, the patient will be referred for cardiac catheterization to determine the level and degree of coronary artery blockage. Treatment of angina depends on the degree of atherosclerotic disease and may include beta-blockers, nitroglycerin and calcium channel blockers (to reduce blood pressure and myocardial wall tension). Angiotensin-converting enzyme inhibitors are also used for inhibition of cardiac remodeling and control of blood pressure. Aspirin, antiplatelet drugs and statins are used to combat the negative effects of atherosclerotic disease. Coronary angioplasty and coronary artery bypass surgery may also be employed. Left-sided chest pain associated with a dermatomal rash (A) is indicative of herpes zoster. Patients with acute herpes zoster will often complain of pain before the appearance of a rash. When herpes zoster affects a dermatome of the left chest, the pain can be mistaken for angina pectoris. The appearance of a vesicular rash in a dermatomal distribution with exquisite tenderness to light touch will differentiate herpes zoster from angina. Angina pectoris is not associated with a rash. Reproducible chest pain on chest wall palpation (B) is not typical of angina pectoris. The pain associated with angina may be reproduced by exertion or stress, but is not typically reproducible by palpation. Chest wall tenderness to palpation is more likely to be of musculoskeletal origin than cardiac origin. Substernal chest pain associated with decreased heart sounds and jugular venous distention (D) is a description of physical findings in a patient with cardiac tamponade. If the pericardium fills with fluid due to hemorrhage, inflammation or infection, the resulting pressure exerted on the myocardium can cause systolic and diastolic cardiac dysfunction. This dysfunction can lead to distended neck veins, hypotension and shock. The heart sounds in such patients may sound distant or muffled as a result of the fluid accumulated in the pericardium.

A 35-year-old Caucasian man presents to the emergency department with slurred speech and right facial drooping. He had a similar episode a few months ago and did not suffer any residual effects. He also has a history of deep venous thrombosis in the left leg but has no history of surgeries, hypertension, or diabetes. Which of the following should be included in the evaluation to determine the cause of this condition? A. Bone marrow biopsy B. D-dimer C. Hemoglobin electrophoresis D. Lupus anticoagulant

Antiphospholipid syndrome is an acquired autoimmune disorder that causes a hypercoagulable state. Individuals with this disorder are known to have recurrent venous or arterial thrombosis at an early age, although not all patients are symptomatic. Any patients with the following presentations should be tested for antiphospholipid syndrome: history of thrombosis in the absence of known risk factors, miscarriage (especially if in the late trimester or recurrent), history of heart murmurs or cardiac valvular vegetations, history of hematologic abnormalities, and pulmonary hypertension. The cause of this disorder is unknown, but it is associated with a variety of autoimmune and rheumatic diseases, certain infections (e.g. syphilis, hepatitis C, malaria), and drugs (e.g. procainamide, phenytoin, amoxicillin). Genetic disposition and familial association have also been linked. Important lab markers include the presence of lupus anticoagulant (associated with the strongest risk for thrombosis), anticardiolipin, and anti-beta 2 glycoprotein I antibodies. Treatment is individualized based on the patient's current clinical status and history of thrombotic events. Known risk factors for hypercoagulability must be addressed including discontinuing oral contraceptives, tobacco cessation, and treating hypertension and hyperlipidemia if present. Any associated autoimmune diseases must be managed as well. Low-dose aspirin is a common therapy for primary prevention of thrombotic events. Thrombotic events are initially treated with intravenous heparin or subcutaneous enoxaparin. Life-long treatment with warfarin is standard for patients who experience recurrent thrombotic events. A bone marrow biopsy (A) is useful when suspecting myeloproliferative disease which is not consistent with the patient's presentation. D-dimer (B) is used to rule out thrombosis and would be a nonspecific finding in this case since it would likely be positive. Hemoglobin electrophoresis (C) is used for diagnosing sickle cell disease. Although some patients with sickle cell disease can present with stroke symptoms or deep venous thrombosis, sickle cell disease occurs more prevalently in those of African or Mediterranean descent. Patients with sickle cell disease also are likely to have a history of chronic hemolytic anemia or vaso-occlusive crisis.

A 32-year-old woman with Marfan syndrome is brought to the emergency department by her husband after she experienced a syncopal episode. She complains of acute onset of sharp substernal chest pain with radiation to her back. On arrival, her vital signs are T 37.3C, HR 130, BP 90/40, RR 27. The blood pressure is similar in bilateral arms. Physical examination reveals no jugular venous distention and heart sounds are auscultated clearly without murmurs, rubs, or gallops. Which of the following is the most appropriate next step in the management of this patients condition? A. Labetalol B. Nitroprusside C. Norepinephrine D. Normal saline

Aortic dissection is an uncommon but life-threatening phenomenon that occurs when damage of the intima allows the entry of blood between the intima and media, creating a false lumen. The most important risk factor for aortic dissection is hypertension. CT angiogram is the gold standard for diagnosis of dissection. Chest x-ray is neither sensitive nor specific for aortic dissection, but findings may include a widened mediastinum, hemothorax, or calcium sign. Aortic dissection typically presents with hypertension. Hypotension, when present, is a poor prognostic indicator as it likely indicates aortic rupture or cardiac tamponade. Severe aortic insufficiency may also cause hypotension due to cardiogenic shock. Pseudohypotension, which occurs when measuring blood pressure in an extremity whose vascular supply is compromised by the dissection, should be ruled out. The cause of the hypotension should be determined and first-line treatment of hypotension in the setting of aortic dissection with aortic rupture or tamponade is fluid resuscitation. Labetalol (A), a short-acting beta-blocker, is a first-line treatment of hypertension in the setting of aortic dissection. It should not be used when there is hypotension. Nitroprusside (B) is used for persistent hypertension after negative inotrope use. It, likewise, should not be used in the setting of hypotension. Vasopressors like norepinephrine (C) should initially be avoided in hypotension in the setting of aortic dissection as positive inotropy may lead to increased stress on the aortic wall. In the setting of refractory hypotension, however, vasopressor use may become necessary.

A 75-year-old man presents with complaints of difficulty breathing when exercising. He notes he was able to walk two miles a day but recently has been experiencing increased shortness of breath and dizziness during daily walks. On physical examination, the patient is found to have a harsh midsystolic ejection murmur, best heard in the second right intercostal space, radiating to the neck. Which of the following is the most likely diagnosis? A. Aortic regurgitation B. Aortic stenosis C. Mitral regurgitation D. Mitral stenosis

Aortic stenosis is the most common cause of left ventricular outflow obstruction in children and adults. Causes of aortic stenosis include calcification of tricuspid aortic valve in elderly, calcification of a congenitally abnormal bicommissural or bicuspid aortic valve in young adults and children, and rheumatic disease. The classic clinical manifestations of aortic stenosis are heart failure, syncope, and angina, representing end-stage disease. With earlier diagnosis by echocardiography, patients most commonly present with dyspnea on exertion, exertional dizziness or syncope, exertional angina, and exercise intolerance. Physical examination may reveal a slow rate of rise in the carotid pulse, harsh systolic "ejection" murmur heard best in the "aortic" area, radiating to carotid arteries, reduced intensity of the second heart sound (S2), and a fourth heart sound (S4). Echocardiography is the primary test in the diagnosis and evaluation of aortic stenosis. Findings may include thickened, calcified, and immobile aortic leaflets, dilated aortic root, and left ventricular hypertrophy. Surgical repair or aortic valve replacement is the only effective long-term treatment and indicated in all symptomatic patients. The murmur of aortic regurgitation (A) is heard during the diastolic phase. It is best heard at the left sternal border while the patient sits and leans forward with the breath held in full expiration. The murmur is high-frequency and has a "blowing" character. The configuration is usually decrescendo. The holosystolic murmur of mitral regurgitation (C) is high pitched, blowing and best heard over the apex of the heart. It characteristically radiates to the left axilla. The mid-diastolic murmur of mitral stenosis (D) is best heard over the apex of the heart with the patient in the left lateral decubitus position. It characteristically starts with an opening snap followed by low-pitched diastolic rumble.

You are treating a 62-year-old man who is seven days status-post coronary angiogram and stent placement, following an acute myocardial infarction. He has been on a heparin drip for five days. Lab calls to notify you that his previously normal platelet count is now 48,000/mcL. Which of the following is the most likely diagnosis? A. Disseminated intravascular coagulation B. Heparin-induced thrombocytopenia C. Immune thrombocytopenia D. Thrombotic thrombocytopenic purpura

B. Heparin-induced thrombocytopenia

An otherwise healthy 20-year-old man suddenly collapses and dies while playing intramural basketball. The patient's brother died two years ago while playing soccer. Which of the following is the most likely diagnosis? A. Dilated cardiomyopathy B. Hypertrophic cardiomyopathy C. Restrictive cardiomyopathy D. Takotsubo cardiomyopathy

B. Hypertrophic cardiomyopathy Patient will be a yoHypertrophic cardiomyopathy is the most common cause of sudden cardiac death in young athletes. Hypertrophic cardiomyopathy is an autosomal dominant disorder. Left ventricular outflow obstruction due to asymmetric hypertrophy of the interventricular septum, diastolic dysfunction due to myocardial hypertrophy, myocardial ischemia, and mitral regurgitation are some of the abnormalities that may develop in patients with hypertrophic cardiomyopathy. Patients may complain of fatigue, dyspnea on exertion, chest pain, palpitations, syncope or dizziness. Patients with hypertrophic cardiomyopathy are at increased risk of dysrhythmias (ventricular dysrhythmias, atrial fibrillation) and sudden cardiac death. Physical exam may be normal or may reveal abnormalities such as a loud fourth heart sound (S4), sustained PMI, and harsh crescendo-decrescendo systolic murmur best heard at the apex and left lower sternal border. ECG should be performed in all patients with a suspected diagnosis of hypertrophic cardiomyopathy. Findings of ECG may include prominent abnormal Q waves, P wave abnormalities, left ventricular hypertrophy (LVH), left axis deviation, and deeply inverted T waves. A clinical diagnosis of hypertrophic cardiomyopathy is confirmed when unexplained left ventricular wall thickness ≥ 15 mm is imaged anywhere in the left ventricular wall on echocardiography. Initial treatment is with negative inotropic agents, including beta blockers, calcium channel blockers (verapamil), or disopyramide. Surgery (myomectomy) is reserved for patients with severe disease. All patients should be educated to avoid strenuous exercise and dehydration. Dilated cardiomyopathy (A) is characterized by dilation and impaired systolic function of the left ventricle or both ventricles. Clinical presentation includes symptoms and signs of a left-sided and right-sided heart failure. Many cases of dilated cardiomyopathy are idiopathic, however, several causes have been identified and include viral myocarditis, ischemic heart disease, hypertension, substance abuse (alcohol, cocaine), doxorubicin. The echocardiogram shows left ventricular dilation, normal or decreased wall thickness, poor wall thickening, and reduced inward endocardial systolic motion. Restrictive cardiomyopathy (C) is characterized by impaired diastolic ventricular filling due to decreased ventricular compliance. Amyloidosis is a common cause. Patients usually present with symptoms and signs of right-sided heart failure. Echocardiogram shows nondilated, non-hypertrophied ventricles with moderate to marked biatrial enlargement. Takotsubo cardiomyopathy (D), or stress cardiomyopathy, is a syndrome characterized by transient regional systolic dysfunction of the left ventricle, mimicking myocardial infarction, but in the absence of angiographic evidence of obstructive coronary artery disease or acute plaque rupture. It is most common in postmenopausal women after a physical or emotional stress.ung athlete Complaining of dyspnea on exertion (most common presenting symptom) PE will show harsh crescendo-decrescendo systolic murmur which increases in intensity with Valsalva maneuver and decreases with squatting Diagnosis is made by echocardiography Most commonly caused by an autosomal dominant genetic defect Treatment is refraining from vigorous physical activity, beta-blockers or calcium channel blockers Comments: Most common cause of sudden death in young athletes

A patient was recently diagnosed with histoplasmosis. Where would you suspect this patient recently traveled? A. Northeastern United States B. Ohio River Valley C. Pacific Northwest D. Southwestern United States

B. Ohio River Valley Patient with a history of travel to Ohio/Mississippi riHistoplasmosis is caused by Histoplasma capsulatum, a fungus that has been isolated from soil contaminated with bird or bat droppings. It is common along river valleys, especially the Ohio River and the Mississippi River valleys. Patients may be asymptomatic or show mild influenza-like illness, often lasting 1-4 days. Moderately severe infections typically present with pulmonary symptoms of cough, chest pain and frequently diagnosed as atypical pneumonia. Progressive disseminated histoplasmosis is seen in patients with underlying HIV. It is characterized by fever, loss of weight, cough, dyspnea, oropharyngeal ulcers, splenomegaly, hepatomegaly, and adrenal insufficiency. Treatment for mild to moderate histoplasmosis is itraconazole, and amphotericin B for severe infection. Patients living in or traveling to the Northeastern United States (A) are at increased risk of being exposed to ticks carrying Borrelia burgdorferi, the cause of Lyme disease. Patients exposed to dried pigeon dung in the Pacific Northwest (C) are at risk of developing cryptococcosis, especially if immunocompromised. Patients living in or traveling to the Southwestern United States (D) can be exposed to Coccidioides immitis or Coccidioides posadasii, both organisms growing in the arid soil that can cause coccidioidomycosis.ver valleys and exposure to bird/bat droppings X-ray will show solitary pulmonary calcification, hilar and mediastinal adenopathy Diagnosis is made by culture Treatment is itraconazole or amphotericin B

A 22-year-old man presents to the office for his one week follow up after his third concussion in four years. The patient plays rugby on his college intramural team. His most recent concussion was complicated by a period of loss of consciousness at the time of injury. He admits to having headaches and intermittent dizziness for the past couple of days. His headache appears to be tension-type in nature. He is eager to return to rugby and is requesting clearance to play. Which of the following is the most appropriate education to give your patient? A. Can return as headaches after a concussion are expected B. Can return as subsequent concussions increase resilience to future ones C. Cannot return as patient is currently symptomatic D. Cannot return as patient meets criteria for chronic traumatic encephalopathy

C. Cannot return as patient is currently symptomatic

A 55-year-old man involved in a motor vehicle collision reports shortness of breath during morning rounds. He incurred a left open tibial fracture as a result of the collision and underwent intramedullary rod placement 48 hours ago. Physical examination reveals a diaphoretic patient in mild distress, rales in the right lower lobe, tachycardia without murmurs, and a swollen left lower extremity. His vital signs are remarkable for a pulse of 110 beats per minute and respiratory rate of 26 breaths per minute. His medical history is remarkable for a recent diagnosis of lung cancer. Which of the following is most likely to confirm the diagnosis? A. Chest X-ray B. CT pulmonary angiography C. D-dimer D. Lupus anticoagulant

CT pulmonary angiography is considered by the American College of Radiology to be the standard of care for detection of pulmonary embolism. An embolism most often breaks off a thrombus. The three primary factors, Virchow's triad, classically considered in thrombus formation are endothelial cell injury, blood stasis, and hypercoagulation. The presence of any of these factors may lead to thrombus formation, with increasing risk as more factors are present. Most pulmonary emboli arise from a thrombus in the lower extremities, though thrombi can form anywhere. The Wells score for pulmonary embolism is often used to establish pretest probability. It assigns points based on the presence of malignancy, prior history of DVT or pulmonary embolism, hemoptysis, immobilization, recent surgery, symptoms of a DVT, tachycardia, and lack of alternative diagnosis. A CT pulmonary angiography is the diagnostic study of choice if pretest probability is high, or when D-dimer testing is positive. Additionally, ventilation-perfusion scanning may be used if a contraindication to CT pulmonary angiography or allergy to contrast media exists. Additional studies that aid in the diagnosis include ABG, serum troponin, ECG, echocardiography, brain natriuretic peptide, and chest X-ray. The most suggestive findings on ABG are hypoxemia, hypocarbia, and respiratory alkalosis. Treatment for pulmonary embolism is initially with intravenous heparin or therapeutic low molecular weight heparin and may be ordered prior to completion of confirmatory testing. Long-term anticoagulation is achieved with warfarin or other anticoagulants. Chest X-ray (A) may be an appropriate test for a patient with a low pretest probability, but the findings are not sensitive or specific enough to be considered diagnostic. Chest X-ray may aid in identifying an alternative diagnosis. D-dimer (C) is a byproduct of fibrin degradation and in low or moderate pretest probability is an appropriate test. However, in this scenario, the patient has a high pretest probability score. Thrombosis is ruled out if the D-dimer is not elevated. It is important to remember that several factors alter the D-dimer reliability, specifically trauma, surgery, hemolysis, and factor VIII deficiency. Lupus anticoagulant (D) is an antibody that tends to attach to endothelial cells and cause thrombosis. Presence of lupus anticoagulant may predispose to the development of thromboembolism, however establishing the diagnosis of pulmonary embolism takes priority to establishing etiology.

A 57-year-old woman presents to the clinic to establish care and reports seven months duration of sudden, abrupt onset of diarrhea. She also reports occasionally experiencing a "fluttery feeling" in her chest accompanied with facial flushing. Her medical history includes hypertension and a significant cough that has been present for two years. She has a 25 pack year tobacco use history. Chest X-ray reveals left lower lobe atelectasis. Which of the following is the most likely diagnosis? A. Angioedema B. Carcinoid tumor C. Cardiac dysrhythmia D. Irritable bowel syndrome

Carcinoid tumor is a type of neuroendocrine tumor involving a spectrum of neoplasms. The location of the tumor is usually in the gastrointestinal system, however, carcinoid tumors are also found in the lungs and elsewhere. Currently, pulmonary carcinoid tumors are understood to arise from a type of epithelial cell, called Kulchitsky cells, of endodermal origin. The tumors are able to secrete many different hormones, peptides, and amines but often lung carcinoid tumors secrete serotonin, which has vasoactive, coagulopathic, and muscular effects. Not every carcinoid tumor leads to the development of systemic symptoms, but when these arise it is termed carcinoid syndrome. The metabolism of serotonin includes 5-Hydroxyindoleacetic acid (5-HIAA) as a byproduct, which may be tested in the urine as a screening measure in patients with features suggestive of carcinoid syndrome. Generally, the main feature of pulmonary carcinoid tumors are related to pulmonary complaints, such as cough, hemoptysis, and shortness of breath, however many are asymptomatic. Features of carcinoid syndrome may include facial or trunk flushing, sudden and severe diarrhea, telangiectasias, wheezing, and palpitations. Other presentations of carcinoid syndrome may be present, and are determined by the substances secreted, such as presentation with confusion, weight gain, hyponatremia, and edema when a tumor secretes antidiuretic hormone. Imaging studies usually indicate an abnormality such as atelectasis or lesions on chest X-rays or computed tomography, and radionuclide studies with somatostatin may show increased lesion uptake. Workup must include evaluation of liver metastasis. Bronchoscopy may reveal a pink or purple lesion that is highly vascularized. Formal diagnosis relies on analysis of biopsied tissue. Further classification of the tumor, based on biopsy, will reveal either a typical carcinoid tumor, or an atypical carcinoid tumor. Atypical carcinoid tumors tend to have a higher mitotic index, and display more pleomorphism. Further classification and delineation of atypical versus typical forms are controversial, and different classification systems and subclassifications are used based on organizational standards. The cornerstone of treatment is with surgical resection. Chemotherapy and radiation therapy are primarily palliative measures, and may be implemented in the setting of poor surgical candidates or if distant metastases are present. Risk factors include family history of pulmonary carcinoid tumors, multiple endocrine neoplasia type 1, and smoking, which increases chances of atypical carcinoid tumors. Complications include pneumonia, metastasis, atelectasis, hypotension, thrombophilia, dysrhythmia, and even death. Angioedema (A) may be a presentation of a carcinoid tumor in the setting of carcinoid syndrome but is relegated to facial swelling and respiratory symptoms. Cardiac dysrhythmia (C) may also be present in the setting of carcinoid syndrome but likely would not account for the diarrhea. Irritable bowel syndrome (D) may be mistakenly diagnosed in a patient with a gastrointestinal carcinoid tumor, or in a patient with carcinoid syndrome. Certainly, the gastrointestinal symptoms are similar, but would not account for cardiopulmonary symptoms.

A 58-year-old woman, newly diagnosed with Sjӧgren's syndrome, returns to the clinic complaining of worsening dry mouth. At her last visit, you recommended she take frequent drinks of water throughout the day and try various over-the-counter products as well as artificial saliva, which she did, but her mouth symptoms persist. What is the most likely medication you would start her on? A. Cevimeline B. Cyclosporine drops C. Low dose prednisone D. Sulfasalazine

Cevimeline is a cholinergic agonist that is approved for dry mouth caused by Sjӧgren syndrome. Sjögren syndrome is an autoimmune disease that attacks the exocrine glands. Cevimeline works by stimulating nerves to increase the amount of saliva production. It is typically dosed at 30 mg three times daily. Most common side effects are excessive drooling and sweating, in which case the frequency of dosing is often reduced. Pilocarpine, also a cholinergic agonist, is a possible option for this patient. This medication can also be dosed three times a day but frequently has to be given four times daily for it to be effective. Cyclosporine drops (B) would only improve dry eye complaints and not dry mouth caused by Sjögren syndrome. Low-dose prednisone (C), or any type of steroid, has not been shown to have any type of impact on saliva production, even if caused by an autoimmune process. Sulfasalazine (D), while shown to be of benefit in other autoimmune processes, has not been shown to increase saliva production.

Which of the following represents the most common type of gallstone disease in the United States? A. Black pigment B. Brown pigment C. Calcium carbonate D. Cholesterol

Cholesterol is the primary component of the vast majority of gallstones in the United States. Cholesterol, phospholipid, and bile salts are secreted by the liver into bile. The gallbladder stores and concentrates bile. Cholesterol can supersaturate the bile during the process of concentration and precipitate cholesterol monohydrate crystals. Aggregation of crystals may form "sludge" and impair normal emptying, which may predispose to further coalescence of the crystals and formation of stones. Gallstones are classified based on the most abundant material present in their composition. It is believed conditions that predispose to gallbladder stasis or increased cholesterol secretion in bile are contributory to the development of cholelithiasis. Pregnancy is a known risk factor for stasis due to increased adiposity and increased progesterone, which slows gallbladder contractility. Additional risks for stasis include neurological injury, fasting or severely restricted caloric intake, use of somatostatin analogues, and structural abnormalities. Factors that predispose to increased cholesterol excretion include obesity, medications such as estrogen or fibrates, parenteral nutrition, and rarely genetic defects causing specific defects in metabolism and processing of cholesterol secretion into bile. Female sex is also a risk factor, largely believed to be related to hormonal influences. Black pigment (A) gallstones are the second most common type of gallstone in the United States. They result from excessive secretion of unconjugated bilirubin into bile, which tends to combine with calcium to produce crystalline deposits that become stones. It is associated with excessive heme concentrations, such as in cirrhosis or hemolysis. Brown pigment (B) stones are more common in Asia and may be associated with liver fluke infestation. Additionally, brown pigment stones often form in the common bile duct. Calcium carbonate (C) is a rare type of gallstone. It may have a higher prevalence in the pediatric population but is still uncommon compared to other stone types.

Which of the following patients with congestive heart failure is most in need of hospitalization? A. A 65-year-old man with stable angina and two pounds of recent weight gain B. A 68-year-old woman whose internal cardiac defibrillator went off five days ago C. A 70-year-old man with 2+ pedal edema and dyspnea on exertion D. A 75-year-old woman with blood pressure of 80/60 mmHg and confusion

Congestive heart failure occurs when cardiac output fails to keep up with the metabolic needs of the tissues. Heart failure may be due to filling defects, poor contractility, or a combination of the two. Patients who suffer congestive heart failure can present with dyspnea, fatigue, orthopnea, peripheral edema, distended neck veins, a sustained and laterally displaced apical impulse, audible extra heart sounds, bibasilar crackles or dullness to percussion at the lung bases. Treatment of congestive heart failure includes addressing the causative factors. Patients with stable congestive heart failure can be treated on an outpatient basis with diuretics, angiotensin-converting enzyme inhibitors, positive inotropic agents, and beta-blockers. Patients with congestive heart failure should be advised to decrease sodium intake, monitor daily weights, and engage in light physical activity as tolerated. Patients with heart failure who are in need of hospitalization include patients who are suffering dyspnea at rest, acute coronary artery syndrome, hemodynamically significant dysrhythmias, or acute decompensation signified by low blood pressure, altered mental status, or worsening renal function. Patients who are suffering new onset acute heart failure with signs of congestion, or patients with long-standing heart failure that are suffering worsening systemic or pulmonary congestion, multiple internal cardiac defibrillator firings, or major electrolyte imbalances can also be considered candidates for hospitalization. A 65-year-old man with stable angina and two pounds of recent weight gain (A) is an example of a heart failure patient who may or may not need hospitalization, based on his current hemodynamic status. Patients with evidence of congestion (such as recent weight gain) and who have comorbidities (such as angina) can be considered candidates for hospitalization if they show other signs of decompensation, such as increased dyspnea or worsening renal function. If the patient's symptoms have remained stable, the clinician can choose to treat these patients on an outpatient basis with increased fluid restriction, adjustments in diuretic dosage, and frequent follow-up. A 68-year-old woman whose internal cardiac defibrillator went off five days ago (B) does not require hospitalization, since the defibrillating event is several days old and the patient has no reported symptoms. She does, however, require evaluation in the office to assess her need for medication and lifestyle adjustment, as well as possible evaluation of the defibrillator device to ensure proper working order. If the patient continues to experience frequent firings of her internal cardiac defibrillator, she will require hospitalization to determine the best regimen for stabilization of her worsening cardiac function. A 70-year-old man with 2+ pedal edema and dyspnea on exertion (C) is an example of a typical heart failure patient. If this man does not have dyspnea at rest and is hemodynamically stable, he is not in need of hospitalization. The 2+ pitting edema signifies that he is fluid overloaded and needs adjustment of his diuretic medications and diet, which can be addressed on an outpatient basis.

The serum creatine kinase-MB peaks after an acute myocardial infarction in which of the following time frames? A. 16-24 hours B. 48-72 hours C. 4-6 hours D. 8-12 hours

Creatine kinase-MB (CK-MB) was the most commonly used serologic test for the diagnosis of myocardial infarction before troponin assays were widely available. CK-MB is predominantly found in the myocardium but also can be found in the skeletal muscle. It begins to rise 3 to 12 hours after the onset of ischemia and reaches a peak after an acute myocardial infarction in 16 to 24 hours. In patients with ischemic symptoms and without skeletal muscle damage, CK-MB elevations are relatively specific for myocardial infarction. CK-MB elevations return to baseline after 48 to 72 hours whereas troponin elevations can persist for as long as 10 to 14 days. This means CK-MB can be used to detect early reinfarction if levels rise again after falling. Timeframes of 4-6 hours (B), 8-12 hours (D), and 48-72 hours (C) are incorrect for the serum creatine kinase-MB (CK-MB) to rise to a peak after an acute myocardial infarction.

Which of the following statements is most correct regarding total parenteral nutrition in a patient with Crohn disease? A. Patients tolerate enteral feedings well after total parenteral nutrition is discontinued B. Total parenteral nutrition carries the risk of deep vein thrombosis and cardiomyopathy C. Total parenteral nutrition is an alternative to chronic low dose steroids D. Total parenteral nutrition is not recommended in those with short bowel syndrome

Crohn's disease is a chronic, inflammatory disease of the bowel involving any part of the gastrointestinal tract (mouth to anus), but with a predilection for the distal ileum and colon. The mucosal inflammation of Crohn's disease is transmural, leading to the formation of fistulas, abscesses, and stricture formation. Affected segments of bowel are often separated by segments of non-diseased bowel and are sometimes referred to as "skip lesions". Symptoms of Crohn's disease can include chronic diarrhea, fever, weight loss, and abdominal pain. Diagnosis is with colonoscopy, upper endoscopy, CT enterography or magnetic resonance enterography. Crohn's disease can be divided into four disease states, with each state necessitating differing treatment strategies. The most mild state is asymptomatic remission, where patients have no symptoms either after successful medical or surgical management, or spontaneous remission. These patients can be maintained with daily dosing of a drug from the 5-ASA class, such as mesalamine. The drug choice will be dependent on the disease site. Patients with mild-to-moderate Crohn's disease will manifest varying degrees of symptoms but will be able to tolerate oral nutrition without dehydration, significant (>10 percent) weight loss, abdominal tenderness, mass, obstruction, or toxicity. Many treatment options exist for these patients, including 5-ASA drugs, corticosteroids, antibiotics, probiotics, antidiarrheal medications and dietary changes. Patients who have failed treatment for mild-to-moderate disease or who manifest severe abdominal pain, significant weight loss, fever, nausea and vomiting, or fever are considered to have moderate-to-severe Crohn's disease. These patients often require hospitalization for intravenous glucocorticoids and often require treatment with biologic agents to obtain remission. Fulminant Crohn's disease is marked by such symptoms as high fevers, persistent vomiting, signs of abscess formation, peritoneal signs, intestinal obstruction, or patients whose symptoms persist despite glucocorticoid therapy or despite therapy with a biologic agent. Some of these patients will require surgical management. Many of these patients will require chronic, low-dose steroid therapy. Total parenteral nutrition can be considered as an alternative to chronic low-dose glucocorticoids. Many patients who receive total parenteral nutrition achieve remission, although often their symptoms will return once enteral feedings are resumed. Patients tolerate enteral feedings well after total parenteral nutrition is discontinued (A) is false. On the contrary, many patients relapse once total parenteral nutrition is stopped. For this reason, total parenteral nutrition is occasionally administered long-term in refractory Crohn's disease. Long-term parenteral nutrition comes with certain risks, such as line sepsis and thrombophlebitis, as well as hypertension, electrolyte imbalance, increased alkaline phosphatase and bilirubin, anemia, tachycardia, fever, dermatitis, hyperglycemia, and vomiting. Deep vein thrombosis and cardiomyopathy (B) are not recognized side effects of total parenteral nutrition. Total parenteral nutrition is not recommended in those with short bowel syndrome (D) is incorrect. Short bowel syndrome is the result of the removal of large portions of the small intestine. Patients with Crohn's disease are prone to abscess formation or stricture formation and at times surgical resection of the affected intestinal portion is necessary. When large portions of the small intestine are removed, malabsorption is common. This malabsorption can lead to life threatening nutritional deficiencies. In such cases, total parenteral nutrition may be life-saving.

Sildenafil is prescribed for an otherwise healthy 54-year-old man who is diagnosed with erectile dysfunction. Which of the following is a potential adverse effect of this medication? A. Constipation B. Cyanopsia C. Prostate specific antigen elevation D. Urinary retention

Cyanopsia, or blue discoloration of vision, is a potential side effect of sildenafil. The effect may be more likely with higher dosages of medication and is usually temporary. Sildenafil inhibits phosphodiesterase type 5 (PDE5), thereby increasing nitric oxide levels and cyclic guanosine monophosphate, which results in smooth muscle relaxation. Sildenafil is used most commonly for erectile dysfunction, however, it may also be used to treat pulmonary hypertension. Cyanopsia is hypothesized to be related to the vascular effects of the inhibition of PDE5, the effects of phosphodiesterase type 6 on the retina, and potentially to retinal exposure to elevated levels of cyclic guanosine monophosphate. Cyanopsia is uncommon but is a potential adverse effect from any PDE5 medication. Additional common side effects include hypotension, flushing, headache, dyspepsia, hearing loss, and nasal congestion. If phosphodiesterase medication is contraindicated or not tolerated, prostaglandin or papaverine cavernosa injections may be a treatment option, but they carry a risk of penile plaque formation. Additionally, vacuum devices may be useful but require significant planning and teaching on how to operate. Constipation (A) has not been significantly associated with sildenafil use. The most commonly reported gastrointestinal adverse effect is dyspepsia. Prostate-specific antigen elevation (C) is not directly related to sildenafil use. Ejaculation is known to cause elevations in prostate-specific antigen, and it is recommended men not ejaculate for at least 48 hours prior to testing. Urinary retention (D) is also not known to be associated with sildenafil, or any PDE5 inhibitor. Evidence suggests inhibition of PDE5 may help reduce the severity of lower urinary tract symptoms.

A 57-year-old woman presents to the clinic complaining of difficulty getting out of bed in the morning and difficulty climbing the stairs into the house. She also reports a weight loss of 20 pounds over the past six months. You suspect a diagnosis of polymyositis. Which of the following physical exam findings is most consistent with the diagnosis of polymyositis? A. Painful proximal muscles without weakness B. Positive Homan's sign C. Several positive tender points D. Weakness of the proximal muscles without pain

D. Weakness of the proximal muscles without pain Polymyositis Patient will be complaining oPolymyositis is an idiopathic, autoimmune, inflammatory myopathy (does not involve skin). A progressive symmetrical weakness of the proximal muscles without pain is the hallmark symptom of polymyositis. Affected muscles typically include neck flexors, shoulder girdle, and pelvic girdle muscles. History typically includes difficulty with the following: getting up from a seated position, getting out of bed, going up or down stairs, and lifting heavier objects. Additional clinical manifestations may include constitutional symptoms (fatigue, fever, weight loss), dysphagia, polyarthritis, interstitial pulmonary disease, respiratory insufficiency due to diaphragmatic and chest wall muscle weakness, and conduction abnormalities. Patients with polymyositis are also at increased risk of malignancy. Typical lab findings include elevated levels of muscle enzymes (Creatine kinase, lactate dehydrogenase, aldolase, aspartate aminotransferase, alanine aminotransferase). Antinuclear antibody (ANA) may be elevated as well. Erythrocyte sedimentation rate is typically normal. Myositis-specific autoantibodies include antisynthetase antibodies, including anti-Jo-1 (most common antibody and associated with interstitial lung disease (ILD), Raynaud phenomenon, arthritis, and mechanic's hands), antibodies to signal recognition particle (almost exclusively seen with polymyositis), and antibodies to Mi-2 (specific for dermatomyositis). Electromyography (EMG) may be abnormal in 90% of patients. Biopsy of one of the affected muscles, typically one of the quadriceps muscles of the thigh, is required for the diagnosis of polymyositis. Treatment may include corticosteroids, immunosuppressive agents, and physical therapy. Painful proximal muscles without weakness (A) is associated with polymyalgia rheumatica, especially in a patient over 50 years of age. Typical lab findings are an elevated erythrocyte sedimentation rate and normal creatinine kinase. Positive Homan's sign (B) is posterior lower leg pain with dorsiflexion of the foot related to thrombosis. There is no association with polymyositis. Several positive tender points (C) would indicate a possible diagnosis of fibromyalgia. Weakness is not usually associated with fibromyalgia and labs are typically normal.f progressive weakness of three months duration PE will show diffuse tenderness of proximal muscles in the shoulder girdle and pelvic girdle without rash or joint swelling and neurologic examination yields normal results Labs will show (+) Anti-Jo, (+) Anti-SRP Treatment is corticosteroids

Which of the following electrocardiographic findings is the hallmark of acute pericarditis? A. Diffuse ST elevation B. Irregularly irregular R-R intervals with no distinct P waves C. Prolonged QT interval D. Tall peaked T waves

Electrocardiogram changes in patients with acute pericarditis are reflective of epicardial inflammation and typically progress through four stages. Stage 1 is characterized by diffuse ST elevation with reciprocal ST depression in leads aVR and V1. The ST elevation is typically concave up. PR depression in the limb leads and left chest leads, usually V5 and V6, also occur in stage 1. PR segment depression is more specific, but not very sensitive for the diagnosis of acute pericarditis. Stage 2 is characterized by normalization of the ST and PR segments. Stage 3 is characterized by the development of diffuse T wave inversions, usually after the ST segments have become isoelectric. Stage 4 is represented by ECG normalization or persistence of T wave inversions. The finding of irregularly irregular R-R intervals with no distinct P waves (B) is the hallmark of atrial fibrillation, not acute pericarditis. Atrial fibrillation is the most common cardiac dysrhythmia and can cause reduced cardiac output and thromboembolic stroke. Prolonged QT interval (C) is the hallmark of long QT syndrome, not acute pericarditis. Long QT syndrome is a disorder of myocardial repolarization that can be congenital or acquired. It is characterized by a prolonged QT interval on ECG and can cause a lethal cardiac dysrhythmia called Torsades de pointes. Tall peaked T waves (D) are a hallmark of hyperkalemia, not acute pericarditis. Hyperkalemia causes a series of ECG changes that progress as the electrolyte imbalance worsens. The first findings include tall peaked T waves with a shortened QT interval. If left untreated, this is followed by a progressive lengthening of the PR interval and QRS duration, P wave disappearance, a sine wave pattern and eventual flat line.

A 64-year-old man presents to the clinic reporting a progressive tremor affecting his hands. He states the symptom onset was approximately seven years ago. He states his tremor is now affecting his writing but appears to improve in the evenings after a glass of wine. He has no significant medical history. He does report that his mother had a similar problem but never sought medical attention. On exam, he has no tremor at rest but develops an increasing amplitude tremor in both upper extremities when reaching for items. No bradykinesia or rigidity is present. The rest of his examination is unremarkable. Which of the following is the most likely diagnosis? A. Essential tremor B. Huntington disease C. Parkinson disease D. Wilson disease

Essential tremor is the most common movement disorder, and is usually a kinetic or postural tremor that progresses with time. The pathophysiology is currently poorly understood and likely heterogeneous, but likely involves an oscillating network of components of the cerebellothalamocortical circuit, as well as possible neurodegeneration involving the cerebellum and the locus coeruleus. Genetic causes have also been implicated, however there is no single defect or anatomical abnormality that explains all variants of presentation. Diagnosis is made clinically on the basis of family history, as an autosomal dominant transmission occurs in many cases, and clinical presentation. Several diagnostic criteria have been created by the Tremor Investigation Group, Washington Heights-Inwood genetic study of essential tremor, and the Movement Disorder Society. The typical presentation involves a history of upper extremity tremor that progresses in amplitude over time. Head, voice, jaw, facial, and lower extremity tremor may also be involved, though this is less common. The tremor is usually bilateral and symmetric, however, slight asymmetry may be noted. The amplitude of the tremor may worsen as the patient reaches for an item, is provoked with stress or certain activities, however, a slight degree of voluntary control may also be exhibited. A characteristic that is common of essential tremor is the cessation of symptoms with ethanol consumption. The mechanism behind this is not fully understood. Magnetic resonance imaging or computed tomography are not usually required for typical presentations but may be beneficial in excluding other causes. The greatest risk factor for the development of essential tremor is a positive family history, although Parkinson disease and age are also risk factors. There is emerging evidence that essential tremor may be associated with degeneration of neural tissue, and associated conditions that include cognitive deficits, dementia, and psychiatric disorders. Treatment is recommended when symptoms interfere with activities or quality of life. Initial treatment may be episodic, with primidone or propranolol, however, long-term treatment is usually with propranolol or topiramate. Deep or superficial brain stimulation may be used for cases refractory to medication, and even more rarely, unilateral thalamotomy may be employed. Other treatment modalities include physical and occupational therapy, as well as psychological therapy as there is a very high rate of depression and anxiety in patients with essential tremor. Referral to a neurologist is warranted in cases that do not respond to initial medications or in the presence of an atypical presentation. Complications include a high rate of disability, diminished quality of life, depression, mood disorders, worsening tremor, and potentially increased mortality. There is also a correlation with dementia. Huntington disease (B) is a progressive autosomal dominant degenerative disease of the brain with significant psychiatric and motor symptoms, and is associated with changes in the caudate nucleus and putamen on imaging. Parkinson disease (C) is associated with the hallmark symptoms of bradykinesia, "cogwheel" rigidity and resting tremors, which are not usual features of essential tremor, and involves death of the substantia nigra dopaminergic neurons. Wilson disease (D) is an autosomal recessive metabolic disorder that results in abnormal deposition of copper in the body tissues, and causes liver disease and psychomotor changes similar to Parkinson disease but in a young patient. Additionally, a Kaiser-Fleischer ring, a brown ring on the periphery of the cornea, is often noted from copper deposition in the eye.

A 21-year-old man presents with a complaint of a painless "lump" near his anus. He reports occasional blood on the toilet paper when he wipes after straining to have a bowel movement. He also complains of intermittent peri-anal itching. On inspection, you note a pink, circular lesion visualized on the anus that is not pedunculated. It is firm and non-tender. What is the most likely diagnosis? A. Anal abscess B. Anal skin tag C. External hemorrhoid D. Internal hemorrhoid

External hemorrhoids represent dilated hemorrhoidal veins that originate below the dentate line. Combined, internal and external hemorrhoids are the most common cause of lower GI bleeds. External hemorrhoids can be visualized at the anal margin and are typically pink and painless until they become thrombosed. Thrombosis is usually preceded by constipation and straining. A thrombosed external hemorrhoid will appear to be blue or purple in color. Bleeding associated with external hemorrhoids is classically described as bright red blood on the toilet tissue observed after wiping. Treatment options for external hemorrhoids range from increased fiber intake and cortisone suppositories to surgical intervention depending on the severity of the clinical presentation. Anal abscesses (A) present as a painful, fluctuant anal mass with or without a fever that is caused by infection of the anal glands. Anal skin tags (B) are commonly found at the anal margin and are described as flesh-colored, painless, pedunculated lesions. Internal hemorrhoids (D) are anatomically located superior to the dentate line and usually require a scope to be visualized (unless they prolapse). Internal hemorrhoids are typically painless.

A 65-year-old woman presents to the ED with sudden onset of right eye pain and blurred vision. Physical examination reveals circumcorneal injection, a 7 mm right pupil that is unresponsive to light and an intraocular pressure of 35 mm Hg. Which of the following is the gold standard test to confirm the diagnosis? A. Dark room provocation B. Dilated fundus examination C. Gonioscopy D. Slit lamp grading

Gonioscopy is considered the gold standard in the diagnosis of acute angle closure glaucoma, an ocular emergency that this patient is experiencing. This method allows the examiner to examine the angle that is formed between the cornea's posterior surface and the iris' anterior surface. An experienced provider, such as an ophthalmologist, should perform this test. Care must be taken to not shine light directly onto the pupil as the angle could widen and create an error in measurement. Dark room provocation (A) is a process in which a patient is placed in a dark room with the head in the prone position, thus displacing the lens forward and encouraging dilation of the pupil. This test is not recommended. Dilated fundus examination (B) should not be utilized to diagnose acute angle closure glaucoma and, in fact, can worsen the condition. Slit lamp grading (D) can be used to estimate the angle width by shining light on the anterior chamber but is not considered as accurate as gonioscopy.

A 45-year-old hypertensive man presents with a persistent, dry cough. He is currently taking clonidine 0.1 mg three times daily, amlodipine 10 mg daily, lisinopril 40 mg daily, and aspirin 81 mg daily. Which of his medications is most likely contributing to his symptoms? A. Amlodipine B. Aspirin C. Clonidine D. Lisinopril

Hypertension is sustained elevation of systolic blood pressure, diastolic blood pressure or both. Development of primary hypertension is influenced by age, genetics, stress, and dietary sodium intake. Although primary hypertension is much more common, hypertension can develop secondary to ingestion of drugs such as nonsteroidal anti-inflammatories, amphetamines, or cocaine. Hypertension can also develop secondary to certain disease states, such as pheochromocytoma, diabetic nephropathy, coarctation of the aorta, sleep apnea, and thyroid disease. Before making the diagnosis of primary hypertension, a limited work-up of secondary causes based on history and physical exam should be done in certain populations including especially young age, especially old age, abrupt onset or resistant hypertension, low serum potassium or physical findings of an abdominal bruit. Patients with a new diagnosis of primary hypertension should be counseled on lifestyle modifications that may lower blood pressure. These include dietary changes, weight loss, limited alcohol use, and exercise. A diet low in sodium (under 2500 mg per day) and rich in potassium, calcium, and magnesium should be encouraged. Medical treatment of hypertension should begin after three to six consecutive high blood pressure readings over several months and failure of lifestyle modifications in the absence of end-organ damage. The first-line agents of choice are thiazide diuretics, long-acting calcium channel blockers, angiotensin-converting enzyme (ACE) inhibitors and angiotensin II receptor blockers (ARBs). Drugs to be used as first-line agents in African-American and Hispanic patients are calcium channel blockers and thiazide diuretics. Drugs to be used as first-line agents in patients with diabetic nephropathy or nondiabetic chronic kidney disease complicated by proteinuria are angiotensin-converting enzyme (ACE) inhibitors or angiotensin II receptor blockers (ARB). The most common side effects of angiotensin-converting enzyme inhibitors (such as lisinopril) are dizziness, dry cough, headache, and hyperkalemia. Among the most serious side effects of angiotensin-converting enzyme inhibitors are angioedema, acute renal failure, and teratogenicity. Amlodipine (A) is a calcium channel blocker. The most common side effects of this class of medicine include peripheral edema, pulmonary edema, and headache. Caution should be used when prescribing this drug to patients with congestive heart failure, severe aortic stenosis, or acute myocardial infarction. Aspirin (B) is an inhibitor of platelet aggregation and an inhibitor of synthesis of prostaglandins by cyclooxygenase. Common side effects of aspirin include increased bleeding tendency, gastrointestinal irritation, and tinnitus. Aspirin is contraindicated in lactating women and in patients with active hemorrhage or gastrointestinal bleeding, hemolytic anemia, hemophilia, nasal polyps associated with asthma, ulcerative colitis, and thrombocytopenia. Clonidine (C) lowers blood pressure through central sympatholytic action. Postsynaptic action on alpha-2 adrenergic receptors also results in analgesia and reduced impulsivity. Dry mouth, headache, and somnolence are common side effects of clonidine. An epidural formulation of clonidine exists for pain control but is contraindicated in obstetric postpartum patients and in perioperative patients due to the risk of severe hypotension and bradycardia.

Which laboratory study is required to confirm the diagnosis of thalassemia? A. Bone marrow biopsy B. Hemoglobin electrophoresis C. Iron studies D. Peripheral smea

Hemoglobin electrophoresis is the confirmatory test used in the diagnosis of thalassemia. Thalassemia is a hereditary disease that results in a microcytic anemia based on a defect in the alpha or beta-globin chains. The defect can either be a gene deletion or a point mutation. Clinical presentation can vary from mild symptoms to severe disease depending on the abnormality. Alpha thalassemia can manifest as mild disease in carriers to moderate disease in individuals with one alpha-globin chain. Beta thalassemia (Cooley anemia) manifests between the ages of four to six months as fetal hemoglobin drops and adult hemoglobin increases. These patients can have severe hemolytic anemia. Laboratory studies include complete blood count, iron studies, peripheral smear, and hemoglobin electrophoresis. Serum iron levels can be normal or increased in patients with thalassemia. The mean corpuscular volume is usually lower in individuals with thalassemia in comparison to individuals with iron deficiency anemia. Hemoglobin electrophoresis is used to evaluate for normal hemoglobin, hemoglobin H, hemoglobin A2, and hemoglobin F. Iron chelation and stem cell transplants are the treatment of choice. Patients with beta thalassemia need transfusions to maintain a normal hemoglobin level. It is important to avoid iron in individuals with thalassemia as it can lead to iron overload. Genetic counseling should be discussed in individuals who are planning to have children. Bone marrow biopsy (A) is incorrect. While bone marrow examination would show erythroid hyperplasia in an individual with thalassemia, it is not required for diagnosis. Iron studies (C) would be performed in an individual with thalassemia, but would not be used to confirm the diagnosis. Patients with thalassemia can have normal-to-elevated iron levels, which puts them at risk of iron overload when misdiagnosed as having iron deficiency anemia. Peripheral smear (D) commonly reveals target cells in patients with thalassemia but is not the confirmatory test for the disease.

A 70-year-old man with a history of HIV infection presents to the office for routine testing of his CD4 count. His last CD4 count was 90 cells/mcL and the results from this visit show a CD4 count of 45 cells/mcL. Which of the following opportunistic infections are patients prone to developing when their CD4 count drops below 50 cells/mcL? A. Histoplasmosis B. Kaposi sarcoma C. Mycobacterium avium complex D. Toxoplasmosis

Human immunodeficiency virus (HIV) infection is a blood-borne infection that is characterized by cellular immune deficiency caused by the depletion of helper T lymphocytes. The loss of these cells results in the development of opportunistic infections and neoplastic processes. Those with CD4 counts below 200 cells/mcL are considered to have developed acquired immunodeficiency syndrome (AIDS) and require prophylactic treatment for a variety of opportunistic infections. Many infections can be prevented by taking antibiotics or antifungal medications. Patients with CD4 counts less than 50 cells/mcL are at risk for infection with mycobacterium avian complex and cytomegalovirus retinitis. Histoplasmosis (A), a lung infection caused by Histoplasma capsulatum fungal spores found in the soil and in droppings of bats and birds, and toxoplasmosis (D), a parasitic infection caused by eating undercooked meat and exposure to infected cat species, are opportunistic infections that may appear with CD4 counts lower than 150 cells/mcL and 100 cells/mcL, respectively. Kaposi's sarcoma (B) is a type of cancer with skin manifestations that is commonly seen in HIV positive patients and is not associated with specific CD4 counts.

A 31-year-old woman, newly diagnosed with rheumatoid arthritis, presents to the clinic to start treatment for her disease. Other than her joint pain and fatigue, she is otherwise healthy and denies any other complaints. She has two children and, although not currently trying to conceive, she is considering having one more child in the near future. What is the most likely medication you would start her on? A. Etanercept B. Hydroxychloroquine C. Leflunomide D. Methotrexate

Hydroxychloroquine is generally considered to be safe until the patient knows they are pregnant. It is then advised that they stop all medications for their rheumatoid arthritis, not only for precautionary measures, but most patients typically have low or no disease activity while they are pregnant. In the event that a patient continues to experience disease activity while pregnant, hydroxychloroquine is considered generally safe to use throughout pregnancy based on analysis of over 400 pregnancies that showed no increased risk for birth defects. Hydroxychloroquine belongs to a class of drugs called DMARDs (Disease Modifying Anti-Rheumatic Drugs) and is used in a number of diseases, including rheumatoid arthritis, systemic lupus erythematosus, mixed-connective tissue disease and other autoimmune diseases. Originally used as an antimalarial drug, its mechanism of action in rheumatic disease is still largely unknown. Etanercept (A) was the first biologic approved for market in rheumatoid arthritis. Placebo controlled trials show its superiority over methotrexate and other DMARD therapy. It is generally considered safe in patients trying to become pregnant and then recommended to discontinue once pregnancy is confirmed. Due to cost and concerns about increased risk of infection, an adequate trial of DMARD therapy is still standard of care first before biologic therapy. Leflunomide (C) and methotrexate (D) are both DMARDs and known teratogenic agents that would harm a developing fetus. Methotrexate is a treatment for ectopic pregnancy. Due to their extended half-lives and effects on rapidly reproducing cells, it is advised that they are discontinued for at least one menstrual cycle before attempting to become pregnant.

A 50-year-old man with a body mass index of 52 kg/m² presents to the clinic complaining of daytime sleepiness and dyspnea on exertion. His peripheral oxygen saturation on room air is 93%. Further testing reveals obesity hypoventilation syndrome with concurrent obstructive sleep apnea. The patient is put on a weight loss regimen which includes diet and exercise. In addition, which of the following is the best choice for initial therapy in this patient? A. Nocturnal bilevel positive airway pressure B. Nocturnal continuous positive airway pressure C. Oral theophylline dosed every twelve hours D. Twenty-four hour supplemental oxygen

Obesity hypoventilation syndrome is defined as alveolar hypercapnia in an awake person which cannot be attributed to any disease process other than obesity. As many as half of the patients with a body mass index over 50 kg/m² suffer obesity hypoventilation syndrome, although any patient with a body mass index over 30 kg/m² may be affected. The majority of patients with obesity hypoventilation syndrome also have concurrent obstructive sleep apnea. Signs and symptoms of obesity hypoventilation syndrome include a thick neck, daytime hypersomnolence, reduced memory and concentration, and loud snoring. Patients with severe or late-stage disease may show signs of right heart failure (cor pulmonale), such as jugular venous distention, hepatomegaly, pedal edema, or dyspnea on exertion. Patients with obesity hypoventilation syndrome have increased cardiovascular morbidity and mortality rates. Diagnosis is made by first excluding other causes of daytime hypercapnia and alveolar hypoventilation. Serum studies can rule out electrolyte disturbances, eosinophilia, or thyroid dysfunction. Pulmonary function tests can rule out chronic obstructive pulmonary disease and interstitial lung disease. Patients should also be examined for bony deformities of the chest (kyphoscoliosis) and questioned about the use of sedatives and alcohol. First-line treatment of obesity hypoventilation syndrome includes weight loss and positive airway pressure therapy. Nocturnal continuous positive airway pressure is the treatment of choice in patients with concurrent obesity hypoventilation syndrome and obstructive sleep apnea. Nocturnal bilevel positive airway pressure (A) is the treatment of choice for patients with concurrent obesity hypoventilation syndrome and obstructive sleep apnea who fail to improve on continuous positive airway pressure, and for patients with obesity hypoventilation syndrome and sleep-related hypoventilation (sleep-disordered breathing not associated with obstructive events). Oral theophylline dosed every twelve hours (C) is not considered first-line therapy in obesity hypoventilation syndrome. Respiratory stimulants, such as acetazolamide and medroxyprogesterone, can improve alveolar ventilation and are sometimes prescribed as a last resort in patients who fail to achieve adequate ventilation with positive airway pressure and weight loss. Theophylline is also a respiratory stimulant, but has not been studied in patients with obesity hypoventilation syndrome. Twenty-four-hour supplemental oxygen (D) is sometimes prescribed for patients with severe hypoventilation syndrome who cannot reach ventilation goals with positive airway pressure treatment and weight loss alone. Supplemental oxygen should never be used alone in patients with obesity hypoventilation syndrome, because it may increase hypercapnia, and does not address the impaired respiratory mechanics or altered chemoreceptor responsiveness that cause daytime hypoventilation in these patients.

A 48-year-old man with a history of hypertension presents to the emergency department with a headache and blurring of vision. He reports poor compliance with his blood pressure medications due to lack of access to a primary care provider. On presentation, his vital signs are 36.9°C, HR 87, BP 190/120, RR 16. He denies chest pain or shortness of breath. Physical examination is unremarkable. His laboratory studies are notable for a creatinine of 2.5 U/L. Which of the following is contraindicated in the control of this patient's blood pressure? A. Esmolol B. Labetalol C. Nicardipine D. Nitroprusside

Hypertensive emergency is an acute elevation of blood pressure associated with end-organ damage, typically of the kidneys, brain, heart, aorta, or eyes. This is in contrast to hypertensive urgency, which is an elevation in blood pressure without signs of end-organ damage. Hypertensive emergency is most common in patients with chronic hypertension who are noncompliant with medications. It results from the eventual overwhelming or failure of compensatory mechanisms for end-organ perfusion in the setting of increased mean arterial pressure. This leads to fibrinoid necrosis and end-organ hypoperfusion and ischemia. Blood pressure control is the cornerstone of management for all hypertensive emergencies, but the method of blood pressure control differs based both on the etiology of the hypertensive emergency and the evidence of end-organ damage. In acute renal failure, as seen in the patient above, the goal for blood pressure control is to decrease the mean arterial pressure by no more than 20% acutely to avoid further organ hypoperfusion. Preferred agents include calcium channel blockers such as nicardipine or clevidipine. Nitroprusside should be avoided in the setting of renal failure due to the risk of cyanate and thiocyanate toxicity. Esmolol (A) is a beta-blocker and labetalol (B) is a nonselective alpha- and beta-blocker that can be used for blood pressure control in hypertensive emergencies. Both are safe to use in the setting of renal failure. Nicardipine (D) is a calcium channel blocker that is also safe for use in the setting of renal failure.

A 55-year-old man presents to the clinic, reporting progressive pain in his lower back, pelvis, and right hip that started eight months ago. He reports pain while walking and it feels worse at night. Ibuprofen provides some relief. On physical exam, he has tenderness and warmth at the lumbar spine, right iliac wing, and right greater trochanter. A mild varus deformity of the right thigh is noted. Lab analysis reveals high serum alkaline phosphatase and radiographs show lytic lesions. Which of the following is the most likely cause of these findings? A. Chronic soft tissue degeneration B. Hypervascular bone C. Osteolytic infectious process D. Soft tissue mass effect

Hypervascular bone, due to the presence of Paget disease of the bone, initially causes osteolysis with subsequent overactive osteoblastic activity leading to bone deposition and resulting in high bone turnover. The exact etiology of Paget disease is unknown, but there appears to be a genetic and geographical predisposition. With the exception of Scandinavia, people of northern European lineage have the highest disease prevalence. The initial effects of Paget disease are reflected as changes in osteoclasts, leading to focal osteolysis. As osteoclastic activity begins to subside, osteoblasts are activated, rapidly proliferate, and cause new bone and collagen synthesis. The collagen and bone deposits are significantly disorganized and have a mosaic appearance, sometimes described as "woven bone." The new bone is structurally weaker, more porous, hypervascular, and prone to fracture. Warmth, tenderness, and pain at affected sites may be noted. Continuous pain that is worse at night is common. Neurologic symptoms from nerve impingement resulting in spinal or cranial nerve deficits may be present as well. Asymmetry of the bones may also be noted. Most patients, however, are asymptomatic. The most common locations for Paget disease are the skull, spine, femur, pelvis (most commonly affected) and sacrum, but it can occur anywhere in the body and there may be multiple lesions undergoing different stages of remodeling. Diagnosis is made by plain radiographs. Lytic lesions, bony enlargement, cortical thickening, increased trabecular pattern, and deformity or bowing may be seen. Skull involvement typically begins with radiolucent areas followed by osteoblastic activity resulting in a classic 'cotton wool' appearance. Bone scintigraphy can identify active lesions and subsequent X-rays of highly active areas should be obtained for a definitive diagnosis. Magnetic resonance imaging may be helpful to evaluate for malignancy or to assess neurological complications if the axial skeleton is involved. Computed tomography may be used to identify fractures, particularly if there is articular involvement. Laboratory analysis may reveal elevated alkaline phosphatase. Biopsy is usually only required to assess lesions with a suspicious appearance or when medical therapy fails to provide relief of pain. Generally, treatment should be initiated if there are complications such as uncontrolled pain, fractures, hypercalcemia, high output heart failure, weight-bearing joints are affected, cranial or spinal lesions are present, alkaline phosphatase is significantly elevated, or if surgery is planned on an affected bone. Nitrogen-containing bisphosphonates are the primary agents used for treatment. Calcitonin may be used if contraindications exist for bisphosphonate use. Calcium and 25-hydroxyvitamin D levels should be obtained prior to treatment. Supplemental calcium and vitamin D3 are routinely administered throughout treatment. Acetaminophen or nonsteroidal anti-inflammatories, in addition to bisphosphonates, are usually given to treat pain. Surgery may be warranted in cases of significant deformity or axial skeleton involvement. Physical therapy and orthotics may be helpful if movement is impaired. Additional complications may include deafness, neuromuscular syndromes, arthritis, bleeding at lesion sites, and neoplastic conversion, particularly sarcoma. Routine serial alkaline phosphatase testing is recommended to monitor disease activity. Chronic degeneration of soft tissues (A) may lead to small subchondral cysts, but is not associated with high bone turnover. Degeneration may occur if pagetic lesions are near joints or have caused significant deformity leading to arthritis, but this is not the primary effect of Paget disease. An osteolytic infectious process (C), such as osteomyelitis, would likely have a more rapid progression, causing severe joint destruction within a short time, as well as fever and additional abnormal lab values (elevated white blood cell count, C-reactive protein, erythrocyte sedimentation rate). A soft tissue mass effect (D) may present in cases where tumor growth occurs, such as sarcoma, however, assessment of this patient reveals the characteristic bony changes on X-ray seen in Paget disease, and no soft tissue defect is noted.

A 59-year-old woman presents to the clinic reporting numbness in her hands and around her mouth, as well as muscle cramps, fatigue, and intermittent hoarseness. Her medical history includes hypertension, hyperlipidemia, diabetes, and a thyroid malignancy which was recently treated by surgical excision. On physical examination, she is noted to have an ataxic gait, hyperresponsive deep tendon reflexes, and appears to be anxious. Additionally, carpal spasms are noted when her blood pressure is taken. Her vital signs are within normal limits for her age. Results of a complete blood count and complete metabolic profile are notable for a low total calcium. Which of the following is the most likely diagnosis? A. Hyperparathyroidism B. Hypoparathyroidism C. Hypophosphatemia D. Hypothyroidism

Hypoparathyroidism is a deficiency of parathyroid hormone (PTH) that results in low levels of calcium, particularly low ionized calcium. Serum concentrations of ionized calcium, the most active form of calcium in the body, are tightly regulated predominantly by the parathyroid via secretion of PTH. This hormone primarily regulates calcium by directly inducing changes in kidney function and bone metabolism, and by indirectly acting on the intestines. PTH inhibits renal reabsorption of phosphate in the proximal tubule while simultaneously stimulating calcium reabsorption in the distal tubule. Additionally, PTH stimulates renal 1-alpha-hydroxylase production, the enzyme that catalyzes the production of 1,25-dihydroxyvitamin D, which acts as a hormone on intestinal epithelial cells and causes an increase in calcium binding protein, and thus facilitates calcium reabsorption. PTH induces a complex interaction in bones to initially cause osteoblasts at the osteocytic membrane to increase extracellular concentrations of calcium. PTH also activates osteoclasts to increase bone resorption and increase ionized calcium over a longer duration. The most common cause of hypoparathyroidism is iatrogenic primary hypoparathyroidism, specifically from surgical resection of the thyroid, parathyroid, larynx, and other neck structures. Other rare disease processes may cause hypoparathyroidism (primary or secondary), such as Wilson disease, hemochromatosis, thalassemia, alkalosis, hypomagnesemia, hypermagnesemia, genetic abnormalities, and gestational maternal hypercalcemia. Diagnosis is established with low PTH, low ionized calcium, and may be suggested by elevated phosphorus. Additional testing includes vitamin D, total calcium with albumin, protein, magnesium, and blood pH, among others, to investigate for other causes of low PTH and calcium. Clinical findings of hypoparathyroidism may include fatigue, a hoarse voice from bronchospasm, mood alterations, personality changes, seizures, nephrolithiasis, hyperactive deep tendon reflexes, and muscle spasms. Two classic exam findings are Chvostek sign, facial twitching caused by tapping around the facial nerve, and Trousseau sign, which is carpal spasming induced with obliteration of the brachial artery. Additionally, cardiac conduction abnormalities may precipitate dysrhythmias and heart failure. Death may result from untreated hypoparathyroidism. Treatment of primary hypoparathyroidism is usually with calcium and vitamin D, however, recombinant human parathyroid hormone is available. Treatment is targeted to achieve symptom relief and achieve a normal level of calcium, using an adjusted serum calcium or ionized calcium. Treatment of secondary hypoparathyroidism is directed at the cause. Hyperparathyroidism (A) is a condition of excess PTH, and results in hypercalcemia. Hypophosphatemia (C) may result from hyperparathyroidism due to the phosphaturic effect of PTH, therefore an elevated phosphate would be expected in hypoparathyroidism. Hypothyroidism (D) is not associated with low calcium and high phosphate, however, hyperthyroidism may cause renal loss of these substances.

An 80-year-old woman with a 36-pack-year history of smoking tobacco presents with a chief complaint of increasing dyspnea on exertion over the past six months. Physical exam reveals clubbed digits and bilateral lower lung crackles. Computed tomography shows honeycombing of the lung parenchyma. What is the most likely diagnosis? A. Bronchiectasis B. Chronic obstructive pulmonary disease C. Idiopathic pulmonary fibrosis D. Sarcoidosis

Idiopathic pulmonary fibrosis (IPF) is a type of fibrosing interstitial pneumonia that most commonly occurs in men over 50 years of age. Although it is linked to cigarette smoking and has been shown to occur in families, the exact cause is unknown. The pathogenesis of IPF is likely related to epithelial cell damage and improper repair leading to chronic and progressive symptoms. The most characteristic symptoms of IPF are a chronic nonproductive cough and gradual exertional dyspnea that usually develops over several months. Medical treatment for IPF has proven difficult, making for a poor prognosis, although nintedanib and pirfenidone seem to delay disease progression. This treatment coupled with supportive care, such as supplemental oxygen, can help maintain a normal level of activity for patients. Since treatment is often delayed by misdiagnosis, it is vital that providers perform a thorough physical exam and order appropriate images. Imaging on high resolution computed tomography (CT) reveals structural changes to the lung parenchyma, such as honeycombing, which refers to the appearance of clusters of cystic airspaces in the lungs. While physical exam may be unremarkable during early disease, it often reveals bibasilar crackles and digital clubbing as the disease progresses, and this clinical presentation along with typical findings on high resolution CT is enough to establish the diagnosis of idiopathic pulmonary fibrosis. Bronchiectasis (A) is a widening of the airways and destruction of the bronchial walls that results from recurrent infection. It often presents with unusually large volumes of foul-smelling or blood-streaked sputum with "tram tracks" present on chest X-ray. Chronic obstructive pulmonary disease (COPD) (B) often presents with increasing dyspnea on exertion in the setting of prolonged tobacco use, but it does not have the classic honeycomb appearance on CT. Sarcoidosis (D) is a disease that causes non-caseating granulomas throughout the body, resulting in a variety of symptoms. Lung imaging often shows bilateral hilar adenopathy, unlike the honeycombing seen in idiopathic pulmonary fibrosis.

A 26-year-old man presents with an acute severe asthma exacerbation. You decide to perform rapid sequence endotracheal intubation due to impending respiratory failure. The patient has no known medication allergies. Which of the following agents is most appropriate for induction in this case? A. Etomidate B. Ketamine C. Midazolam D. Propofol

In a majority of the population, acute severe asthma can be successfully treated with aggressive use of β-agonists, anticholinergics, and glucocorticoids. However, prompt intubation may be necessary if these interventions are inadequate and the patient is in obvious respiratory distress. Signs of respiratory distress in asthmatics may include extreme fatigue, depressed mental status, cyanosis, accessory muscle use, chest wall retractions, brief, fragmented speech, inability to lie supine, profound diaphoresis, agitations, and rapid, shallow breathing. Ketamine should be used as in induction agent for intubation in patients with severe asthma. It is a dissociative anesthetic that improves pulmonary function and is of benefit in severe asthmatics. It acts as a direct smooth muscle dilator, increases circulating catecholamines, which indirectly dilate smooth muscle, inhibits vagal outflow, decreases the production of vasodilatory nitric oxide, and does not cause histamine release. In addition, ketamine is less likely to cause hypotension upon administration, making it a favorable choice for induction in the setting of acute severe asthma. Etomidate (A) is a hemodynamically neutral, sedative hypnotic frequently used for induction. It is not shown to provide bronchodilatory effects and does not provide analgesia, thus requiring an agent such as fentanyl for premedication. Propofol (D) is a non-barbiturate, sedative hypnotic that has similar, but less profound bronchodilatory effects to ketamine. Due to its tendency to cause hypotension during induction, it should be avoided in patients who are hypotensive. In a hemodynamically stable patient, propofol would be an appropriate second choice if ketamine was contraindicated or unavailable. Midazolam (C) is a benzodiazepine that is generally used as an adjunctive agent during or before induction. Due to its mild systemic vasodilatory properties, it is not ideal in the setting of acute severe asthma.

A 52-year-old man with a history of hepatitis B presents with complaints of fever, weakness, and pain in his right thigh and knee. He reports a 25-pound weight loss over the past three months. Physical examination reveals patchy, well demarcated reticular rash and palpable tender lumps over the lower extremities. Foot drop is noted as well. Immunofluorescence testing for antineutrophil cytoplasmic antibodies is negative. Renal biopsy reveals inflammation of the medium-sized arteries as well as polymorphonuclear cells. Which of the following is the most likely diagnosis? A. Eosinophilic granulomatosis with polyangiitis B. Granulomatosis with polyangiitis C. Henoch-Schönlein purpura D. Polyarteritis nodosa

Polyarteritis nodosa is a systemic necrotizing vasculitis that typically affects medium-sized muscular arteries. Although most cases are idiopathic, it is closely associated with Hepatitis B and C. Patients typically present with systemic symptoms of fatigue, weight loss, weakness, fever, arthralgias, and signs of multisystem involvement (skin lesions, hypertension, renal insufficiency, neurologic dysfunction, abdominal pain). Skin manifestations include tender erythematous nodules, purpura, livedo reticularis, ulcers, and bullous or vesicular eruption. Kidneys are the most commonly involved organ leading to hypertension and renal failure. Neurologic disease manifests as mononeuropathy multiplex (wrist drop, foot drop) and is one of the most common findings in patients with polyarteritis nodosa. Gastrointestinal symptoms may include abdominal pain due to mesenteric ischemia, weight loss, nausea, vomiting, and gastrointestinal bleeding. Muscle involvement is common and manifests as myalgia and muscle weakness. Serum creatinine kinase levels may be elevated. A clinical diagnosis is suspected based on the presence of characteristic symptoms, physical exam findings, and compatible laboratory test results. A positive immunofluorescence test for antineutrophil cytoplasmic antibodies (ANCA) strongly argues against polyarteritis nodosa. Erythrocyte sedimentation rate (ESR) and C reactive protein are usually elevated. Biopsy of a clinically affected organ is required to confirm the diagnosis. An alternative to a biopsy for diagnosing is mesenteric or renal angiography. These studies demonstrate multiple aneurysms and irregular constrictions (beading) in the larger vessels with occlusion of the smaller penetrating arteries. Glucocorticoids are the treatment of choice. Other immunosuppressive medications may need to be added, depending on disease severity. Eosinophilic granulomatosis with polyangiitis (A) is an eosinophilic-rich necrotizing vasculitis predominantly affecting small- to medium-sized vessels. Patients often will complain of chronic rhinosinusitis and asthma. There will often be prominent peripheral blood eosinophilia. Antineutrophil cytoplasmic antibodies (p-ANCA) are present in approximately 40-60% of patients. Granulomatosis with polyangiitis (B) is a necrotizing vasculitis predominantly involving small- to medium-sized vessels. It typically produces granulomatous inflammation of the upper and lower respiratory tracts. Patients will also have necrotizing, pauci-immune glomerulonephritis. Antineutrophil cytoplasmic antibodies (c-ANCA) are present in more than 80% of patients. Henoch-Schönlein purpura (C) is a systemic vasculitis characterized by the tissue deposition of IgA1-dominant immune complexes, affecting mostly small vessels. The vasculitis typically affects the skin and gastrointestinal tract, and often causes arthritis.

A 73-year-old man with a history of atrial fibrillation and hypertension presented to the emergency department with a severe headache and change in mental status beginning suddenly two hours prior to arrival. Given patients acute presentation and history of known dysrhythmia, stroke protocol was initiated and an ischemic stroke was confirmed on a non-contrast CT scan of the head. If the patient had an embolic event in his left anterior cerebral artery, which of the following findings would be expected on physical examination? A. Broca aphasia B. Incontinence C. Left-sided upper extremity weakness D. Macular-sparing homonymous hemianopsia

Incontinence is a potential presenting symptom of a cerebrovascular accident occurring in one of the anterior cerebral arteries. The anterior cerebral arteries are responsible for the anterior and medial vascularization of the cerebrum. Based on the cortical homunculus, this would have a greater effect on the lower extremity and genitals. A cerebrovascular accident is most commonly caused by ischemia of the brain, commonly from a thrombus or embolus. A smaller percentage of strokes are a result of hemorrhage within the brain. Cerebrovascular accidents are more common in patients with hypertension, history of cigarette use, and heart disease. Presenting symptoms of strokes are dependent on the area of the brain affected by the ischemia or hemorrhage. Symptoms often associated with cerebrovascular accidents are facial droop, slurred speech, and unilateral weakness. A non contrast CT scan of the head should be performed emergently to determine if the stroke is hemorrhagic or ischemic in nature. Patients with an ischemic stroke presenting within 4.5 hours of symptom onset who do not have any thrombolytic exclusion criteria should receive recombinant tissue plasminogen activator (r-tPA). Broca aphasia (A) is the correct choice for a stroke affecting the middle cerebral artery, most commonly the left middle cerebral artery depending on hemisphere dominance of the brain. This would manifest as an expressive aphasia. Left-sided upper extremity weakness (C) would be seen in a stroke affecting the right middle cerebral artery. Middle cerebral artery strokes are more commonly associated with weakness in the upper extremities based on the cortical homunculus. Macular-sparing homonymous hemianopsia (D) would be the correct choice for a stroke affecting the posterior cerebral arteries. The posterior cerebral artery is responsible for the vasculature of the occipital lobe and visual fields.

A 35-year-old man presents to the emergency department with low-grade fever, flu-like symptoms, petechiae, and a new onset murmur. Which of the following diagnostic tests is most sensitive to diagnose this condition? A. Chest computed tomography B. Electrocardiogram C. Transesophageal echocardiogram D. Transthoracic echocardiogram

Infective endocarditis is an infection of one or more heart valves. Complications include severe valvular insufficiency, heart failure, and myocardial abscesses. If left untreated, infective endocarditis is almost always fatal. Symptoms of subacute endocarditis can be subtle and nonspecific, including low-grade fever, anorexia, weight loss, influenza-like syndromes, and pleuritic pain. Patients may also present with the following classic findings of petechiae, splinter hemorrhages, Osler nodes, Janeway lesions, and Roth spots. Signs of neurologic involvement include embolic stroke with focal neurologic deficits caused by vegetations breaking off from the valves (most common neurologic presentation), intracerebral hemorrhage, and multiple microabscesses. The Duke diagnostic criteria are used to examine both blood cultures and echocardiographic findings to make a definitive diagnosis of infective endocarditis. Transesophageal echocardiogram is the diagnostic test of choice for identifying valvular vegetations in patients who are at risk since it is the most sensitive test. Note that the diagnosis of infective endocarditis cannot be excluded based on negative echocardiogram findings. Antibiotics are the mainstay of treatment and should empirically provide coverage against Streptococci and methicillin-resistant Staphylococcus aureus. Transthoracic echocardiogram (D) may be helpful in pediatric patients, but is not the diagnostic tool of choice. Computed tomography (A) and electrocardiogram (B) do not help in the diagnosis of infective endocarditis.

A 24-year-old woman presents to the Emergency Department with fever and shortness-of-breath for the last 48 hours. Her past medical history is significant for recent IV drug abuse. On physical exam, you auscultate a pansystolic ejection murmur best heard at the left lower sternal border. You also note nontender macular lesions on the palms of her hands and soles of her feet. Laboratory analysis is significant for a white blood cell count of 20,000/mcL, erythrocyte sedimentation rate of 67 mm/hour, and C-reactive protein of 6.5 mg/L. Which imaging modality would be most appropriate to confirm your diagnosis? A. Cardiac catheterization B. Chest X-ray C. CT angiogram of the chest D. Transthoracic echocardiogram

Infective endocarditis is an infection of the endocardium of the heart. In general, this infection occurs at one or more of the valves of the heart. Significant risk factors include advanced age, male gender, injection drug abuse, poor dentition, and prosthetic heart valves. In the setting of injection drug abuse, the tricuspid valve is most commonly affected. Signs and symptoms include fever, malaise, exercise intolerance and a new-onset heart murmur. Janeway lesions (nontender erythematous macular lesions of the palms and soles), Osler nodes (tender lesions on the pads of fingers and toes) and Roth spots (hemorrhagic retinal lesions) may also be found and are highly suggestive of infectious endocarditis. Transthoracic echocardiography is the primary imaging modality indicated for evaluation of infective endocarditis. Transthoracic echocardiography is preferred due to the non-invasive approach combined with high sensitivity and specificity. Many patients may also require transesophageal echocardiography to rule out complications such as abscess, leaflet perforation, and pseudoaneurysm. Due to its invasive approach, however, it is not generally recommended as first-line imaging. Cardiac catheterization (A) is an invasive method of inserting a catheter into the coronary arteries of the heart and is generally used in the evaluation of acute coronary syndrome rather than endocarditis. Chest X-ray (B) will be non-specific in diagnosing infective endocarditis but may be indicated early on in the evaluation to rule out other possible etiologies of the patient's symptoms. CT angiogram of the chest (C) is generally used in the setting of acute pulmonary embolism and would provide limited benefit in diagnosing infective endocarditis.

Which of the following statements is most accurate regarding health maintenance for a patient with chronic pancreatitis? A. Abstinence from alcohol will improve symptoms but will not decrease mortality B. Enteric coated pancreatic enzymes require coadministration with an H2 antagonist C. Long chain triglycerides can provide extra calories in patients with weight loss D. Malabsorption of fat soluble vitamins may occur but is rarely clinically symptomatic

Inflammation of the pancreas which is persistent and causes structural damage and eventual decline in pancreatic function is known as chronic pancreatitis. The cause of chronic pancreatitis may be obstructive or necrotic in nature. In obstructive pancreatitis, ductal stone formation leads to chronic inflammation which causes progressive pancreatic fibrosis and eventual atrophy. In necrotic pancreatitis, recurrent attacks of acute pancreatitis lead to necrotic tissue, which is eventually replaced by fibrotic tissue. Smoking and alcohol consumption are two main risk factors for pancreatitis. Other etiologies of pancreatitis include autoimmune disease, genetic disorders, and tumors. Symptoms of chronic pancreatitis are abdominal pain, nausea, vomiting and weight loss. Steatorrhea develops later in the disease course after the pancreas has lost much of its exocrine function. Diagnosis can be made via visualizations of calcifications on plain film or CT scan, magnetic resonance cholangiopancreatography, endoscopic ultrasound, or pancreatic function tests (such as stimulation of the pancreas and subsequent collection of duodenal fluid for analysis). Treatment of chronic pancreatitis involves lifestyles changes, pain management, and administration of pancreatic enzymes. Cessation of alcohol intake and smoking is imperative. Small, low-fat meals should be advised. Pain can be managed with tricyclic antidepressants, narcotics, or occasional hospitalizations to keep the patient nothing by mouth (NPO) for a short duration and allow pancreatic rest. Pancreatic enzymes are prescribed for patients with chronic pancreatitis to break the feedback loop of cholecystokinin release and pancreatic exocrine stimulation. Medium chain triglycerides are easily absorbed and can provide extra calories for patients suffering weight loss. Malabsorption of vitamins due to pancreatic exocrine malfunction may occur but is usually not clinically symptomatic. Replacement of the fat soluble vitamins and cyanocobalamin may be necessary for patients with substantial steatorrhea. Patients who fail medical therapy may be candidates for surgical procedures, such as denervation, decompression, and resection. Abstinence from alcohol (A) is essential for patients with chronic pancreatitis, although abstaining from alcohol may not improve the patient's symptoms. Studies have shown that patients with chronic pancreatitis who continue to consume alcohol have higher mortality rates. Smoking cessation is also essential, as smoking increases the risk of pancreatic cancer and may accelerate the progression of chronic pancreatitis. Several formulations of pancreatic enzyme supplements exist, and many of them are enteric coated (B). This helps to prevent inactivation of the enzymes by gastric acid. If non-enteric coated formulations are prescribed, the patient will also need to take an H2 antagonist or a proton pump inhibitor to suppress gastric acid and reduce inactivation of the enzymes. Medium chain triglycerides are prescribed for patients with chronic pancreatitis and weight loss, because they do not require bile acid to be absorbed. Thus, they can provide extra calories for cachectic patients. Long chain triglycerides (C) require bile acid to be degraded, and cannot be directly absorbed by the intestinal mucosa. Medium chain triglycerides can be directly absorbed from the intestinal mucosa and therefore cause less stimulation of pancreatic secretion.

A 24-year-old woman presents to the clinic with sudden onset of fever, chills, dry cough, malaise, substernal soreness, coryza, nasal congestion, and myalgias. Her symptoms came on suddenly yesterday while she was at work. Her husband was recently sick with similar symptoms. Physical exam reveals a temperature of 38.6°C, pharyngeal injection, conjunctival injection, and facial flushing. A chest X-ray is obtained and is negative for any acute cardiopulmonary process. What is the most likely diagnosis? A. Adenovirus B. Influenza C. Legionella D. Pertussis

Influenza is an orthomyxovirus that affects patients in the fall and winter months in temperate climates. It can be divided into three types: type A, type B, and type C. Types B and C exclusively infect humans. Influenza is characterized by an abrupt onset of fever, chills, malaise, coryza, and cough. Patients sometimes complain of substernal soreness, sore throat, and conjunctival injection. The incubation period for illness is one to four days with symptoms lasting between one to seven days after onset. Complications can be severe and include viral pneumonitis, neurologic complications, seizure, and respiratory failure. Influenza can cause necrosis of the respiratory epithelium leading to secondary bacterial infections. Some groups are at greater risk for complications. These include patients with asthma, lung disease, immunosuppression, neurologic disorders, and pregnant women. Infection during pregnancy is an important risk factor for spontaneous abortion in preterm labor. There is also higher mortality associated with male sex, underlying liver disease, and existing cancer. The diagnosis can be confirmed with a nasopharyngeal swab that is sent for rapid immunofluorescence assay. PCR techniques are becoming more readily available and can also detect influenza antigen. A complete blood count (CBC) commonly shows leukopenia but may demonstrate leukocytosis. Antiviral treatment within 48 hours of symptom onset has been shown to reduce the duration of illness significantly. Outpatient treatment includes oral antiviral medications such as oseltamivir. Yearly vaccination with the influenza virus before influenza season remains an important preventative strategy against the spread of infection. Adenovirus (A) typically causes a sore throat, fever, coryza, pharyngeal edema, and pharyngeal exudate on day two to three of the illness. Although the symptoms are similar to influenza, patients with adenovirus typically do not complain of myalgias and substernal soreness. Legionella (C) typically occurs in elderly patients and in addition to respiratory symptoms is associated with diarrhea. Pertussis (D), or whooping cough, is typically associated with the characteristic paroxysmal cough lasting several weeks.

A 17-year-old woman with a history of severe menorrhagia was admitted for a blood transfusion after fainting at school. Her history is positive for fatigue, exertional dyspnea, and increased cravings for ice over the past several weeks. Laboratory findings show hemoglobin 6 g/dL, hematocrit 22%, mean corpuscular volume 72 fL, total serum iron 23 mcg/dL, and total iron binding capacity 92 micromol/L. The patient was found to have conjunctival pallor and koilonychia on physical examination. What additional physical exam finding would you expect to find? A. Facial plethora B. Jaundice C. Loss of tongue papillae D. Petechiae

Loss of tongue papillae or atrophic glossitis is seen in patients with iron deficiency anemia. Iron deficiency anemia is common in women of reproductive age and children. Causes of iron deficiency anemia include blood loss, malabsorption of iron, a decrease in dietary intake of iron, and pregnancy. Symptoms include weakness, fatigue, exercise intolerance, exertional dyspnea, and pica or ice cravings. On physical examination patients are found to have pallor, atrophic glossitis, angular cheilitis, and koilonychia (spoon nails). Laboratory findings include decreased hemoglobin and hematocrit, decreased mean corpuscular volume, decreased absolute reticulocyte count, decreased total serum iron, increased serum transferrin, increased total iron binding capacity, and decreased serum ferritin. Treatment includes addressing the cause of the iron deficiency anemia and replenishing iron with oral iron supplements (IV supplements can be used in patients with severe blood loss or impaired absorption of oral iron). Transfusion may be indicated in patients with blood loss causing hemoglobin <10 g/dL in certain populations. Facial plethora (A) is incorrect as it is another term for ruddy cyanosis, which is associated with polycythemia vera. Jaundice (B) is incorrect as it is more commonly seen in hemolytic anemia, such as thalassemia, sickle cell anemia, G6PD deficiency, and hereditary spherocytosis. Petechiae (D) is incorrect as it is a physical examination finding commonly associated with platelet dysfunction or thrombocytopenia.

Which of the following conditions is most commonly associated with malar or "butterfly" rash? A. Parvovirus B19 infection B. Pregnancy C. Rosacea D. Systemic lupus erythematosus

Malar rash is a common skin finding in systemic lupus erythematosus (SLE). It is characterized by erythema involving the cheeks and bridge of the nose, but sparing the nasolabial folds. This characteristic distribution is also referred to as a "butterfly rash." This rash may precede systemic manifestations of systemic lupus erythematosus by months to years, or it may occur concomitantly. A malar rash will often worsen with sun exposure. Because of its strong association, the presence of a malar rash should prompt the clinician to evaluate the patient for a possible diagnosis of systemic lupus erythematosus. Additional clinical manifestations of systemic lupus erythematosus may include fatigue, discoid lesions, painless oral or nasal ulcers, hair loss, Raynaud phenomenon, migratory or symmetrical joint pain and swelling, and symptoms of pericarditis. Systemic lupus erythematosus is most commonly seen in African-American patients of childbearing age. Antinuclear antibodies (ANA) is the best initial screening test (most sensitive but not specific). The presence of either anti-double-stranded DNA (dsDNA) and anti-smith (anti-SM) antibodies is diagnostic of SLE (very specific but not sensitive). Treatment of SLE depends on severity and symptoms of the disease. NSAIDs are used for less severe symptoms. Corticosteroids are used for acute exacerbations. Hydroxychloroquine is the best long-term therapy. It reduces systemic lupus erythematosus symptoms, reduces flares, and progression of the disease. Annual eye examination by an ophthalmologist is necessary due to retinal toxicity. Cyclophosphamide is used for active lupus nephritis. Parvovirus B19 infection (A) commonly causes erythema infectiosum, which is a mild febrile illness in children. Though a malar, or "slapped cheek" appearance is characteristic, the presence of fever is not suggestive of systemic lupus erythematosus. Melasma is a rash commonly found during pregnancy (B), and may have a malar pattern. This rash is characterized by darkening of the skin with increased keratinocytes. Rosacea (C) is a chronic erythematous skin condition, often involving the central face. It generally involves the nasolabial folds and has papules and pustules, which is not characteristic of systemic lupus erythematosus.

A patient with a history of radiation therapy for a neck neoplasm presents for evaluation of difficulty swallowing solid foods, but not liquids. Upper gastrointestinal radiography reveals a proximal esophageal stricture. Which of the following is the most appropriate therapy? A. Initiate H-2 antagonist therapy B. Initiate proton pump inhibitor therapy C. Refer for surgical resection D. Schedule mechanical dilation

Mechanical dilation is the primary treatment for esophageal strictures. Strictures in the proximal esophagus are generally related to squamous cell carcinoma, radiation therapy, infectious complications from human immunodeficiency virus, medication complications (pill esophagitis), or a manifestation of diseases of the skin. Distal esophageal strictures are usually related to reflux, collagen vascular disease, nasogastric tube complications, or adenocarcinoma. The most common symptom is dysphagia, often progressive. A history of heartburn or indigestion may also be reported. Physical examination is usually nonspecific but may implicate other disease processes. Diagnosis is usually made by barium esophagram, however, other studies, such as computed tomography or esophageal pH monitoring, may be required if malignancy or uncontrolled reflux are suspected. Esophagogastroduodenoscopy (EGD) may be used as both a diagnostic and treatment modality and can facilitate mechanical dilation with a bougie (wire-guided or weighted) or balloon. Additional treatments that can be performed during EGD include intralesional steroid injection, stricturoplasty, and stenting. Mechanical dilation risks include perforation, pain, and the need for redilation. Medical treatment may also be important, depending on the etiology of the stricture, as some evidence suggests that use of proton pump inhibitors can reduce redilation rates. Other measures that may be helpful include avoidance of tobacco, caffeine, spicy foods, peppermint, and large meals. Complications from untreated esophageal stricture include progressive dysphagia, obstruction of the esophagus (in severe cases), vomiting, choking, aspiration, dehydration, and malnutrition, in addition to poor quality of life. H-2 antagonist therapy (A) is not indicated as a primary treatment method for esophageal stricture and has shown minimal benefit in reducing recurrence. Proton pump inhibitor therapy (B) may be useful in preventing recurrence of strictures related to esophagitis but does not address current lesions. It is common to require high doses to achieve reductions in redilation. Surgical resection (C) is a potential option for certain patients with a distal esophageal stricture but is not usually a treatment option for proximal strictures.

In the treatment of Alzheimer disease, which of the following pharmacologic treatment options is found to be neuroprotective? A. Donepezil B. Memantine C. Selegiline D. Vitamin E

Memantine is classified as an N-methyl-D-aspartate (NMDA) receptor antagonist. Studies have found excessive NMDA stimulation can cause ischemia in the cortex and hippocampus. By inhibiting NMDA, memantine works to prevent this ischemia and is found to be neuroprotective in dementia treatment. Memantine is found to be most beneficial when used in conjunction with cholinesterase inhibitors to treat moderate-to-severe Alzheimer disease. The most common side effects of memantine include dizziness, confusion, and hallucinations. Donepezil (A) is a cholinesterase inhibitor commonly used in the treatment of Alzheimer disease. Cholinesterase inhibitors, including rivastigmine and galantamine, work to improve cholinergic function by inhibiting the cholinesterase enzyme. Cholinesterase inhibitors are not neuroprotective. Selegiline (C) is a monoamine oxidase inhibitor that has been studied in the treatment of Alzheimer disease but has limited benefit in the setting of many side effects. Vitamin E (D) has been found to be beneficial in delaying the progression of functional impairments in Alzheimer disease but has no effect on cognitive performance nor neuroprotection.

Which of the following occurs with multiple sclerosis? A. Abnormality in the genetic code for specific muscle proteins B. Axonal degeneration in the peripheral nervous system C. Blocking of receptors at the neuromuscular junction of skeletal muscles D. Destruction of myelinated axons in the central nervous system

Multiple sclerosis (MS) is an immune-mediated disorder in which the immune system responds abnormally, attacking and destroying myelinated axons in the central nervous system. Individuals with MS have symptomatic episodes that can occur months or years apart with different locations in the body being affected. The cause is unknown. MS more commonly affects women and while it can occur at any age, the average age of symptom onset is approximately 30 years. Clinical manifestations include paresthesias, muscle cramping or spasms, bowel or bladder dysfunction, ataxia, tremor, cognitive changes, facial weakness, and facial muscle twitching. Diagnosis is based on clinical manifestations and magnetic resonance imaging of the brain, which confirms the diagnosis. Treatment involves two components: immunomodulatory therapy to address the immune disorder and symptomatic therapy. Regardless of treatment, patients with MS generally end up with significant physical disability after 20-25 years of having the disease. Muscular dystrophy is a group of progressive muscle disorders without any abnormality in the central or peripheral nervous system. The cause of muscular dystrophy is an abnormality in the genetic code for specific muscle proteins (A) which causes muscle weakness. Symptoms become apparent in childhood when the child begins walking. While treatment has improved, muscular dystrophy cannot be cured and individuals with the disease generally die by the age of 30 years. Guillain-Barré syndrome is a post-infectious disease mediated by the immune system which results in axonal degeneration in the peripheral nervous system (B). This causes the ascending paresthesias and weakness that are the hallmark of the disease. Myasthenia gravis is an autoimmune disorder that causes abnormalities in neuromuscular transmission. Antibodies form that block receptors at the neuromuscular junction of skeletal muscles (C), causing the classic weakness presentation starting in the eye muscles. It then involves the facial muscles, followed by the extremities, with respiratory muscles being the last affected. Patients need to be monitored closely for respiratory crisis and may need admission to the intensive care unit along with intubation.

A 38-year-old, previously healthy woman presents with pedal edema, dyspnea, fever and fatigue. Electrocardiogram reveals non-specific ST-T wave changes. Cardiac troponin and CK-MB are elevated. Echocardiogram shows segmental wall motion defects. Cardiac catheterization is negative for perfusion defects and the chest radiograph shows bilateral pleural effusions. Which of the following is the most likely diagnosis? A. Endocarditis B. Myocardial infarction C. Myocarditis D. Pneumonia

Myocarditis is an inflammation of the myocardium leading to cardiac cell necrosis. This inflammation can be caused by infection, autoimmune processes, cardiotoxic drugs, systemic inflammatory disorders, radiation therapy, and can sometimes be idiopathic. Signs and symptoms of myocarditis vary greatly and are dependent on the severity and etiology of the disease. Patients may be asymptomatic, may suffer signs of heart failure, or may present with sudden cardiac death. Diagnosis of myocarditis can be difficult as it can mimic other disease states. Electrocardiogram findings in myocarditis are often non-specific but can mimic myocardial infarction, with ST elevations or nonspecific ST-T wave changes. Echocardiography may be normal or may reveal segmental wall motion defects, systolic or diastolic dysfunction, or left ventricular dilatation. Cardiac enzymes such as cardiac troponin and CK-MB are often elevated due to myocardial injury. Cardiac catheterization and cardiac MRI can help to differentiate myocardial infarction from myocarditis. An endomyocardial biopsy can give a definitive diagnosis, but is not often employed due to its low sensitivity and high risk of complications. Treatment of myocarditis depends on the etiology of the myocardial inflammation. Infectious myocarditis in the United States is often of viral etiology and is therefore treated with symptomatic care and medical management of any associated dysrhythmias or heart failure. Myocarditis due to drug toxicity is treated by withdrawing the offending agent and administering corticosteroids. Steroids are also used to treat myocarditis caused by sarcoidosis or giant-cell myocarditis. Endocarditis (A) involves an infection of the endocardium and is usually bacterial in origin. Symptoms and signs can range from fever and malaise to embolic events, valvular insufficiency, stroke or sudden death. Predisposing factors are intravenous drug abuse, bacteremia, or an abnormality of the endocardium. Diagnosis can be made using the Duke clinical diagnostic criteria, blood cultures and echocardiography. Endocarditis may demonstrate paravalvular abscesses or mycotic aneurysms on echocardiogram. Treatment is with intravenous antibiotics. Myocardial infarction (B) is myocardial cell death as a result of cardiac ischemia. The patient in the above vignette demonstrated some signs of myocardial infarction, such as elevated cardiac enzymes and ST-T segment changes on electrocardiogram, but on cardiac catheterization no perfusion defects were noted. A patient who has suffered an acute myocardial infarction typically has perfusion deficits that are often treated with either stenting or coronary artery bypass grafting. Another modality which can help differentiate myocardial infarction from myocarditis is a cardiac MRI, where myocarditis and myocardial infarction have different, distinct patterns of late gadolinium enhancement. Pneumonia (D) may be suspected in a patient with fever, fatigue and dyspnea but will not often cause pedal edema. Also, pneumonia would not be expected to increase levels of cardiac enzymes or result in cardiac wall motion defects on echocardiography. Pneumonia may cause pleural effusions, but is more likely to demonstrate infiltrates on chest radiograph.

When evaluating a patient with chest pain, you note ST-segment elevation in leads II, III and aVF. Which of the following is the most likely diagnosis? A. Anterior wall myocardial ischemia B. Inferior wall myocardial ischemia C. Lateral wall myocardial ischemia D. Posterior wall myocardial ischemia

New abnormalities in the ST-segment and T waves on ECG in a patient with chest pain represents an acute myocardial ischemia or infarction until proven otherwise. Determining the location of infarct is crucial for proper management of these patients. Right ventricular ischemia or infarction will present with inferior ECG changes (ST-segment shifts or Q waves in leads II, III, and aVF). If there is evidence of inferior wall ischemia, right-sided leads, especially V3R and V4R, should be obtained to assess for a possible right ventricular ischemia/infarction. Patients with right ventricular myocardial infarction may present with hypotension, elevated jugular venous pressure, hepatomegaly and clear lungs. These patients are preload dependent and should not be treated with medications that lower preload (nitrates or diuretics) as it will cause cardiovascular collapse. Intravenous fluids should be administered to patients with hypotension without evidence of pulmonary congestion or right heart failure. Anterior wall myocardial ischemia (A) is generally represented by ST-T wave elevation or Q waves in leads V1-V4. Lateral wall myocardial ischemia (C) is generally represented by ST-T wave changes or Q waves in leads I and aVL. Posterior wall myocardial ischemia (D) will show large R wave in V1 and V2, ST-segment depression in V1 and V2, upright and prominent T waves in V1 and V2.

Which of the following diagnostic studies is the most appropriate for a 33-year-old healthy man with a history of episodic tension-type headache in the setting of a normal neurological examination? A. CT scan B. MRI with contrast C. MRI without contrast D. No diagnostic study is necessary

No diagnostic imaging study is necessary for a healthy patient presenting with stable tension-type headaches in the absence of any neurological findings. Tension headache is a clinical diagnosis, based on the International Classification of Headache Disorders diagnostic criteria. Tension headaches are non-pulsating, non-localized headaches that are located bilaterally on the cranium. These headaches are not severe in nature and are not aggravated by everyday physical activity. There is no nausea, vomiting, photophobia, or phonophobia associated with the headache. An individual must have at least 10 episodes of the headache lasting for at least 30 minutes for diagnosis. CT scan and MRI are not necessary in patients presenting with tension-type headache symptoms unless they have abnormal neurological findings or the headache is atypical and not fulfilling tension headache criteria. Headaches from a brain tumor can often mimic these types of headaches. Over-the-counter analgesics, such as aspirin, acetaminophen, and ibuprofen, are most commonly used to treat tension headaches in addition to non-pharmacologic treatments, such as ice and rest. Tricyclic antidepressants are commonly used as a preventative measure for tension headaches. CT scan (A) is incorrect, as neuroimaging is not recommended in the diagnosis of tension headaches. CT scan would be used in an emergency setting to rule out intracranial injury or hemorrhage as a cause of the headache. MRI with contrast (B) is incorrect. An MRI with and without contrast would be used in an individual whose neurological examination revealed abnormalities requiring further workup for headache cause. MRI without contrast (C) is incorrect. As stated previously, MRI with and without contrast is the correct neuroimaging choice for an individual whose neurological examination revealed an abnormality or whose history was concerning for an underlying cause of headache symptoms.

A 35-year-old man presents to the office with acute onset right flank pain radiating to the groin with intermittent nausea and vomiting. A urinalysis and computed tomography of the abdomen and pelvis without contrast were ordered. An urgent referral for outpatient urologist evaluation would be most appropriate for which of the following findings? A. A 3 mm stone in the distal ureter B. A 9 mm mid-ureteral stone C. Hydronephrosis with urinary tract infection D. Ureteral obstruction from a stone in a transplanted kidney

Pain associated with nephrolithiasis is caused by dilatation, stretching, and spasm due to acute ureteral obstruction. Patients with acute renal colic classically present with sudden and severe radiating flank pain and may have nausea and vomiting. Workup includes urinary dipstick analysis to test for bacteriuria and microscopic hematuria, serum creatinine level to assess renal function, CBC, and serum electrolytes. Noncontrast abdominopelvic computed tomography scan is the study of choice is most adult patients, although renal ultrasonography can also be used. Medical treatment targets both symptomatic relief and expulsion of the stone. Opioids and nonsteroidal anti-inflammatory drugs are used to reduce pain and inflammation, whereas antiemetics are useful for treating nausea. Intravenous fluids are essential in treating dehydration in the vomiting patient. Calcium channel blockers and alpha blockers have been shown to facilitate stone passage by causing smooth muscle relaxation. Stones that are < 4 mm in diameter are likely to pass on their own with minimal medical management. Stones that are > 8 mm in diameter, however, are unlikely to pass spontaneously and may require stent placement, percutaneous nephrostomy, extracorporeal shockwave lithotripsy, ureteroscopy, and nephrostolithotomy. Therefore, an urgent referral for outpatient urologist evaluation is recommended for a patient with a 9 mm mid-ureteral stone who is likely to need further management. A 3 mm stone in the distal ureter (A) will have a high likelihood of passing on its own with outpatient medical treatment and close follow up by a primary care provider. Patients with hydronephrosis with urinary tract infection (C) or ureteral obstruction from a stone in a transplanted kidney (D) should be referred to the emergency department immediately for treatment and should not wait to be referred to an outpatient urologist.

A 60-year-old man with intermittent claudication returns to the clinic for interpretation of the results of his ankle-brachial index. Which of the following statements is most accurate? A. A falsely high index may indicate severely hardened, non-compressible leg vessels B. A low ankle-brachial index indicates peripheral venous insufficiency C. An ankle-brachial index of 1.2 indicates severe disease D. It is the ratio of ankle to arm diastolic blood pressure

Peripheral arterial disease is the result of accumulation of plaque in the arteries of the lower extremities, causing decreased blood flow to the tissues. Patients with peripheral arterial disease may be asymptomatic or may suffer claudication (i.e., pain on ambulation which is alleviated by rest), loss of hair in the lower extremities, atrophic skin, ischemic ulcers, or gangrene. Risk factors for peripheral arterial disease include dyslipidemia, hypertension, diabetes, cigarette smoking, male sex, family history of atherosclerosis, obesity, and advanced age. Diagnosis of peripheral arterial disease is made using the ankle-brachial index, doppler ultrasonography, or angiography (if surgery is required). The ankle-brachial index is calculated by measuring the systolic blood pressure in the arm and at the ankle. A ratio of ankle systolic blood pressure to brachial systolic blood pressure under 0.90 is indicative of peripheral arterial disease. A falsely high ankle-brachial index can occur when a patient has severely hardened peripheral arteries which are non-compressible. If the ankle-brachial index is over 1.3 but there remains high clinical suspicion of disease, doppler ultrasound is performed to assess for arterial stenosis or occlusions. Treatment of peripheral arterial disease consists of addressing the underlying disease processes, as well as administering antiplatelet drugs. In severe cases, stent placement or surgery may be warranted. A low ankle-brachial index indicates peripheral venous insufficiency (B) is incorrect. A low index indicates peripheral arterial insufficiency. Venous insufficiency can present with lower extremity edema, varicosities, increased pigmentation, and venous stasis ulcers. A blood pressure reading of the lower extremity will not provide clinically useful information regarding the veins of the lower leg. Doppler ultrasound may be used to diagnose venous insufficiency, although physical exam is often sufficient. Venous insufficiency is treated using compression stockings and elevation of the limb. An ankle-brachial index of 1.2 indicates severe disease (C) is incorrect. A normal range for the ankle-brachial index is 0.91-1.3. If the ankle-brachial index is 0.71-0.90, the patient is considered to have mild peripheral arterial disease. An ankle-brachial index of 0.41-0.70 is considered moderate, and an ankle-brachial index at or below 0.40 indicates severe disease. If the results of the ankle-brachial index are above 1.3 and the patient has clinical signs of peripheral arterial insufficiency, a doppler should be performed to rule out arterial stenosis or occlusion. It is the ratio of ankle to arm diastolic blood pressure (D) is incorrect. The ankle-brachial index is the ratio of ankle systolic blood pressure to arm systolic blood pressure.

A 60-year-old man with insulin-dependent diabetes mellitus, gastroparesis, hypertension, and congestive heart failure has just received a new diagnosis of pheochromocytoma. Which of the following medications should be discontinued? A. Furosemide B. Insulin lispro C. Lisinopril D. Metoclopramide

Pheochromocytoma is a rare neuroendocrine neoplasm which secretes catecholamines and can cause hypertension, headache, sweating, tachycardia and can lead to hypertensive crises. Most cases of pheochromocytoma are discovered incidentally on abdominal CT scan or MRI. The discovery of an adrenal mass should lead to further testing, including plasma and urinary measurements of catecholamines and fractionated metanephrines. Once pheochromocytoma is diagnosed, surgical excision is the next step in treatment. To prepare for surgical excision, medications which cause stimulation of pheochromocytoma activity must be discontinued. These medications include beta-blockers (in the absence of alpha-blocking agents), glucagon, metoclopramide, and histamine. Beta-blockers are routinely administered to patients with pheochromocytoma, but only after they have been established on treatment with alpha-adrenergic blocking agents such as phenoxybenzamine. If beta-blockers are administered prior to alpha-adrenergic blockade, the result can be further elevation in blood pressure due to unopposed alpha-adrenergic action in the peripheral blood vessels. Surgical excision of a pheochromocytoma is a high-risk procedure and must be performed by a skilled surgical team. Proper preoperative medical preparation to ensure volume expansion and blood pressure control is essential. Furosemide (A) is not known to stimulate pheochromocytoma activity, although volume expansion is often necessary in preparation for surgical excision of a pheochromocytoma, and this may necessitate changes in the dosage of diuretics. The volume expansion of a patient with congestive heart failure would have to be carefully managed so as to avoid further congestion. Insulin lispro (B) and all other insulin derivatives are not known to stimulate pheochromocytoma activity. In fact, insulin must not be removed from the above patient's treatment plan, because he is insulin-dependent. Glucagon is known to increase pheochromocytoma activity, so hypoglycemic states should be avoided in patients with the disease. Lisinopril (C) is not an inciter of pheochromocytoma activity and may be used in patients with the neoplasm. Blood pressure control prior to surgery and avoidance of hypertensive crisis is very important and will often involve several blood pressure-lowering agents.

A 55-year-old woman with a history of HIV presents to the clinic complaining of a productive cough and dyspnea for two weeks. Her oxygen saturation is 92%, temperature is 101.5°F, pulse is 95/minute, respirations are 17/minute, and blood pressure is 130/85. She has a 10-year history of rheumatoid arthritis and is currently being treated with infliximab. Chest X-ray shows diffuse, bilateral, interstitial infiltrates. The diagnosis is confirmed by examination of respiratory secretions. Which of the following is the most appropriate therapy? A. Azithromycin B. Levofloxacin C. Prednisone D. Trimethoprim-sulfamethoxazole

Pneumocystis pneumonia (PCP) is a form of pneumonia caused by the yeast-like fungus Pneumocystis jirovecii. HIV-infected patients with a CD4 count < 200 are at the highest risk of PCP. Clinical manifestations include dyspnea on exertion, dry cough, and fever. Laboratory findings may reveal elevated lactate dehydrogenase (LDH). Chest X-ray features of PCP are bilateral, diffuse, interstitial infiltrates. Trimethoprim-sulfamethoxazole is the recommended first-line treatment for PCP in an HIV-infected patient of any severity. For patients with allergy to trimethoprim-sulfamethoxazole, desensitization should ideally be performed since TMP-SMX is the most effective regimen. If the patient has a history of severe reaction, alternatives should be considered. Clindamycin with primaquine is an alternative in mild, moderate, and severe disease. IV pentamidine can also be used in severe disease. Azithromycin (A) is the first-line treatment option for patients with community-acquired pneumonia that do not require hospitalization and live in areas with low macrolide resistance. It is not recommended for fungus related infections. Levofloxacin (B) is the first-line treatment option for patients with community-acquired pneumonia that live in areas with high macrolide resistance, older adults, patients with comorbidities, and those who were recently treated with antibiotics. It is also not recommended for fungus related infections. Adjunctive prednisone (C) is recommended in HIV-uninfected patients who, while breathing room air, have an arterial blood gas measurement that shows a partial pressure of oxygen ≤ 70 mmHg or an alveolar-arterial (A-a) oxygen gradient ≥ 35 mmHg or hypoxemia on pulse oximetry.

A 25-year-old woman presents with intermittent flank pain, hypertension, and blood in her urine. Renal ultrasound shows polycystic kidneys bilaterally. Which of the following complications will this patient most likely develop? A. End stage renal disease B. Intracranial aneurysms C. Portal hypertension D. Renal carcinoma

Polycystic kidney disease is a progressive autosomal dominant disease. It is characterized by cyst formation and kidney enlargement and can affect other organs (e.g. liver, pancreas, spleen) as well. Abdominal, flank or back pain is the most common initial complaint and is seen in almost all patients. The pain can be caused by enlargement of one or more cysts, bleeding within or related to the cyst which can lead to hematuria, and acute urinary tract infection. One of the earliest signs of polycystic kidney disease is hypertension (usually with elevated diastolic blood pressure), which may worsen as renal insufficiency progresses. Other physical exam signs include palpable, bilateral flank masses, nodular hepatomegaly, and symptoms related to renal failure in end stage renal disease. Genetic testing is sometimes used, but imaging is a much more widely used diagnostic test. Ultrasonography is the technique of choice and can be used for screening other family members. Management of disease includes blood pressure control (with angiotensin-converting-enzyme inhibitors and angiotensin receptor blockers), maintaining electrolyte levels due to renal insufficiency and failure, treating urinary tract infections, treating cyst infections, and pain management. More invasive treatments include surgical drainage of infected cysts that do not respond to antibiotics, nephrectomy as a last resort for those who have uncontrolled pain, partial hepatectomy, and liver transplantation. Up to half of patients with polycystic kidney disease develop end stage renal disease by age 60 and require renal replacement therapy which may include hemodialysis, peritoneal dialysis, or renal transplantation. Intracranial aneurysm (B) with vessel rupture is a dangerous complication of polycystic kidney disease, however, it is not the most common complication. Portal hypertension (C) is associated with polycystic kidney disease due to extensive hepatic cysts causing distortion of the portal vein, but this is not as common as end stage renal disease. Renal carcinoma (D) is a differential diagnosis that needs to be ruled out in patients with renal cysts but is not directly associated with polycystic kidney disease.

A 64-year-old man presents to the clinic with generalized pain in his neck, shoulders, and hips. He states the pain is worse in the mornings and is sometimes associated with one-sided headaches. His erythrocyte sedimentation rate and C-reactive protein are both elevated. You diagnose him with polymyalgia rheumatica. What other condition would you suspect in this patient? A. Fibromyalgia B. Giant cell arteritis C. Systemic lupus erythematosus D. Trigeminal neuralgia

Polymyalgia rheumatica is a systemic, inflammatory rheumatic arthritis that is characterized by morning stiffness and muscle aches in the shoulders, neck, and hips. Patients may present with functional limitations, including difficulty putting on a coat or shirt, getting out of a chair, combing hair, donning socks and shoes. Systemic manifestations of malaise, fatigue, depression, anorexia, weight-loss, and low-grade fever may be present as well. The incidence increases with age, and it is most commonly found in elderly patients > 50 years of age. Women and individuals of Scandinavian and northern European descent are most commonly affected. Polymyalgia rheumatica is a clinical diagnosis. Elevated erythrocyte sedimentation rate (ESR) and C-reactive protein (CRP) aid in diagnosing. Polymyalgia rheumatica is associated with an increased incidence of giant cell arteritis. New-onset headache, monocular visual impairment, or jaw pain with mastication in patients with a diagnosis of polymyalgia rheumatica should be concerning for giant cell arteritis. Diagnosis of giant cell arteritis is confirmed by temporal artery biopsy. Treatment for both conditions is glucocorticoid therapy. Fibromyalgia (A) is characterized by widespread musculoskeletal pain, extreme fatigue, headaches, and insomnia. Erythrocyte sedimentation rate and C-reactive protein are usually normal in these patients. Systemic lupus erythematosus (C) is an autoimmune disorder which can commonly cause muscle pain and myalgias, however, it is not generally associated with polymyalgia rheumatica. Trigeminal neuralgia (D) commonly presents with one-sided headache and facial pain, however, erythrocyte sedimentation rate and C-reactive protein are generally not elevated in this condition.

A 44-year-old man presents for a follow-up visit for his known gastroesophageal reflux disease. He has been taking a histamine-2-receptor antagonist twice a day. He continues to have symptoms at least three times per week. Which of the following is the next best therapy? A. Calcium carbonate B. Famotidine C. Omeprazole D. Sucralfate

Proton pump inhibitors (PPIs), such as omeprazole, should be used in patients who fail twice-daily histamine-2-receptor antagonist therapy, in patients with erosive esophagitis, or patients with severe and frequent symptoms of gastroesophageal reflux disease (GERD). PPIs inhibit gastric acid secretions by irreversibly binding and inhibiting the hydrogen-potassium ATPase pump. They should be taken 30 minutes before the first meal of the day. Proton pump inhibitors, compared with histamine-2-receptor antagonists, are more effective in relieving symptoms of GERD. Long-term use of proton pump inhibitors has been shown to increase the risk of infections, iron and vitamin B12 deficiency, hypomagnesemia, hip fracture and calcium malabsorption, atrophic gastritis, and kidney disease. Because of this, they should be used at the lowest possible dose and for the shortest duration needed. Calcium carbonate (A), an antacid, can be used as an initial therapy for GERD symptoms along with lifestyle and dietary modifications. The role of calcium carbonate is limited to on-demand use for relief of mild GERD symptoms that occur less than once per week. The mechanism of action is to neutralize gastric acid. Antacids begin to provide relief of symptoms within five minutes but have a short duration of effect of 30 to 60 minutes. Famotidine (B) is another histamine-2-receptor antagonist and can be used for mild and intermittent symptoms of GERD. It decreases the secretion of acid by inhibiting the histamine-2-receptor on the gastric parietal cell. They have a slower onset of action than antacids but significantly longer duration of action of up to four to ten hours. Sucralfate (D), a surface agent, adheres to the mucosal surface of the stomach, promotes healing, and protects from peptic injury by mechanisms not completely understood. Given the short duration of action and limited efficacy as compared with proton pump inhibitors, the use of sucralfate is limited to the management of GERD in pregnancy.

A 25-year-old man presents to the clinic with fever and a sore throat. A throat swab is performed and the culture comes back positive for group A streptococcus bacteria. What system of his body is at highest risk for permanent damage if proper treatment for his infection is not administered? A. Cardiovascular B. Gastrointestinal C. Musculoskeletal D. Pulmonary

Rheumatic fever is an inflammatory disease of the heart, typically pancarditis, that can occur with inadequate treatment of strep throat or scarlet fever. In developed countries, like the United States, scarlet fever is rare but group A Streptococcus infection is common. The onset of rheumatic fever typically begins two to four weeks after a group A Streptococcus infection and can occur at any age, although most cases occur in children between 5 and 15 years of age. Antibodies can start to attack various tissues of the body, most concerning of which are the valves of the heart which can lead to permanent damage 10-20 years after the original illness. Surgery may be required to repair or replace the damaged valves. Appropriate antibiotics are required and nonsteroidal anti-inflammatories for supportive treatment are the standard of care. Gastrointestinal (B) symptoms include stomach pain and loss of appetite, but no underlying pathologic changes are known to occur in rheumatic fever. Symptoms typically resolve with resolution of the underlying infection. The musculoskeletal (C) system is the second most commonly affected by rheumatic fever, hence its name, with swelling and tenderness in joints. Most commonly affected joints are wrists, elbows, knees, and ankles, with possible nodules near elbows and knees. Joint complaints typically resolve with nonsteroidal anti-inflammatory treatment. Pulmonary (D) symptoms include shortness of breath, but no underlying pathologic changes occur in rheumatic fever. Symptoms such as fever and fatigue typically resolve with resolution of the underlying infection.

A 56-year-old woman presents to the office with complaints of double vision, drooping eyelids, and speech that becomes slurred late in the day. Patient reports symptoms have been ongoing for weeks. Physical exam reveals ptosis and speech that is "nasal" in quality. The patient underwent a positive tensilon test. Blood work was positive for acetylcholine receptor binding antibodies. Which of the following is first line in the treatment of this patient's neuromuscular disease? A. Carbidopa/levodopa B. Edrophonium C. Plasmapheresis D. Pyridostigmine

Pyridostigmine is the current treatment choice for a patient with myasthenia gravis. Myasthenia gravis is a neuromuscular disease that results in progressive muscle weakness as a result of antibodies to acetylcholine receptors at the neuromuscular junction. The more common presenting symptoms include ptosis, diplopia, dysphagia, dysarthria, and proximal muscle weakness. Orbital symptoms are the most common upon presentation. Symptoms can be transient with various remissions. Diagnosis is made with the ice pack test and the tensilon test. The tensilon test is performed by infusing a patient with edrophonium and observing for improvement in ocular symptoms. Serologic testing can be performed for acetylcholine receptor antibodies or muscle-specific tyrosine kinase receptor-associated protein antibodies. Nerve stimulation studies and electromyography can be further performed for diagnosis confirmation. First line treatment is pyridostigmine with immunomodulators as second line. Plasmapheresis and intravenous immunoglobulin (IVIG) can be used in a myasthenic crisis and preoperatively or while bridging to other therapies. Complications of myasthenia gravis include myasthenic crisis, which is a neuromuscular respiratory failure as a result of dysphagia and aspiration. Carbidopa/levodopa (A) is incorrect. Levodopa is the first line pharmacologic treatment for symptomatic Parkinson disease. It is combined with carbidopa to help counteract the systemic side effects associated with levodopa dosing. Edrophonium (B) was used historically in the diagnosis of myasthenia gravis as the drug involved in the tensilon test. Given its short duration of action and rapid onset it would not be an appropriate first line treatment for the disease. Plasmapheresis (C) is not the first line treatment for myasthenia gravis. It is performed to remove the acetylcholine receptor antibodies from the blood during a myasthenic crisis, prior to surgery, or when bridging a patient to slow acting immunomodulators.

A 75-year-old woman presents with complaints of weakness. She has difficulty getting up from a seated position, going up or down stairs, and lifting heavier objects that she could before without difficulty. You decide to order a muscle biopsy to rule out or confirm your suspicion of polymyositis. What muscle do you most likely want biopsied? A. Brachioradialis B. Gastrocnemius C. Latissimus dorsi D. Quadriceps femoris

Quadriceps femoris, or possibly anyone of the four large muscles of the thigh, are most commonly used for biopsy in confirming the diagnosis of polymyositis. Polymyositis is an idiopathic, inflammatory myopathy characterized by symmetrical, proximal muscle weakness. Thus the proximal muscles are used for biopsy and, due to their size, most commonly muscles in the lower extremity are biopsied. If upper extremity is desired, the deltoid is the most commonly biopsied. If positive, biopsy shows inflammatory changes in the muscle, including an abundance of plasma cells, necrotic myofibers, T lymphocytes, and macrophages. Proximal weakness can be demonstrated on physical exam, delayed nerve conduction and muscle contraction on electromyography, and elevated creatinine kinase 5-50 times the reference range is typically present and checked prior to ordering the biopsy. Brachioradialis (A) is not typically affected and is more distal to the proximal muscles of the arm that are affected by polymyositis. Gastrocnemius (B), the large muscle of the posterior lower leg, is not typically affected and is distal to the proximal muscles of the thigh that are affected by polymyositis. Latissimus dorsi (C) is a large flat muscle located on the posterior back. Muscles of the back, chest, and abdomen are not known to be affected by polymyositis.

A 72-year-old man presents to the clinic for his annual health check. He has a 12-year history of benign prostatic hyperplasia. He denies recent worsening of his voiding symptoms. Vitals are within normal limits. Previous digital prostate exams revealed a smooth, enlarged prostate, but now a small hard nodule is felt on the right lobe. On labs, prostate-specific antigen is 9.2 ng/mL. Records show his prostate-specific antigen was 3.4 ng/mL 12 years ago. Which of the following is the most appropriate next step? A. Increase his tamsulosin dose B. Obtain a baseline computed tomography of his prostate and recheck in six months C. Referral to oncology for radiation therapy D. Referral to urology for prostate biopsy

Referral to urology for prostate biopsy is the best course of action at this point. Based on a suspicious nodule felt on exam and elevated prostate-specific antigen (PSA), this patient likely has prostate cancer. Biopsy will confirm the diagnosis and help with staging. The biopsy is performed with transrectal ultrasound (TRUS) guidance. A minimum of 12 cores is typically taken. In cases where the digital exam is normal but PSA is elevated, it is recommend that the PSA be repeated prior to biopsy being done, and then recorded for prognostic purposes. Increase his tamsulosin dose (A) is not warranted in this situation, as his voiding symptoms are unchanged. Tamsulosin is an alpha-1 blocker used in the treatment of benign prostatic hyperplasia. Obtain a baseline CT scan of the prostate and recheck in six months (B) would only delay diagnosis of cancer and its treatment. Currently, magnetic resonance imaging (MRI) is being studied to possibly help improve the accuracy of transrectal ultrasound guidance and areas being biopsied, but currently there is no role for CT scanning. Referral to oncology for radiation therapy (C) might be the next step after biopsy, depending upon results, but would not be next in the evaluation of this patient's work up.

A 65-year-old man with significant tobacco use presents with general malaise, unexplained weight loss, and occasional flank pain. Computed tomography of the abdomen shows a localized renal mass in the left kidney. What is the only curative treatment for this condition? A. Chemotherapy B. Radiofrequency ablation C. Stem cell transplantation D. Surgical resection

Renal cell carcinoma is the most common type of kidney cancer in adults. Cigarette smoking doubles the risk of developing renal cell carcinoma. Other risk factors include obesity, occupational exposure to certain chemicals and hypertension. Diagnosis is commonly delayed as symptoms are nonspecific and insidious in development. Few patients present with the classic triad of flank pain, hematuria, and flank mass. Other more common signs and symptoms include weight loss, fever, hypertension, hypercalcemia, night sweats, malaise and left-sided varicocele in men. Helpful laboratory tests include urinalysis with urine cytology to evaluate for abnormal cells, complete blood count to assess for the presence of anemia or infection, and an electrolyte panel with renal profile to assess kidney function. Imaging studies used to diagnose and stage renal masses include urography, computed tomography, positron emission tomography scan, ultrasonography, and magnetic resonance imaging. Surgical resection remains the only known curative treatment for localized renal cell carcinoma, but can also be used to improve outcomes or for palliative care. Therapy using immunomodulatory agents is standard in patients with metastatic disease. Chemotherapy is only used occasionally and is only effective for certain types of tumors. Chemotherapy (A) and radiofrequency ablation (D) are only used in certain cases to treat tumors, but are not considered curative. Stem cell transplantation (B) is being studied as an experimental treatment at this time.

A 62-year-old man presents reporting urinary hesitancy, dribbling, and feeling of incomplete voiding. He states his symptoms have been progressively worsening for the last six months. He denies hematuria, pain with urination, or scrotal swelling. Which of the following physical exam findings is most consistent with the diagnosis of benign prostatic hypertrophy? A. Asymmetry of the prostate B. Boggy consistency of the prostate C. Decreased anal sphincter tone D. Rubbery consistency of the prostate

Rubbery consistency of the prostate on digital rectal exam is a typical finding in benign prostatic hypertrophy. A prostate should be non-tender, symmetric, and have a smooth texture. Measurements of size via palpation are often inaccurate, however, a normal prostate is generally about the size of a walnut, or less than 20 grams. Rectal mucosa should also be smooth and free of palpable deformity. Additionally, anal sphincter tone should be normal and a bulbocavernosus muscle reflex should be noted. The size of the prostate does not determine the severity of symptoms. The American Urological Association has developed symptom and quality of life assessment tools that focus on the frequency of symptoms and impact of benign prostatic hypertrophy, or lower urinary tract symptoms (LUTS). The severity of the symptoms guides treatment. The most common symptoms include a sensation of incomplete voiding, urinary frequency, urinary urgency, weak stream, straining, and nocturia. Progression of symptoms and severity is variable. Generally, a diagnosis is made clinically, with urinalysis, serum creatinine, and prostate-specific antigen testing performed as a baseline assessment for abnormalities which may suggest complications or other causes of symptoms, such as bladder or prostate cancer. Treatment most often consists of an α-adrenergic blocker, such as doxazosin, and potentially the addition of a 5-α-reductase inhibitor, such as finasteride. If medical therapies fail, further invasive treatment by a urologist may be indicated. Complications may include urinary tract infections, cystitis, interstitial cystitis, bladder outlet obstruction, urinary retention, and rarely hydronephrosis and renal injury. Asymmetry of the prostate (A) is worrisome for cancer, particularly if there are areas of nodularity or if the prostate is hard. A Boggy consistency of the prostate (B) is more consistent with prostatitis, which may also present with fever, perineal pain (particularly with sitting), urethral discharge, with lower urinary tract symptoms. Prostate palpation is generally contraindicated if prostatitis is suspected. Decreased anal sphincter tone (C) is more likely to represent a neurogenic bladder, with detrusor and urinary sphincter overactivity or underactivity as a result, depending on the location of the neurologic abnormality.

A 35-year-old woman presents to the clinic complaining of worsening symptoms related to her recently diagnosed scleroderma (systemic sclerosis). Which of the following symptoms is most consistent with the diagnosis of scleroderma? A. Dry mouth and dry eye complaints B. Skin tightening around the fingers with pitting at the fingertips C. Tenderness at the temporal areas D. Weakness of proximal muscles without pain

Scleroderma is an autoimmune disease causing vascular damage and excess production and deposition of collagen in the extracellular matrix of the skin and other tissues. Skin involvement is a nearly universal feature of scleroderma and it is characterized by variable extent and severity of thickening and hardening of the skin. Skin tightening around the fingers with pitting at the fingertips is the most common early sign of scleroderma. Other common clinical manifestations may include Raynaud phenomenon, gastrointestinal symptoms of GERD, dysphagia, esophageal strictures, pulmonary disease, cardiac and renal involvement. There are two types of scleroderma: diffuse and limited. Classically, limited cutaneous scleroderma causes CREST syndrome; Calcinosis of the skin, Raynaud phenomenon, Esophageal dysmotility, Sclerodactyly, and Telangiectasia. Laboratory studies may reveal elevated antinuclear antibody (present in all patients but has low specificity), anti-topoisomerase I (anti-Scl-70) antibody (specific for diffuse disease), anti-centromere antibody (specific for limited disease). There is currently no treatment for the underlying disease process. Treatment is symptomatic and based on the organ system involved. Calcium-channel blockers for Raynaud phenomenon, ACE inhibitors for renal hypertensive crisis, H2 blockers and PPIs for gastrointestinal symptoms, bosentan for pulmonary hypertension, and cyclophosphamide for pulmonary fibrosis. Dry mouth and dry eye complaints (A) are the hallmark of symptoms of Sjögren's syndrome. Tenderness at the temporal areas (C) is the classic finding of temporal arteritis. Temporal tenderness is not typical of systemic sclerosis. Weakness of proximal muscles without pain (D) is the common finding with polymyositis. Systemic sclerosis causes weakness but typically with associated pain and more commonly in the distal extremities.

A 47-year-old woman presents to the clinic reporting bilateral swelling of her cheeks, as well as dry mouth and eyes. She reports insidious progression of the symptoms, with no distinct date of onset. Physical exam reveals dry mucous membranes and bilateral parotid gland swelling. Lab results indicate elevated rheumatoid factor, elevated antinuclear antibody with a speckled staining pattern, and a positive anti-Ro antibody test. Which of the following is the most likely diagnosis? A. Lymphoma B. Rheumatoid arthritis C. Sjögren syndrome D. Systemic lupus erythematosus

Sjögren's syndrome (SS) is a chronic autoimmune inflammatory disorder characterized by diminished lacrimal and salivary gland function due to lymphocytic infiltration of the secretory glands. Sjögren syndrome may be primary, not associated with other autoimmune rheumatic diseases, or secondary to other rheumatologic diseases, particularly systemic lupus erythematosus and rheumatoid arthritis. Women are affected at a much higher rate compared to men. The disease results in chronic activation of the immune system, particularly in polyclonal B cells, which may predispose to lymphoma. In glandular tissues, lymphocytic infiltration is commonly noted with a preponderance of CD4 cells. Presentation may include keratoconjunctivitis sicca syndrome (dry eyes, mouth), xerostomia (dry mouth), and parotitis. Raynaud phenomenon, arthritis, dry cough, angular cheilitis, and oral candidiasis. Rarely, neurological symptoms, sequelae of renal damage (hypokalemia, proteinuria), and gastrointestinal symptoms can occur. Tests include a Schirmer test (quantifies tear production), salivary flow rate, minor salivary gland biopsy, anti-Ro and anti-La antibodies, rheumatoid factor, antineutrophil antibody titer (a speckled stain pattern may indicate Sjögren syndrome), ocular staining (rose bengal test), and a slit lamp exam. Magnetic resonance imaging or ultrasound of the parotid glands may be used. Diagnosis may be complicated if there is a history of head or neck radiation, infection with human immunodeficiency virus, active hepatitis C, sarcoidosis, amyloidosis, graft-versus-host disease, or IgG4 defects. Treatment for Sjögren syndrome is largely symptomatic. Keratoconjunctivitis sicca symptoms are initially treated with artificial tear lubricants, pilocarpine or cevimeline. Avoidance of anticholinergic and antihistamine medications, as well as the use of artificial saliva products, are the primary approaches to addressing dry mouth. Steroids, immunosuppressants, and biologic medications are implemented in more severe cases. Nonpharmacologic treatment may include the use of humidifiers and moisturizing creams for dry skin. In addition to multidisciplinary medical care, routine dental visits and topical fluoride use are also advised due to an increased risk for caries. Lymphoma (A) may be a complication of Sjögren syndrome but would be expected to present with additional findings such as leukopenia, lymphocytosis, fever, lymphadenopathy, and splenomegaly. Rheumatoid arthritis (B) is frequently encountered with Sjögren syndrome but would present with joint manifestations, such as stiffness and pain from involvement of serosa, as well as a positive cyclic citrullinated peptide antibody test. Systemic lupus erythematosus (D) is also a potential comorbid condition accompanying Sjögren syndrome, but would also likely be associated with a photosensitive rash (malar or discoid), arthritis, serositis, and presence of additional autoantibodies (anti-Smith, anti-dsDNA).

A 23-year-old woman, with no risk factors for heart disease, presents to the clinic with complaints of sternal chest pain, fatigue, and shortness of breath. She was recently diagnosed with sarcoidosis. In your review of systems check, other than pulmonary, what area do you want to focus on the most? A. Gastrointestinal B. Musculoskeletal C. Renal D. Skin

Skin and lymph nodes are the two most commonly affected areas, after the lungs, by sarcoidosis. History and physical exam should include a thorough review of skin changes. Sarcoidosis is an inflammatory condition characterized by the presence of noncaseating granulomas in organs and tissues, such as the lungs, skin, lymph nodes, eyes, kidneys, joints, and various others. In the skin, various rashes - papular, nodular, or plaque-like can occur; scarring, changes in skin color, and erythema nodosum may develop. Lungs are most commonly involved and characterized by bilateral hilar adenopathy and diffuse reticular or ground glass opacities on chest X-ray. Symptoms include pain around the sternum, shortness of breath, fatigue, dry cough, and wheezing. Gastrointestinal (A) involvement is rare with sarcoidosis. Less than 1% have clinically recognizable GI disease. Musculoskeletal (B) may occur in up to 10% of sarcoidosis patients. Cystic lesions may be seen on X-ray of the joints, but typically swelling around the joints (periarthritis) and inflammation of tendons are the only findings. Chronic arthritis is rare. Renal (C) involvement is also less common in sarcoidosis compared to other systems. Electrolyte abnormalities involving calcium metabolism can lead to deposition of calcium, leading to kidney failure. Interstitial nephritis or membranous nephropathy can also result depending upon the area of kidney involvement.

Which of the following represents the best treatment plan for correction of systemic acidosis in status epilepticus? A. Fomepizole 15 mg/kg IV infusion over 30 minutes B. Intravenous insulin infusion at 0.1 units/kg/hour C. Sodium bicarbonate 2-5 mEq/kg IV infusion over four to eight hours D. Watchful waiting for auto-correction of the acidosis once seizure activity is controlled

Status epilepticus refers to a prolonged epileptic crisis (> or equal to 5 minutes of continuous seizure activity) or more than one seizure without recovery from the postictal state in between episodes. Patients may present with a long history of seizure disorder or may be experiencing their first seizure event. In patients with a history of seizure disorder, a change to their medication regimen is the most common precipitating factor in status epilepticus. Other causes of the disorder are stroke, hemorrhage, tumor, infection, electrolyte abnormalities and toxins. Patients in status epilepticus may present with a sustained tonic-clonic seizure or may suffer sustained simple partial or complex partial seizures. Complications resulting from status epilepticus can be life-threatening and include respiratory failure, hypotension, acidosis, hyperthermia, rhabdomyolysis, and aspiration. Treatment of status epilepticus involves stabilizing the patient and rapid administration of a benzodiazepine such as midazolam. This is followed by administration of an intravenous anticonvulsant such as phenytoin. If the seizure activity does not cease after these two measures, then a barbiturate or a general anesthetic such a propofol are used. Correction of the systemic acidosis found in status epilepticus is not necessary since the acidosis is thought to have anticonvulsant effects. Also, the acidosis corrects itself quickly once seizure activity has ceased, and treatment with alkalinizing agents often leads to a rebound systemic alkalosis. Treatment of status epilepticus with benzodiazepines and anticonvulsants should not be delayed, due to the serious nature of the disorder. However, after the patient is stabilized and no longer actively seizing, measures must be taken to investigate the cause of the seizure activity. These measures are symptom-specific but can include complete blood count, complete metabolic panel, toxicology screening, electroencephalography, computed tomography or magnetic resonance imaging of the brain, chest radiograph, or lumbar puncture. Fomepizole 15 mg/kg IV infusion over 30 minutes (A) is the treatment of choice for use in metabolic acidosis secondary to methanol and ethylene glycol overdose. After the initial infusion, 10 mg/kg IV of fomepizole is given every 12 hours for four doses, then 15 mg/kg is given every 12 hours until the serum concentration of fomepizole is between 8.6 and 24.6. Treatment is maintained until ethylene glycol or methanol serum levels are under 20 mg/dL. Intravenous insulin infusion at 0.1 units/kg/hour (B) is the treatment of choice for ketoacidosis. Diabetic ketoacidosis is treated with fluid resuscitation and insulin therapy until the ketoacidosis abates. Aggressive correction of the blood glucose to under 200 mg/dL should not be attempted in the first four to five hours of treatment, as profound hypoglycemia can result. Sodium bicarbonate 2-5 mEq/kg IV infusion over four to eight hours (C) is the treatment of choice in certain cases of severe acute metabolic acidosis. Adjustments in the rate of infusion are made based on the serum pH. Patients with metabolic acidosis as a result of status epilepticus, as mentioned previously, do not often require normalization of their acidosis, as the blood pH tends to self-regulate quickly once the seizure activity has ceased. If sodium bicarbonate infusion is administered to patients with status epilepticus, the result is often rebound alkalosis. Studies have also shown that acidosis can have anticonvulsant effects.

A 32-year-old man is evaluated in the emergency department for complaints of shortness of breath and palpitations that began at rest approximately 30 minutes ago. He recalls experiencing similar symptoms three days ago that self-resolved after 10 minutes. He denies additional medical history and is not taking any prescription or alternative medications. On physical exam, he has clear and equal breath sounds bilaterally. His blood pressure is 134/88 mm Hg, heart rate 180 beats per minute, and respiratory rate is 24. An ECG shows a narrow-complex supraventricular tachycardia. You have him perform vagal maneuvers but do not see a change in his heart rate or rhythm. Which of the following is the next best treatment? A. Adenosine 6 mg IV B. Amiodarone 300 mg IV C. Atropine 0.5 mg IV D. Synchronized cardioversion

Supraventricular tachycardia (SVT) is a tachydysrhythmia occurring above the ventricles as the name implies. The most common type of SVT in adults is atrioventricular nodal reentrant tachycardia (AVNRT). AVNRT often presents in teenagers and young adults. In stable patients, vagal maneuvers (e.g. bearing down) can be performed initially. Vagal maneuvers increase vagal tone on the AV node. If vagal maneuvers are unsuccessful in the stable patient, adenosine 6 mg IV push should be rapidly administered due to its short half-life. Adenosine works by transiently inhibiting transmissions through the AV node. A 12 mg dose can be followed if the 6 mg dose does not treat the dysrhythmia. Adenosine terminates AVNRT in over 80 percent of cases. Atropine (C) is an anticholinergic medication that inhibits vagal tone on the AV node. Because of this, it is used in the treatment of symptomatic bradycardia, not tachycardia. Amiodarone (B) is a class III antiarrhythmic. It is most commonly used for the treatment of wide complex tachydysrhythmias. Synchronized cardioversion (D) is indicated for all tachydysrhythmias in unstable patients (i.e. hypotensive patients, patients with chest pain or difficulty breathing, mental status changes).

Which of the following is associated with aortic stenosis? A. de Musset sign B. Holodiastolic decrescendo murmur C. Syncope D. Wide pulse pressure

Syncope is a common presenting feature of aortic stenosis. In aortic stenosis, the aortic valve becomes hardened and its leaflets do not fully open during systole, causing a harsh systolic murmur. Aortic stenosis also leads to a fixed stroke volume despite vasodilation and often a subsequent decline in systemic blood pressure. Such a decline can lead to exertional syncope. The classic triad seen with aortic stenosis is chest pain, dyspnea, and syncope. Aortic regurgitation is a disease of the aortic valve that causes retrograde flow of blood from the aorta back into the left ventricle during diastole. Aortic regurgitation leads to a wide pulse pressure (D) (defined as the difference between systolic and diastolic blood pressures greater than 60 mm Hg). The murmur of aortic regurgitation is diastolic and in chronic disease can lead to a diastolic decrescendo murmur (B). A head bob which occurs with each heartbeat is known as de Musset sign (A), and this is also a sign of aortic regurgitation, not stenosis.

A 67-year-old woman presents to the emergency department for evaluation of a productive cough for five days. She is alert and oriented. Physical exam reveals crackles at the left lung base. Blood pressure is 102/68, temperature is 100.9oF, respirations are 33 per minute, heart rate is 105 beats per minute, and oxygen saturation is 93% on room air. Labs reveal a BUN of 18 mg/dL and creatinine of 1.0 mg/dL Her chest X-ray shows an infiltrate in the left lower lobe. What is the most appropriate course of action? A. Admission to the ICU and start treatment with levofloxacin B. Admission to the medical ward and start treatment with ceftriaxone and azithromycin C. Discharge the patient home with a five-day course of azithromycin and instructions to follow up with primary care in 48-hours D. Discharge the patient home with a seven-day course of doxycycline and instructions to follow up with primary care within three days

The CURB-65 algorithm can be used along with clinical judgement in determining the appropriate site of care for a patient with community acquired pneumonia. One point is assigned to each of the five clinical features: The appropriate treatment for a patient without risk factors for resistance who is being admitted to a medical ward is a combination of a macrolide and an anti-pneumococcal beta-lactam. Outpatient treatment with azithromycin (C) or doxycycline (D) (second line agent) would be appropriate for a patient with a CURB-65 score of 0 or 1. This patient has a CURB-65 score of 2 (age + tachypnea), thus ICU admission (A) is not necessary. Also, a patient being admitted to the ICU should be started on a dual antibiotic therapy rather than monotherapy.

A 68-year-old man presents to the clinic with a past medical history of hepatitis B infection and hepatic cirrhosis. Which of the following is the best choice for surveillance of development of hepatocellular carcinoma in this patient? A. Abdominal CT scan every 12 months B. Abdominal CT scan every two years C. Liver ultrasound every 12 months D. Liver ultrasound every six months

The major risk factor for hepatocellular carcinoma is cirrhosis of any etiology. Chronic hepatic diseases such as hepatitis B (dominant cause worldwide), hepatitis C (dominant cause in the United States and Europe), steatotic liver disease, hemochromatosis, Wilson's disease, and alcoholic hepatitis can increase the risk for development of cirrhosis and hepatocellular carcinoma. Screening patients with cirrhosis for the development of hepatocellular carcinoma has shown to improve early detection rates and survival. Patients with cirrhosis of any type should receive a liver ultrasound every six months to monitor for the development of hepatocellular cancer. Patients who have coinfection with hepatitis and human immunodeficiency virus and patients who have neurovisceral porphyrias should undergo regular surveillance as well. Treatment choices for hepatocellular carcinoma depend on the stage of the disease at diagnosis and can include surgical resection, liver transplantation, radiofrequency ablation, percutaneous ethanol injection, radiation therapy, and chemotherapy. Abdominal CT scans every 12 months (A) or every two years (B) are not recommended screening methods for the development of hepatocellular carcinoma. CT scans involve ionizing radiation and can increase the incidence of carcinoma. CT scanning can be used to further evaluate a mass found on ultrasound or to stage the carcinoma once it has been identified. Liver ultrasound every 12 months (C) is recommended for patients with neurovisceral porphyrias. Patients with cirrhosis require more frequent surveillance. Patients who are carriers of hepatitis B require surveillance if they meet the following criteria: Asian man over 40-years-old, Asian woman over 50-years-old, African or North American Black patients, patients with a family history of hepatocellular carcinoma, and any patients with hepatitis B and cirrhosis. White patients who are hepatitis B carriers but have low viral load do not require regular surveillance for development of hepatocellular carcinoma. Likewise, patients with chronic liver disease such as Wilson's disease, hemochromatosis, steatotic liver disease, and alcoholic liver disease do not require regular surveillance for development of hepatocellular carcinoma if cirrhosis has not yet developed.

Which of the following is the mode of genetic transmission for hemophilia B? A. Autosomal dominant B. Autosomal recessive C. X-linked dominant D. X-linked recessive

The mode of inheritance for hemophilia B is X-linked recessive. Hemophilia B is a bleeding disorder characterized by a deficiency of factor IX. Most cases of hemophilia B are hereditary, but the disorder can also be acquired through spontaneous mutation or autoimmunity. Hemophilia B is less common than hemophilia A. Hemophilia can be categorized as mild, moderate, and severe, based on the percentage of factor activity. Transmission occurs from female carrier to half of her male children; father-to-son transmission does not occur. Most patients with severe hemophilia present during the first 18 months of life with easy bruising, hemarthrosis, or excessive bleeding after an invasive procedure. Patients with severe hemophilia can present with spontaneous bleeding or bleeding that is out of proportion to the degree of injury. Mild hemophilia may only become apparent when the patient encounters significant hemostatic challenges, such as surgery or trauma. Common sites of bleeding include joints, muscles, oropharynx, gastrointestinal tract, and genitourinary tract. Intracranial hemorrhage is rare compared to other sites of bleeding but can be life-threatening. Initial laboratory studies should include prothrombin time (PT), partial thromboplastin time (PTT), and platelet count. Hemophilia will result in prolonged PTT and normal PT and platelet count. Factor replacement is the mainstay of treatment. The dosage of factor replacement is dependent upon the severity of the bleed and the severity of the factor deficiency. Hemophilic arthropathy is one of the most common complications of hemophilia. In autosomal dominant (A) disorders, affected offspring have one copy of the abnormal gene on a non-sex chromosome. Examples of common autosomal dominant disorders include polycystic kidney disease, familial adenomatous polyposis (FAP), Huntington disease, hypertrophic cardiomyopathy, and multiple endocrine neoplasia. In autosomal recessive (B) disorders, the affected individual possesses two copies of the abnormal gene. Common autosomal recessive disorders include cystic fibrosis, phenylketonuria, sickle cell disease, congenital adrenal hyperplasia, and Wilson disease. X-linked dominant (C) disorders occur when the dominant gene is located on the X chromosome. X-linked dominant disorders are not as common as X-linked recessive disorders. X-linked dominant disorders include Rett syndrome, craniofrontonasal dysplasia, and fragile X syndrome.

A 37-year-old man was found to have a pituitary adenoma on MRI after presenting to the hospital complaining of headaches and visual impairment. Based on the location of the benign tumor, compression of which cranial nerve is most likely causing the patient's visual symptom? A. Abducens nerve B. Oculomotor nerve C. Optic nerve D. Trochlear nerve

The optic nerve is the most commonly affected cranial nerve in pituitary adenomas. The pituitary sits in the sella turcica of the skull. Due to the location of the tumor, the pituitary adenoma most commonly grows in the suprasellar direction applying pressure to the optic chiasm of the optic nerve (cranial nerve II). The result of the optic chiasm compression results in bitemporal hemianopsia. Patients will complain of diminished or impaired vision in their temporal visual fields. Pituitary adenomas are also associated with headaches and other visual disturbances depending on the size and direction of growth of the tumor. Diagnostic studies can include MRI, CT scan, and evaluation of hormone levels depending on the type of tumor. Transsphenoidal resection of the adenoma is the common treatment for this type of tumor. Abducens nerve (A) is incorrect. The abducens nerve exits the skull at the superior orbital fissure but is not commonly affected by pituitary adenomas. Oculomotor nerve (B) is incorrect. After the optic nerve, the oculomotor nerve is the second most commonly affected cranial nerve involved in pituitary adenomas. The oculomotor nerve is compressed when the pituitary adenoma grows laterally instead of superiorly. Patients may present complaining of diplopia if this nerve is involved. Trochlear nerve (D) is incorrect as it is not commonly affected by pituitary adenomas. Like the abducens nerve, it also exits the skull through the superior orbital fissure.

A 26-year-old pregnant woman presents to the office to discuss her concerns regarding genetic conditions that may affect her child. She states that her father was diagnosed with Huntington disease twelve years ago. Patient reports the only other individual in her family known to have Huntington disease was her paternal grandmother. The patient has not personally been tested. Which of the following values indicates the likelihood that the patient received the abnormal gene from her father if it was passed down in an autosomal dominant disorder? A. 100% B. 25% C. 50% D. The patient cannot receive the abnormal gene from her father

The patient has a 50% chance of receiving the Huntington disease gene from her father. Huntington disease is an autosomal dominant neurodegenerative disorder, meaning that if the abnormal gene is inherited from the parent, the disease can develop. The gene implicated in Huntington disease is the huntingtin gene (HD) and the disorder develops based on the number of cytosine-adenine-guanine (CAG) trinucleotide repeats in the gene. Symptoms of Huntington disease include chorea, rigidity, hypotonia, saccadic eye movements, psychiatric symptoms, and eventual cognitive decline. Symptoms are progressive. Diagnosis is made based on presenting symptoms, family history, and genetic testing. Treatment is not curative but rather focused on symptom relief. Tetrabenazine and neuroleptics may be used to lessen chorea and psychiatric symptoms. 100% (A) is incorrect. The only individuals in her family who had Huntington disease were the patients paternal grandmother and her father. This makes it most likely that her father has one dominant and one recessive huntingtin gene. The patients father would need to have received the dominant trait from both his mother and father in order to have a 100% possibility of passing the abnormal gene to his daughter. 25% (B) is incorrect. This would be the correct choice for an autosomal recessive disease. The patient cannot receive the abnormal gene from her father (D) is incorrect given the genetic nature of the disease. Her father would pass down either the dominant or recessive trait of the gene.

A 67-year-old man presents to the emergency department with an acute exacerbation of chronic bronchitis and is hospitalized. His home medications include inhaled albuterol and ipratropium. Administration of which of the following is contraindicated? A. Albuterol 2.5 mg per nebulizer B. Levofloxacin 500 mg IV C. Methylprednisolone 125 mg IV D. Theophylline 200 mg per oral route

Theophylline is a medication that may be used for the treatment of chronic obstructive pulmonary disease (COPD) if other treatments fail or if they are contraindicated. Oral theophylline is not initiated in acute exacerbations of COPD, however chronic theophylline treatment should be continued in acute exacerbations, providing it is maintained at a therapeutic serum level (between 10 and 20 mcg/mL) and no other contraindications exist. Theophylline is a methylxanthine derivative that has effects on adenosine-mediated calcium channels and can result in increased muscular contraction, as well as having effects on phosphodiesterases (particularly isoenzymes III and IV) which produce respiratory smooth muscle relaxation and inhibition of inflammatory cells. Historically, theophylline was used quite often in COPD and asthma treatment, but it has a narrow therapeutic range, and can easily become toxic. The effects of toxicity are primarily due to sympathomimetic response, such as tachycardia, hypotension, altered cognition, hyperglycemia, and seizure. With the advent of newer treatments, such as inhaled beta agonist and anticholinergics, theophylline has been relegated to use in refractory cases, as the risk of toxicity is too high to justify the possible benefit. Albuterol (A) is a beta agonist and is one of the primary medications used for exacerbations of COPD or asthma. It is much safer than theophylline, and requires no special monitoring. Levofloxacin (B), is a fluoroquinolone antibiotic with broad coverage and may be indicated in patients that have an acute COPD exacerbation and require hospitalization. Methylprednisolone (C) is a glucocorticoid and effectively reduces inflammation. In COPD, steroids help to improve hypoxemia, lung function, and decrease hospital days and treatment failure rates.

A 24-year-old woman is admitted to the intensive care unit after developing thyroid storm status post appendectomy and is being treated with propylthiouracil. What is the mechanism of action of this drug? A. Decreases adrenergic tone B. Decreases vasomotor instability C. Inhibits conversion of thyroxine to triiodothyronine D. Inhibits release of thyroid hormone

Thyroid storm is a rare, life-threatening condition which is characterized by severe clinical manifestations of thyrotoxicosis. It is most commonly precipitated by acute events like thyroid surgery, nonthyroidal surgery, trauma, infection, an acute iodine load, or childbirth. It can also develop in patients with longstanding untreated hyperthyroidism. Clinical features include tachycardia, congestive heart failure, hyperpyrexia (104°F to 106°F), agitation, anxiety, delirium, stupor, and coma. Physical examination may reveal a goiter, lid lag, hand tremor, and warm, moist skin. Laboratory studies often show a low TSH (thyroid-stimulating hormone) and a high free T4 (thyroxine) or T3 (triiodothyronine) concentration. Diagnosis is based on the presence of severe and life-threatening symptoms in a patient with evidence of hyperthyroidism via laboratory studies. Treatment consists of a combination of medications, but generally involves a beta blocker (propranolol), a thionamide (propylthiouracil or methimazole), iodine solution, and glucocorticoids. For patients admitted to the intensive care unit, propylthiouracil (PTU) is the preferred thionamide over methimazole. While both thionamides block the synthesis of thyroid hormone, propylthiouracil also inhibits conversion of thyroxine to triiodothyronine. Beta blockers decrease adrenergic tone (A) which is reflected in the decrease in heart rate and blood pressure following administration. Propranolol is the preferred beta blocker as it also reduces serum T3 levels through inhibition of type 1 deiodinase when given in high doses. Vasomotor instability (B) is decreased by the administration of glucocorticoids, which also act to reduce T4 to T3 conversion, and may aid in the treatment of concomitant adrenal insufficiency. Iodine solution inhibits release of thyroid hormone (D). It should be given one hour after the first dose of thionamide to avoid the iodine solution being used as a substrate for new hormone synthesis.

Which of the following best describes the etiology of the jaundice seen in patients with thyroid storm? A. Direct constricting effects of thyroid hormone on the biliary duct B. Hepatic tissue hypoxia due to increased peripheral consumption of oxygen C. Hypotension leading to decreased gut motility D. Impaired reabsorption of thyroid hormone in the enterohepatic circulation

Thyroid storm refers to thyrotoxicosis in its extreme form. Patients with Grave's disease or toxic multinodular goiter can become acutely decompensated through an excess of circulating thyroid hormone and subsequent hypermetabolism. Signs and symptoms of thyroid storm include hyperpyrexia, nausea, vomiting, diarrhea, mental status changes, jaundice, high-output congestive heart failure, cardiac tachyarrhythmias, hypertension, and diaphoresis. The exact mechanism whereby thyrotoxicosis evolves into thyroid storm in certain patients is unknown, although stress, infection, trauma to the thyroid gland, diabetic ketoacidosis, surgery, pregnancy, and radioactive iodine therapy are all known precipitating factors. Certain drugs, such as anesthesia agents, anticholinergics, adrenergics, nonsteroidal anti-inflammatory drugs, salicylates, and chemotherapeutic agents, can also trigger thyroid storm in a patient with underlying thyrotoxicosis. Heart failure in thyroid storm is the result of increased oxygen demand by the tissues and the heart's inability to keep up with such a high demand (high-output heart failure). Likewise, jaundice is the result of hepatic tissue hypoxia due to increased peripheral consumption of oxygen and unmet oxygen needs of the hepatocytes. Jaundice can also result from hepatic congestion secondary to heart failure. While less common than other signs and symptoms of thyroid storm, the presence of jaundice portends a poor prognosis. Treatment of thyroid storm typically begins with administration of propranolol to slow the heart rate, lower the body's metabolic demands, and block peripheral conversion of T4 to T3. Next, propylthiouracil 600 mg is administered, followed by iodine administration. Correction of volume and electrolyte imbalance as well and cooling of body temperature is essential. Corticosteroids are given to decrease the rate of peripheral conversion of T4 to T3. Thyroid hormone has no direct constricting effects on the biliary duct (A). Excess thyroid hormone in a patient with thyroid storm causes an increase in metabolism that affects nearly every organ system. Increases in metabolic rate cause increases in demand for oxygen by peripheral tissues, as well as organ tissues. This can lead to hypoxia of hepatocytes and jaundice. It also leads to high-output heart failure as well as encephalopathy and mental status changes. The adrenergic effects of thyroid hormone cause increased blood pressure, not hypotension leading to decreased gut motility (C). In fact, patients in thyroid storm experience increased gut motility with nausea, vomiting, and diarrhea. Patients with thyroid storm may become hypotensive, but this is more indicative of heart failure and shock than of direct effects of thyroid hormone on blood vessels. A patient in thyroid storm who becomes hypotensive requires immediate fluid resuscitation. There is no impaired reabsorption of thyroid hormone in the enterohepatic circulation (D) of patients in thyroid storm. In fact, thyroid hormone is reabsorbed and recirculated through the enterohepatic system in a healthy individual and during thyroid storm. Administering cholestyramine during thyroid storm can inhibit the reabsorption of thyroid hormone from the gut.

A 32-year-old man with a history of inherited hemolytic anemia and methicillin-resistant Staphylococcus aureus (MRSA) presents to the office complaining of an abscess. Physical exam reveals a 4 cm fluctuating abscess in the left axilla with extensive surrounding cellulitis. Foul smelling purulent material is expressed during incision and drainage. Patient reports a history of a hemolytic episode last year after being given an antibiotic by another provider for a similar abscess. What antibiotic was likely given for the patient's previous abscess? A. Cephalexin B. Doxycycline C. Nitrofurantoin D. Trimethoprim-sulfamethoxazole

Trimethoprim-sulfamethoxazole and other medications in the sulfonamide drug class are known to be oxidative drugs. Oxidative drugs can lead to hemolytic episodes in patients with G6PD (glucose-6-phosphate dehydrogenase) deficiency due to the resultant decrease in glutathione allowing oxidative injury of the red blood cells. This injury to the red blood cells makes the cells susceptible to hemolysis due to their consequential rigid shape. G6PD deficiency is an X-linked recessive inheritance disorder. Because of this inheritance pattern the disease is less commonly seen in females. Most individuals are asymptomatic until they are exposed to a triggering substance, resulting in the development of jaundice, pallor, and dark colored urine. A small group of patients with severe G6PD deficiency can have chronic hemolysis. Hemolytic episodes are most commonly caused by medications (aspirin, dapsone, methylene blue, nitrofurantoin, phenazopyridine, primaquine, quinidine, sulfonamides, and uricase), infection, Henna dyes, fava beans, and bitter melon. Screening tests include the fluorescent spot test and the methemoglobin reduction test. Individuals with a positive screening test will go on to have a confirmatory test performed. Treatment includes avoiding offending agents. For hemolytic episodes, aggressive fluid hydration and potential transfusion for severe anemia are indicated. Cephalexin (A) is incorrect as it is not an oxidative drug nor is it known to precipitate hemolytic episodes in patients with G6PD deficiency. Cephalexin would not be the correct treatment choice for a skin infection caused by MRSA. Doxycycline (B) is not known to be an oxidative drug leading to hemolytic episodes in patients with G6PD deficiency. Nitrofurantoin (C) is an oxidative drug known to cause hemolytic episodes in patients with G6PD deficiency, but it would not be the appropriate antibiotic choice for a skin infection as it is commonly reserved for infections of the urinary tract.

A patient presents with a grade II/VI late systolic murmur heard best in the fifth intercostal space in the midclavicular line. Which of the following cardiac processes best correlates with this finding? A. Left-to-right blood flow across a small, muscular ventricular septal defect B. Regurgitant blood flow from the left ventricle to the left atrium C. Regurgitant blood flow from the right ventricle to the right atrium D. Turbulent blood flow across redundant mitral valve tissue

Turbulent blood flow across redundant mitral valve tissue describes mitral valve prolapse, which is the most common cause of a late systolic murmur and is typically preceded by a click. Mitral valve prolapse occurs when the leaflets of the mitral valve billow into the left atrium during systole. Mitral valve prolapse can be the result of a disruption or elongation of the leaflets, chordae, or papillary muscles. Patients may be born with elongated leaflets or may acquire mitral valve prolapse secondary to myocardial ischemia and subsequent papillary muscle dysfunction. Some genetic syndromes such as Marfan syndrome and certain connective tissue disorders can also cause mitral valve prolapse. The murmur of mitral valve prolapse occurs later in systole because the disparity of valvular redundant tissue to valvular ring increases with a decrease in left ventricular volume. Maneuvers which decrease left ventricular volume, such as Valsalva and standing, can cause the murmur of mitral valve prolapse to occur earlier in systole and last longer. Patients with mitral valve prolapse may develop mitral valve regurgitation if the prolapse is great enough to cause lack of apposition of the mitral leaflets. Mitral valve regurgitation is diagnosed by transesophageal echocardiogram. Definitive treatment is through surgical or percutaneous repair of the mitral valve, although this is reserved for only the severest cases. Left-to-right blood flow across a small, muscular ventricular septal defect (A) is most likely to cause an early systolic murmur because the defect in the muscular wall closes soon after systole. Larger ventricular septal defects may cause holosystolic murmurs because the pressure in the left ventricle remains higher than the pressure in the right ventricle throughout systole. Some patients with large ventricular septal defects that also suffer from pulmonary hypertension will have early systolic murmurs because the pressure in the right ventricle approaches left ventricular pressure due to right ventricular hypertrophy. A late systolic murmur is uncommon in ventricular septal defect. Regurgitant blood flow from the left ventricle to the left atrium (B) describes mitral valve regurgitation. Regurgitant blood flow is greatest during the early part of systole and levels off as atrial and ventricular pressures approximate each other. Thus, the murmur of mitral regurgitation is classically a holosystolic decrescendo murmur. Patients who suffer mitral valve regurgitation as a result of mitral valve prolapse can exhibit a holosystolic murmur, as opposed to the late systolic murmur of isolated mitral valve prolapse. Regurgitant blood flow from the right ventricle to the right atrium (C) describes tricuspid valve regurgitation. The murmur of tricuspid regurgitation is classically holosystolic, but may also be early systolic if the right ventricular pressures are normal. The murmur of tricuspid regurgitation is similar to mitral regurgitation but can be distinguished by location (lower left second and third intercostal spaces) and by variability with respiration.

A previously healthy 30-year-old woman presents to your office with complaints of dysuria, flank pain, chills, and vomiting. Urine dipstick results are positive for nitrites and leukocyte esterase. Which of the following is the most appropriate therapy? A. Ciprofloxacin B. Fosfomycin C. Nitrofurantoin D. Penicilli

Urinary tract infections are classified as either cystitis, which is a bacterial infection affecting the bladder, or pyelonephritis, which is a bacterial infection affecting the kidneys. Bacteria generally reach the kidneys via the lower urinary tract; however, they may also reach the renal parenchyma through the bloodstream. Pyelonephritis is a potentially life-threatening infection in which timely diagnosis and treatment have a significant impact on outcomes. The majority of uncomplicated cases are caused by the bacteria Escherichia coli. Uncomplicated cases are defined as being in a patient who does not have an underlying condition that may cause therapy to fail, such as chronic kidney disease, pregnancy, poorly-controlled diabetes mellitus, indwelling catheter, or any immunocompromising condition. There is a high incidence of urinary tract infections in sexually active young women. The classic presentation of patients with pyelonephritis includes fever that may be above 103°F, costovertebral angle tenderness, and nausea or vomiting or both. Symptoms develop quickly, usually over the course of hours. Diagnosis is determined by history, physical exam and urinalysis results. Initial urine dipstick done in office will generally return positive for nitrites, leukocyte esterase, and hematuria. Urine specimens should then be sent to be cultured to determine the bacteria involved and antibiotic sensitivity. Previously healthy women with pyelonephritis may be managed as outpatients and oral empiric therapy with fluoroquinolones such as ciprofloxacin should be started immediately. Admission for pyelonephritis is indicated for patients with severe symptoms such as persistent vomiting, signs of dehydration, women who are pregnant, the elderly, the immunocompromised, or those with comorbid disorders such as chronic lung disease or diabetes mellitus. Fosfomycin (B) is a broad-spectrum antibiotic and nitrofurantoin (C) is a nitrofuran antimicrobial agent. It is recommended to avoid both of these medications in treating pyelonephritis due to their mechanism of action not being effective in reaching the renal tissues. Penicillin (D) does not play a role in the treatment of pyelonephritis. It is used in the treatment of syphilis, among other infections.


Related study sets

19 Socio - Religious Reform Movements

View Set

Chapter 46: Management of Patients With Gastric and Duodenal Disorders 2

View Set

Gr 6 Reading Review Part 2 "How Smart are Animals?"

View Set

(3) Chapter 17: Bipolar and Related Disorders *

View Set

Care of Patients with Hypersensitivity (Allergy) and Autoimmunity Ch 20 (Elsevier)

View Set